Pharmacology Exam 1

¡Supera tus tareas y exámenes ahora con Quizwiz!

5. A Nurse is working with a newly licensed nurse who is administering medications to a client. Which of the following actions should the nurse identify as an indication that the newly hired nurse understands medication error prevention? a. Taking all medications out of the unit- dos rappers before entering the client's room b. Checking the prescription when a single dose requires administration of multiple tablets c. Administering a medication, then looking up the usual dosage range d. Relying on another nurse to clarify a medication prescription

Answer: B ATI Chapter 2

3. a nurse on a medical surgical unit administers a hypnotic medication to an older adult client at 2100. The next morning, the client is drowsy and wants to sleep instead of eating breakfast. Which of the following factors should the nurse identify as a possible reason for the clients drowsiness? a. Reduced cardiac function b. first pass effect c. reduced hepatic function d. increase gastric motility

Answer: C ATI Chapter 6

5. A nurse is reviewing a client's medical record notes a new prescription for verifying the trough level of the client's medication. Which of the following actions should the nurse take? a. Obtain a blood specimen immediately prior to administering the next dose of medication b. Verify that the client has been taking the medication for 24 hours before obtaining a blood specimen c. Ask the client to provide a urine specimen after the next dose of medication d. Administer the medication, and obtain a blood specimen 30 minutes later.

Answer: a ATI Chapter 1

1. A provider prescribes phenobarbital to a client who has had a seizure disorder. The medication has a long half-life of 4 days. How many times per day should the nurse expect to administer this medication? a. 1 b. 2 c. 3 d. 4

Answer: a ATI Chapter 1

3. A nurse is assessing the IV catheter insertion site for a client receiving a non vesicant solution and notes swelling at the site with decreased skin temperature. Which of the following actions should the nurse take? Select all that apply a. Stop the infusion b. Start a new IV c. Apply warm compresses to the insertion site d. Evaluate the client's arm e. Obtain a specimen for culture at the insertion site

Answer: a, c, d ATI Chapter 4

4. A nurse is assessing a client before administering medications. Which of the following data should the nurse obtain? Select all that apply a. Use of herbal products b. Daily fluid intake c. Ability to swallow d. Previous surgical history e. Allergies

Answer: a, c, e ATI Chapter 2

2. a nurse is preparing to administer an IM dose of penicillin to a client who has a new prescription. The client states when they took penicillin three years ago, they developed a rash. Which of the following actions should the nurse take? a. Administer the prescribed dose b. Withhold the medication c. Ask the provider to change the prescription to an oral form d. Administer an oral antihistamine at the same time

Answer: b ATI Chapter 5

3. A nurse is reviewing a client's prescribed medications. Which of the following situations represents a contraindication to medication administration? a. Drink grapefruit juice, which could reduce a medications effectiveness b. Medication has orthostatic hypotension as an adverse effect c. Medication is approved for ages 12 and older, and the client is 8 years old d. An anti-anxiety medication that has adverse effect of drowsiness is described as a preoperative sedatives

Answer: c ATI Chapter 2

3) Which nursing diagnosis would be most appropriate for a patient who is receiving an opiate analgesic that depresses the central nervous system (CNS)? A) Risk for activity intolerance B) Risk for injury C) Impaired physical mobility D) Fatigue

B) Risk for injury Quiz 1 Health Care Quality and Safety

2) The nurse should withhold the drug and contact the prescriber if the patient reported an allergy to the drug with which symptom occurring shortly after the drug was taken? A) Constipation B) Wheezing C) Dry Mouth D) Vesicular rash

B) Wheezing Quiz 1 Health Care Quality and Safety

6. A nurse is preparing to administer Heparin 15,000 units subcutaneously every 12 hours. The amount available is heparin injection 20,000 units/mL. How many mL should the nurse administer per dose? Round the answer to the nearest tenth. Do not use a trailing zero

0.8 ATI Chapter 3

2. A nurse is infusing IV amphotericin B to a client who has a systemic fungal infection. The nurse should monitor the client for which of the following manifestations as an adverse effect of this medication? A. Hypoglycemia B. Constipation C. Fever D. Hyperkalemia

2. A. Amphotericin B can cause hyperglycemia. B. Amphotericin B can cause diarrhea. C. CORRECT: Amphotericin B can cause fever, chills, and nausea during the infusion. Pretreatment with diphenhydramine and acetaminophen can reduce these effects. D. Amphotericin B can cause hypokalemia. CHAPTER 47 MYCOBACTERIAL, FUNGAL, AND PARASITIC INFECTIONS

2. A nurse is assessing a client who has a severe infection and has been receiving cefotaxime for the past week. Which of the following findings indicates a potentially serious adverse reaction to this medication that the nurse should report to the provider? A. Diaphoresis B. Epistaxis C. Diarrhea D. Alopecia

2. A. Diaphoresis is not an adverse effect of cefotaxime. Common adverse effects include rashes, nausea, headache, dizziness, and weakness. B. Epistaxis is not an adverse effect of cefotaxime. C. CORRECT: Diarrhea is an adverse effect of cefotaxime and other cephalosporins that requires reporting to the provider. Severe diarrhea might indicate that the client has developed antibiotic-associated pseudomembranous colitis, which could be life-threatening. D. Alopecia is not an adverse effect of cefotaxime. CHAPTER 44 ANTIBIOTICS AFFECTING THE BACTERIAL CELL WALL

2. A nurse is administering gentamicin by IV infusion at 0900. The medication will take 1 hr to infuse. When should the nurse plan to obtain a blood sample for a peak blood level of gentamicin? A. 1000 B. 1030 C. 1100 D. 1130

2. A. The IV infusion should end at 1000, but that is not the time for the nurse to collect a blood specimen for the peak blood level. B. CORRECT: Obtain the blood specimen for the peak blood level at 1030, 30 min after the end of the IV infusion. For the trough level, collect the blood sample just before starting the infusion. C. Collecting the specimen for the peak blood level 1 hr following the end of the IV infusion would yield an inaccurate peak level. D. Collecting the specimen for the peak blood level 1.5 hr following the end of the IV infusion would yield an inaccurate peak level. CHAPTER 45 ANTIBIOTICS AFFECTING PROTEIN SYNTHESIS

3. A nurse is administering IV amphotericin B to a client who has a systemic fungal infection. The nurse should monitor which of the following laboratory values? (Select all that apply.) A. Blood albumin B. Blood amylase C. Blood potassium D. Hematocrit E. Blood creatinine

3. A. Amphotericin B does not affect blood albumin levels. B. Amphotericin B does not cause pancreatitis. C. CORRECT: Hypokalemia is a serious adverse effect of amphotericin B. Monitor blood potassium values for hypokalemia. D. CORRECT: Amphotericin B can cause bone marrow suppression. Monitor CBC and platelet count periodically. E. CORRECT: Amphotericin B can cause nephrotoxicity. Monitor kidney function (with blood creatinine, BUN, and creatinine clearance). CHAPTER 47 MYCOBACTERIAL, FUNGAL, AND PARASITIC INFECTIONS

4. A nurse is caring for a client who has a new prescription for enfuvirtide to treat HIV infection. The nurse should monitor the client for which of the adverse reactions of this medication? (Select all that apply.) A. Bleeding B. Pneumonia C. Cerebral edema D. Localized erythema E. Hypotension

4. A. Bleeding is not an adverse effect of enfuvirtide. B. CORRECT: Bacterial pneumonia with fever, cough, and difficulty breathing are manifestations of an adverse reaction to enfuvirtide. Assess breath sounds regularly. C. Cerebral edema is not an adverse reaction to enfuvirtide. D. CORRECT: Enfuvirtide is administered subcutaneously. Injection-site reactions (redness, itching, and bruising) are common. E. CORRECT: A systemic allergic reaction can occur when taking enfuvirtide. Manifestations of hypersensitivity include rash, hypotension, fever, and chills. CHAPTER 48 VIRAL INFECTIONS, HIV, AND AIDS

5. A nurse is teaching a client who has active tuberculosis about the treatment regimen. The client asks why four different medications are necessary. Which of the following responses should the nurse make? A. "Four medications decrease the risk for a severe allergic reaction." B. "Four medications reduce the chance that the bacteria will become resistant." C. "Four medications reduce the risk for adverse reactions" D. "Four medications decrease the chance of having a positive tuberculin skin test."

5. A. Taking several antituberculosis medications concurrently does not decrease the chance of an allergic reaction to any of the individual medications. B. CORRECT: If the client took only one medication to treat active tuberculosis, resistance to the medication would occur quickly. Taking three or four medications decreases the possibility of resistance. C. Taking several antituberculosis medications concurrently does not minimize the chance of adverse effects to any of the medications. Risk for liver toxicity increases when more than one medication that causes liver toxicity is taken (isoniazid, rifampin, and pyrazinamide). D. Taking several antituberculosis medications concurrently does not change the fact that the client will have a positive tuberculin skin test indefinitely. CHAPTER 47 MYCOBACTERIAL, FUNGAL, AND PARASITIC INFECTIONS

A primary care provider should prescribe a lower dose of aztreonam (Azactam) for a patient who has a respiratory tract infection and also has which of the following? a. glaucoma b. closed-head injury c. heart failure d. renal impairment

A primary care provider should prescribe a lower dose of aztreonam (Azactam) for a patient who has a respiratory tract infection and also has which of the following? a. glaucoma b. closed-head injury c. heart failure d. renal impairment ATI Infection

1. Which patient is the highest priority when providing nursing care to multiple patients? a. A patient who received oral drugs 1 hour ago and complains of tingling around his mouth. b. A patient who has been prescribed a drug the nurse is not familiar with. c. A patient who would like to know if a newly prescribed drug is available in generic form. d. A patient who has requested a laxative because he has not had a bowel movement in 2 days.

A. A patient who received oral drugs 1 hour ago and complains of tingling around his mouth. Quiz 1 Health Care Quality and Safety

9. A nurse is caring for a patient who has been taking low-dose aspirin for several days. The nurse notes that the patient has copious amounts of watery nasal secretions and an urticarial rash. The nurse will contact the provider to discuss: a. administering epinephrine. b. changing to a first-generation NSAID. c. reducing the dose of aspirin. d. giving an antihistamine.

ANS: A Aspirin can cause a hypersensitivity reaction in some patients. This may start with profuse watery rhinorrhea and progress to generalized urticaria, bronchospasm, laryngeal edema, and shock. It is not a true anaphylactic reaction, because it is not mediated by the immune system. Epinephrine is the treatment of choice. Patients with sensitivity to ASA often also have sensitivity to NSAIDs; the first indication with this patient is to treat the potential life-threatening effect, not to change the medication. Reduction of the dose of ASA is not indicated, because this reaction is not dose dependent. Antihistamines are not effective, because this is not an allergic reaction. REF: p. 854

11. A patient has been taking methadone [Dolophine] for 5 months to overcome an opioid addiction. The nurse should monitor the patient for which of the following electrocardiographic changes? a. Prolonged QT interval b. Prolonged P-R interval c. AV block d. An elevated QRS complex

ANS: A Methadone prolongs the QT interval. It does not prolong the P-R interval, cause AV block, or produce an elevated QRS complex. REF: p. 270

12. A patient who has developed opioid tolerance will experience which effect? a. Decreased analgesic effect b. Decreased constipation c. Increased euphoria d. Increased respiratory depression

ANS: A Patients who develop tolerance to opioids will develop tolerance to its analgesic, euphoric, and sedative effects and will also develop tolerance to respiratory depression. Very little tolerance develops to constipation. REF: p. 264

9. A patient taking high doses of a glucocorticoid develops weakness in the muscles of the upper arms and in the legs. What will the nurse do? a. Contact the provider to ask about reducing the dose. b. Encourage the patient to restrict sodium intake. c. Reassure the patient that this is an expected side effect. d. Tell the patient to stop taking the drug.

ANS: A High-dose glucocorticoid therapy can cause myopathy, manifesting as weakness. If muscle weakness occurs, the dose should be reduced. Reducing the sodium intake is recommended to minimize sodium and water retention. Muscle weakness is not an expected side effect, because it indicates myopathy. It is incorrect to tell the patient to stop taking the drug, because a glucocorticoid must be withdrawn slowly to allow time for recovery of adrenal function. REF: p. 873

1. Which statement is true regarding the patient's perception of his or her care? a. Patent perception is just as important as the outcome of care. b. Patient perception is insignificant compared to the outcome of care. c. Patient satisfaction has no relation to quality of care. d. Patient satisfaction is insignificant compared to the outcome of care.

ANS: A The patient's perception of his or her care is just as important as the outcome of the care. If the patient perceives the care as meeting the aspects of quality, then patient satisfaction increases. Giddens Health Care Quality

12. A nurse is discussing methicillin-resistant Staphylococcus aureus (MRSA) with a group of nursing students. Which statement by a student correctly identifies the basis for MRSA resistance? a. "MRSA bacteria have developed PBPs with a low affinity for penicillins." b. "MRSA bacteria produce penicillinases that render penicillin ineffective." c. "MRSA occurs because of host resistance to penicillins." d. "MRSA strains replicate faster than other Staphylococcus aureus strains."

ANS: A MRSA strains have a unique mechanism of resistance, which is the production of PBPs with a low affinity for penicillins and all other beta-lactam antibiotics. MRSA resistance is not related to beta-lactamase production. MRSA resistance refers to bacterial and not host resistance. The resistance of MRSA strains is not related to speed of replication. REF: p. 1017

9. A patient will receive buprenorphine [Butrans] as a transdermal patch for pain. What is important to teach this patient about the use of this drug? a. Avoid prolonged exposure to the sun. b. Cleanse the site with soap or alcohol. c. Remove the patch daily at bedtime. d. Remove hair by shaving before applying the patch.

ANS: A Patients using the buprenorphine transdermal patch should be cautioned against heat, heating pads, hot baths, saunas, and prolonged sun exposure. The skin should be cleaned with water only. The patch should stay on for 7 day REF: p. 273

A nursing student asks a nurse about pharmaceutical research and wants to know the purpose of randomization in drug trials. The nurse explains that randomization is used to do what? a . To ensure that differences in outcomes are the result of treatment and not differences in subjects b . To compare the outcome caused by the treatment to the outcome caused by no treatment c . To make sure that researchers are unaware of which subjects are in which group d . To prevent subjects from knowing which group they are in and prevent preconception bias

ANS: A Randomization helps prevent allocation bias, which can occur when researchers place subjects with desired characteristics in the study group and other subjects in the control group so that differences in outcome are actually the result of differences in subjects and not treatment. Comparing treatment outcome to no treatment outcome is the definition of a controlled study. The last two options describe the use of blinding in studies; blinding ensures that researchers or subjects (or both) are unaware of which subjects are in which group so that preconceptions about benefits and risks cannot bias the results. page 15-16

1. A patient has a Pseudomonas aeruginosa infection that is sensitive to aminoglycosides, and the prescriber orders gentamicin. The patient tells the nurse that a friend received amikacin [Amikin] for a similar infection and wonders why amikacin was not ordered. What will the nurse tell the patient? a. "Amikacin is given when infectious agents are resistant to other aminoglycosides." b. "Amikacin is more vulnerable to inactivation by bacterial enzymes." c. "Amikacin is a narrow-spectrum drug and will probably not work for this infection." d. "Gentamicin is less toxic to the ears and the kidneys."

ANS: A Resistance to amikacin is uncommon at this point; to minimize the emergence of amikacin-resistant bacteria, this drug is reserved for infections in which resistance to other aminoglycosides has developed. Amikacin is the least susceptible to inactivation by bacterial enzymes. Amikacin is a broad-spectrum antibiotic. All aminoglycosides are ototoxic and nephrotoxic. REF: p. 1050 | p. 1055

17. A patient is receiving tobramycin three times daily. A tobramycin peak level is 4.5 and the trough is 1.2. What will the nurse do? a. Give the next dose as ordered. b. Hold the next dose and notify the provider. c. Monitor the patient for signs of nephrotoxicity. d. Tell the patient to report tinnitus.

ANS: A These levels are within normal limits, so the next dose may be given safely. It is not necessary to withhold the next dose. These levels do not indicate any increased risk of nephrotoxicity or ototoxicity. REF: p. 1052 / p. 1054

1. A patient asks the nurse what can be given to alleviate severe, chronic pain of several months' duration. The patient has been taking oxycodone [OxyContin] and states that it is no longer effective. The nurse will suggest discussing which medication with the provider? a. Fentanyl [Duragesic] transdermal patch b. Hydrocodone [Vicodin] PO c. Meperidine [Demerol] PO d. Pentazocine [Talwin] PO

ANS: A Transdermal fentanyl is indicated only for persistent, severe pain in patients already opioid tolerant. Hydrocodone, a combination product, has actions similar to codeine and is not used for severe, chronic pain. Meperidine is not recommended for continued use because of the risk of harm caused by the accumulation of a toxic metabolite. Pentazocine is an agonist-antagonist opioid and is less effective for pain; moreover, when given to a patient who is already opioid tolerant, it can precipitate an acute withdrawal syndrome. REF: p. 267 / pp. 270-271 / pp. 272-273

7. A patient will be discharged home to complete treatment with intravenous cefotetan with the assistance of a home nurse. The home care nurse will include which instruction when teaching the patient about this drug treatment? a. Abstain from alcohol consumption during therapy. b. Avoid dairy products while taking this drug. c. Take an antihistamine if a rash occurs. d. Use nonsteroidal anti-inflammatory drugs (NSAIDs), not acetaminophen, for pain.

ANS: A Two cephalosporins, including cefotetan, can induce a state of alcohol intolerance and cause a disulfiram-like reaction when alcohol is consumed; therefore, patients should be advised to avoid alcohol. It is not necessary to avoid dairy products. Patients who experience a rash should report this to their provider. Cefotetan can also promote bleeding, so drugs that inhibit platelet aggregation should be avoided. REF: p. 1027

The nurse is providing multiple medications to a patient whose spouse brings grapefruit juice every morning. The nurse will be concerned about which classes of drugs? (Select all that apply.) a. Calcium channel blockers b. Selective serotonin reuptake inhibitors c. Aminoglycosides d. Beta blockers e. Penicillins

ANS: A, B Calcium channel blockers and selective serotonin reuptake inhibitors have been shown to reach increased and/or toxic levels when taken with grapefruit juice. Grapefruit juice is not contraindicated with aminoglycosides, beta blockers, or penicillins. page 59-60

3. A patient is being treated with an antibiotic for an infected orthopedic injury. What explanation should the nurse give to the patient about this medication? a. "Antibiotics will decrease the pain at the site." b. "An antibiotic helps to kill the infection causing the inflammation." c. "An antibiotic inhibits cyclooxygenase, an enzyme in the body." d. "Antibiotics will reduce the patient's fever."

ANS: B Antimicrobials treat the underlying cause of the infection which leads to inflammation. Analgesics and nonsteroidal anti-inflammatory drugs (NSAIDs) help to treat pain. NSAIDs and other antipyretics are cyclooxygenase inhibitors. Antipyretics help to reduce fever. Chapter 23 Giddens

11. A patient is taking erythromycin ethylsuccinate for a chlamydial infection and develops vaginal candidiasis. The prescriber orders ketoconazole to treat the superinfection. What will the nurse do? a. Administer the erythromycin and the ketoconazole as ordered. b. Contact the provider to discuss changing to a different antifungal medication. c. Contact the provider to discuss increasing the dose of erythromycin. d. Contact the provider to suggest using erythromycin stearate.

ANS: B Erythromycin can prolong the QT interval when present in large concentrations. When erythromycin is combined with a CYP3A4 inhibitor, such as ketoconazole, the risk of sudden cardiac death increases fivefold. The nurse should discuss changing the antifungal medication to one that is not a CYP3A4 inhibitor. It is not correct to give the ketoconazole without questioning the order. Increasing the dose of erythromycin would increase the risk of QT prolongation. Changing to a different preparation of erythromycin would not alter the risk. REF: p. 1041

4. Prior to drug administration the nurse reviews the seven rights, which include right patient, right medication, right time, right dose, right education, right documentation, and what other right? a. Room b. Route c. Physician d. Manufacturer

ANS: B The right route (e.g., oral or intramuscular) is an essential component to verify prior to the administration of any drug. The patient does not need to be in a specific location. There may be a number of physicians caring for a patient who prescribe medications for any given patient. A similar drug may be made by a number of different companies, and checking the manufacturer is not considered one of the seven rights. However, the nurse will want to be aware of a difference, because different companies prepare the same medication in different ways with different inactive ingredients, which can affect patient response. Giddens Safety

13. A patient received 500 mg of azithromycin [Zithromax] at 0800 as a first dose. What are the usual amount and time of the second dose of azithromycin? a. 250 mg at 2000 the same day b. 500 mg at 2000 the same day c. 250 mg at 0800 the next day d. 500 mg at 0800 the next day

ANS: C Azithromycin generally is given as 500 mg on the first day and then 250 mg/day for the next 4 days, so the second dose would be 24 hours after the first dose. REF: p. 1041

11. A patient shows signs and symptoms of conjunctivitis. Which aminoglycoside would the nurse expect to be ordered? a. Amikacin [Amikin] b. Kanamycin [Kantrex] c. Neomycin [Neomycin] d. Paromomycin [Humatin]

ANS: C Neomycin is used for topical treatment of infections of the eye, ear, and skin. Amikacin, kanamycin, and paromomycin are not topical treatments and are not indicated for eye infections. REF: p. 1055

2. Which cephalosporin may be used to treat meningitis? a. Cefaclor b. Cefazolin c. Cefoxitin d. Cefotaxime

ANS: D Cefotaxime has increased ability to reach the cerebrospinal fluid (CSF) and to treat meningitis. Cefaclor, cefazolin, and cefoxitin do not reach effective concentrations in the CSF. REF: pp. 1025-1026

7. A patient is about to receive prednisone for tendonitis. The nurse reviewing the chart would be concerned about which of the following in the patient's medical history? a. Asthma and allergic rhinitis b. Gouty arthritis c. Seborrheic dermatitis d. Systemic fungal infection

ANS: D Glucocorticoids are contraindicated in patients with a history of systemic fungal infections. Glucocorticoids are used to treat asthma, allergic rhinitis, gout, and seborrheic dermatitis. REF: pp. 877-878

1. The nurse has been caring for a patient who has been taking antibiotics for 3 weeks. Upon assessing the patient, the nurse notices the individual has developed oral thrush. What describes the etiology of the thrush? a. Antibiotic resistance b. Community-acquired infection c. Nosocomial infection d. Superinfection

ANS: D Oral thrush is a manifestation of a superinfection. The development of thrush is not a symptom of antibiotic resistance. Oral thrush typically is not a community-acquired infection. The development of thrush is not a nosocomial infection. REF: p. 1005

8. Before administering celecoxib [Celebrex], it is most important for the nurse to assess the patient for a history of what? a. Allergy to sulfonamides b. History of hepatitis C c. Hypothyroidism d. Diabetes mellitus

a. Allergy to sulfonamides Celecoxib contains a sulfur molecule and therefore can precipitate an allergic reaction in patients allergic to sulfonamides. Accordingly, the drug should be avoided by patients with a sulfa allergy. The other conditions listed should be part of the nurse's assessment but are not the most important. Chapter 71

9. The nursing student learns that not all drugs produce effects binding to a receptor. Which drugs do not act through receptors? a. Antacids b. Analgesics c. Antihistamines d. Steroid hormones

a. Antacids Antacids do not act through receptors. Antacids neutralize gastric activity by direct chemical interaction with stomach acid. Chapter 5

1. The nurse is reviewing the therapeutic effects of nonsteroidal anti-inflammtory drugs (NSAIDS) which should include which of the following effects? a. Antipyretic b. Anxiolytic c. Sedative d. Antimicrobial

a. Antipyretic Quiz 3 Inflammation and Immunity

3. The nurse is administering warfarin, an anticoagulant, to a patient with a low albumin level. Which effect of this medication should the nurse expect to observe? a. Increased PT/INR levels b. Deep vein thromboses c. Reduced risk of bruising d. Increased platelet aggregation

a. Increased PT/INR levels Warfarin is an anticoagulant with a high affinity for binding with albumin. If the albumin level is low, more free drug is available for action, resulting in an increased prothrombin time/international normalized ratio (PT/INR). Deep vein thromboses can be prevented with warfarin. An increased risk of bruising and bleeding would occur with more free drug available. Warfarin acts on vitamin K, not on platelets. Aspirin is an example of an antiplatelet aggregator. Chapter 4

4. The nurse is concerned with minimizing adverse drug-drug interactions for the patient. Which drug characteristic could result in the most serious consequences from a drug-drug interaction? a. Low therapeutic range b. High biologic half-life c. Low potency d. First-pass effect

a. Low therapeutic range Interactions are especially important with drugs that have a narrow therapeutic range, because an interaction that produces a modest increase in drug levels can cause toxicity. Chapter 6

2. What is a result of the Prescription Drug User Fee Act (PDUFA) of 1992? a. New drugs reach the market sooner than ever before. b. Controlled substances are categorized into schedules. c. Drugs for acquired immunodeficiency syndrome (AIDS) receive accelerated approval. d. More research is conducted on drug safety in children.

a. New drugs reach the market sooner than ever before. PDUFA was a response to complaints about the length of time required for approval of new drugs by the U.S. Food and Drug Administration (FDA). Under this act, drug sponsors pay fees to the FDA that is used to fund additional reviewers. The FDA then must adhere to strict turnaround times for drug review processes. Chapter 3

9. What is the objective of drug therapy? a. To provide maximum benefit with minimal harm b. To provide minimum benefit with maximum harm c. To provide total relief of symptoms regardless of harm d. To provide as much benefit as possible

a. To provide maximum benefit with minimal harm The objective of drug therapy is to provide maximum benefit with minimal harm. All drugs have the potential to produce side effects. Because drugs are not ideal, we must exercise skill and care if treatment is to result in more good than harm. Chapter 1

4. A nurse prepares to administer acetaminophen [Tylenol] to a patient with an oral temperature of 101.7°F. Which preparation would the nurse expect to have the most rapid onset of action? a. Tylenol elixir b. Tylenol tablets c. Tylenol capsules d. Tylenol gel caps

a. Tylenol elixir A liquid does not have to dissolve first to allow absorption; therefore, the onset of action occurs more quickly than with capsules, tablets, or gel caps. Chapter 4

2. Which side effects of opioid analgesics can have therapeutic benefits? (Select All That Apply) a. Vasodilation b. Urinary retention c. Cough suppression d. Biliary colic e. Suppression of bowel motility

a. Vasodilation c. Cough suppression e. Suppression of bowel motility Quiz 2 Pain and Thermoregulation

A health care professional is caring for a patient who is about to receive gentamicin to treat a systemic infection. The health care professional should question the use of the drug for a patient who is also taking which of the following drugs? a. Furosemide b. diphenhydramine c. acetaminophen d. levothyroxine

a. furosemide Rational: Gentamicin, an aminoglycoside, and furosemide, and high-ceiling diuretic, are ototoxic drugs. ATI Infection

A health care professional is caring for a patient who is about to begin take isoniazid (INH) to treat tuberculosis. The health care professional should tell the patient to report which of the following adverse effects of the drug? (Select all that apply.) a. Jaundice b. numbness of the hands c. dizziness d. hearing loss e. oral ulcers

a. jaundice b. numbness of the hands c. dizziness Rational: Isoniazid, an antimycobacterial drug, can cause liver toxicity, especially in patients who abuse alcohol. Isoniazid can cause peripheral neuropathy, dizziness, ataxia, and seizures. ATI Infection

A health care professional is caring for a patient who is about to begin taking metronidazole (Flagyl) to treat an anaerobic intra-abdominal bacterial infection. The health care professional should recognize that cautious use of the drug is indicated if the patient also has which of the following? a. seizure disorder b. hearing loss c. asthma d. anemia

a. seizure disorder Rational: Metronidazole, an antiparasitic drug can cause ataxia, vertigo, and seizures. It requires cautions use with patients who have a history of seizure activity, liver or renal failure, or heart failure ATI Infection

A health care professional is caring for a patient who is about to begin using nystatin to treat oral Candida albicans that resulted from tetracycline therapy. Which of the following instructions should the health care professional include about using the antifungal preparation? a. swish the suspension in the mouth before swallowing it b. rinse the mouth immediately with water c. drink the suspension with a straw d. use cotton-tipped applicator to dab the mouth lesions with the suspension

a. swish the suspension in the mouth before swallowing it Rational: Tell the patient to swish the nystatin oral suspension inside the mouth for at least 2 min and then swallow it. This allows the drug to coat the inside of the mouth, where it exerts its therapeutic effect against Candida albicans lesions. ATI Infection

6. Before administrating the dosage of a prescribed medication, the nurse observes precipitation formation of the intravenous (IV) solution. What is the priority nursing action? a. Verify the prescription. b. Discard the IV solution. c. Prepare another dose to administer. d. Check the expiration date of the drug.

b. Discard the IV solution. If a precipitate appears in the IV solution, that solution should be discharged. Chapter 6

3. Which aspect of drug therapy indicates to the nurse whether a drug is having a beneficial effect? a. Performing a preadministration assessment b. Evaluating therapeutic responses c. Minimizing adverse effects d. Managing toxicity

b. Evaluating therapeutic responses Evaluation is one of the most important aspects of drug therapy, because it tells the nurse whether a drug is having its intended effect. The other aspects of drug administration are important but do not give information about a drug's effectiveness. Chapter 2

2. The nurse is preparing to give a medication for pain. The label states that the drug is "lipid soluble." How soon should the nurse expect to observe the effects of the drug? a. Slowly b. Rapidly c. Unpredictably d. Variably

b. Rapidly Cell membranes are composed of lipids; therefore, a lipid-soluble drug passes through rapidly. A water-soluble drug passes through more slowly. The nurse would expect to observe the effects of a lipid-soluble drug more quickly, because the drug is absorbed more rapidly. Chapter 4

12. Which statements about the care of a patient with aspirin poisoning does the nurse identify as true? (Select all that apply.) a. Warming blankets are routinely used to raise the patient's temperature. b. Diuretics and fluid restrictions are needed to correct the fluid overload commonly seen with aspirin poisoning. c. Bicarbonate infusions are used to reverse acidosis and promote renal excretion of salicylates. d. Activated charcoal is contraindicated in the treatment of aspirin poisoning. e. Hemodialysis or peritoneal dialysis can accelerate salicylate removal.

c. Bicarbonate infusions are used to reverse acidosis and promote renal excretion of salicylates. e. Hemodialysis or peritoneal dialysis can accelerate salicylate removal. Aspirin poisoning is an acute medical emergency that requires hospitalization. Treatment is largely supportive and consists of external cooling (eg, sponging with tepid water), infusion of fluids (to correct dehydration and electrolyte loss), infusion of bicarbonate (to reverse acidosis and promote renal excretion of salicylates), and mechanical ventilation (if respiration is severely depressed). Absorption of aspirin can be reduced by gastric lavage and by giving activated charcoal. If necessary, hemodialysis or peritoneal dialysis can accelerate salicylate removal. Chapter 71

1. Which legislation sets rules for the manufacture and distribution of drugs considered to have the potential for abuse? a. Food, Drug, and Cosmetic Act of 1938 b. Harris-Kefauver Amendments of 1962 c. Controlled Substances Act of 1970 d. Food and Drug Administration Modernization Act of 1997

c. Controlled Substances Act of 1970 The Controlled Substances Act of 1970 sets rules covering drugs of abuse and defined categories of controlled substances. Chapter 3

7. Which term is commonly used for nonprescription drugs? a. Legend b. Generic c. Over-the-counter d. Pharmaceutical

c. Over-the-counter Over-the-counter (OTC) drugs are also known as nonprescription drugs. Chapter 3

4. Which drug property is most enhanced by the presence of many different types of receptors throughout the body? a. Potency b. Safety c. Selectivity d. Convenience

c. Selectivity Because each receptor regulates just a few processes, selective drug action is possible. Multiple types of receptors do not have as much effect on potency, safety, or convenience. Chapter 5

4. Tetracycline can cause stomach discomfort and esophageal issues (ie esophageal ulceration).What is the best teaching strategy used by the nurse for a patient prescribed this medication? a. Take the medication right before bed b. Take the medication with an antacid c. Stay upright for 30 minutes following the medication d. Take the medication with a glass of milk

c. Stay upright for 30 minutes following the medication Quiz 4 Infection

1. A patient who has developed an opioid tolerance will experience which effect? a) Increased euphoria b) Increased respiratory depression c) Decreased constipation d) Decreased analgesic effect

d) Decreased analgesic effect Quiz 2 Pain and Thermoregulation

7. For which reason should the nurse follow safe medication administration for intravenous (IV) medications? a. The IV route can result in delayed absorption of the medication. b. The IV route results in a delayed onset of action. c. Control over the levels of drug in the body is unpredictable. d. IV administration is irreversible.

d. IV administration is irreversible. The IV route allows precise control over levels of drug in the blood and results in rapid onset of action. Absorption of IV medication is instantaneous and complete. Once a drug has been injected, there is no turning back; the drug is in the body and cannot be retrieved. Chapter 4

While administering IV cefotetan to a patient to treat bacterial meningitis, the health care professional finds the IV insertion site warm and reddened. Which of the following actions should the health care professional take? a. slow the cefotetan infusion b. administer diphenhydramine c. request a prescription for another antibiotic d. stop the cefotetan infusion

d. stop the cefotetan infusion Rational: The health care professional should stop the infusion, remove the IV catheter, assess for tissue damage, and treat it accordingly. ATI Infetion

A health care professional is caring for a patient who is taking ciprofloxacin (Cipro) to treat a urinary tract infection and has rheumatoid arthritis, for which he takes prednisolone (Prelone). Recognizing the adverse effects of ciprofloxacin, the health care professional should tell the patient to report which of the following? a. tachycardia b. hair loss c. insomnia d. tendon pain

d. tendon pain Rational: Ciprofloxacin, a fluoroquinolone, can cause an Achilles tendon to rupture, especially in patients who take glucocorticoids or in older adults. The health care professional should tell the patient to report tendon pain, and stop taking the drug ATI Infection

2. A nurse is preparing to administer clindamycin 200 mg by intermittent IV bolus. The amount available is clindamycin injection 200 mg in 100 ml 0.9% sodium chloride (0.9%NaCl) to infuse over 30 minutes. The nurse should set the IV pump to deliver how many mL/hr? Round to the nearest whole number. Do not use a trailing zero

200 ATI Chapter 3

1. A nurse is preparing to administer Vancomycin 1 G by intermittent IV bolus. Available is Vancomycin 1G in 100 ml of dextrose 5% in water ( D5W) to infuse over 45 minutes. The drop factor of the manual IV tubing is 10 gtt/mL. The nurse should have just the manual IV infusion to deliver how many gtt/min? Round to the nearest whole number. Do not use a trailing zero.

22 gtt/min ATI Chapter 3

2. A nurse educator is reviewing medication metabolism at an in-service presentation which of the following factors should the educator include as a reason to administer lower medication dosages? (Select all that apply) a. Increased renal excretion b. Increase medication- metabolizing enzymes c. Liver failure d. Peripheral vascular disease e. Concurrent use of medication the same pathway metabolizes

Answer: c ATI Chapter 1

4. All of the following are temperature reducing responses to the process of fever except: a. Sweating b. Shivering c. Vasodilation d. Increased respiratory rate

b. Shivering Quiz 2 Pain and Thermoregulation

5. A nurse is preparing to administer amoxicillin 20mg/kg/day PO 2 divided equally every 12 hr to a preschooler who weighs 44 lb. Amount available is amoxicillin suspension 250 mg/5mL. How many mL should the nurse administer per dose? Round the answer to the nearest whole number. Do not use a trailing zero.

4 ATI Chapter 3

1. A nurse is teaching a client about taking tetracycline to treat a GI infection due to Helicobacter pylori. Which of the following statements should the nurse identify as an indication that the client understands the instructions? A. "I will take this medication with 8 ounces of milk." B. "I will let my doctor know if I start having diarrhea." C. "I can stop taking this medication when I feel completely well." D. "I can take this medication just before bedtime."

A. Tetracycline can form a nonabsorbable chelate if clients take it with dairy products. They should take it with water on an empty stomach. B. CORRECT: Diarrhea can indicate that the client is developing a suprainfection, which can be very serious. The client should notify the provider if diarrhea occurs. C. The client should take the full prescription of tetracycline and not stop the medication if he begins to feel well. D. Taking tetracycline in the morning helps prevent esophageal ulceration, which can occur if the client takes it just before lying down CHAPTER 45 ANTIBIOTICS AFFECTING PROTEIN SYNTHESIS

2. Which infection(s) may be treated with linezolid [Zyvox])? (Select all that apply.) a. Community-acquired pneumonia (CAP) that is penicillin sensitive b. Nosocomial pneumonia caused by methicillin-sensitive Staphylococcus aureus (MSSA) c. Pneumonias caused by Mycoplasma avium d. Superficial methicillin-resistant Staphylococcus aureus skin infections (MRSA) e. Vancomycin-resistant infections

ANS: A, B, E Linezolid is indicated for CAP caused by PCN-sensitive strains of Streptococcus pneumoniae, nosocomial pneumonia caused by MSSA and MRSA, and vancomycin-resistant enterococcal (VRE) infections. It is not recommended for M. avium infections or for superficial skin infections caused by MRSA. REF: p. 1043

1. Tetracyclines are considered first-line drugs for which disorder(s)? (Select all that apply.) a. Chlamydia trachomatis cervicitis b. Clostridium difficile diarrhea c. Lyme disease d. Methicillin-resistant Staphylococus aureus skin infections e. Typhus fever

ANS: A, C, E Tetracyclines are drugs of first choice for rickettsial diseases, infections caused by C. trachomatis, brucellosis, cholera, Mycoplasma pneumonia, Lyme disease, anthrax, and gastric infections caused by H. pylori. They are not first-line drugs for CDAD or MRSA skin infections. REF: p. 1037

1. The nurse assesses the patient and notes all of the following. Select all of the findings that indicate the systemic manifestations of inflammation. a. Oral temperature 38.6° C/101.5° F b. Thick, green nasal discharge c. Patient complaint of pain at 6 on a 0 to 10 scale on palpation of frontal and maxillary sinuses d. WBC 20 cells/McL 109/L e. Patient reports, "I'm tired all the time. I haven't felt like myself in days."

ANS: A, D, E Systemic manifestations of inflammatory response include elevated temperature, leukocytosis, and malaise and fatigue. Purulent exudates and pain are both considered local manifestations of inflammation. Chapter 23 Giddens

1. Which disorders may be treated with glucocorticoids? (Select all that apply.) a. Allergic rhinitis b. Anaphylaxis c. Bronchiolitis d. Gout e. Ulcerative colitis

ANS: A, D, E Glucocorticoids are used topically for allergic rhinitis and are given systemically for gout and ulcerative colitis. They are not useful in anaphylactic reactions, for which epinephrine is the drug of choice, or in bronchiolitis, which generally is induced by a virus, not by an immune response. REF: p. 871

3. A nurse is teaching a client about transdermal patches. Which of the following statements should the nurse identify as an indication that the client understands? a. I will clean the site with an alcohol swab before I apply the patch b. I will rotate the application sites weekly c. I will apply the patch to an area of skin with no hair d. I will place the new patch on the site of the old patch

Answer: c ATI Chapter 1

A nurse is teaching a patient about a prescription for a monoamine oxidase (MAO) inhibitor for depression. What will the nurse teach the patient to avoid while taking this drug? a. Alcoholic beverages b. Aged cheeses c. Brussels sprouts and cabbage d. Grapefruit juice

ANS: B Aged cheeses are rich in tyramine, which interacts with MAO inhibitors to raise blood pressure to life-threatening levels. Patients taking MAO inhibitors should be taught to avoid tyramine-rich foods. Chianti wine contains tyramine, but other alcoholic beverages do not. Brussels sprouts and cabbage are foods rich in vitamin K, which can interfere with the effects of warfarin. Grapefruit juice inhibits CYP3A4 and interferes with the metabolism of many medications. page 60

A nurse is teaching a patient about a drug that induces P-glycoprotein. The nurse will explain that this drug may cause which effect on other drugs? a. Decreased absorption in the intestines b. Decreased elimination through the kidneys c. Increased brain exposure d. Increased fetal absorption

ANS: B Drugs that induce PGP can increase drug export from cells of the intestinal epithelium into the intestinal lumen, thus decreasing absorption of the drug. PGP inducers also increase drug elimination and decrease brain and fetal drug exposure. page 58

1. The nurse is caring for a patient with a diagnosed case of Clostridium difficile. The nurse expects to implement which of the following interventions? (Select all that apply.) a. Administration of protease inhibitors b. Use of personal protective equipment c. Patient teaching on methods to inhibit transmission d. Preventing visitors from entering the room e. Administration of intravenous fluids f. Strict monitoring of intake and output

ANS: B, C, E, F Protease inhibitors are used for treatment of viral infections, not bacterial infections. The nurse wants to protect visitors from exposure to the bacteria and protect the patient from secondary infection while immunocompromised, but the patient will need the support of family and close friends. Contact isolation precautions must be strictly followed along with the use of personal protective equipment and teaching on methods to inhibit transmission to help break the chain of infection. Intravenous fluids and strict intake and output monitoring will be important for the patient suffering the effects of Clostridium difficile, because it causes diarrhea with fluid loss. Chapter 24 Giddens

1. Which organisms can be treated with penicillin G (Benzylpenicillin)? (Select all that apply.) a. Methicillin-resistant Staphylococcus aureus b. Neisseria meningitidis c. Pseudomonas aeruginosa d. Streptococcus pyogenes e. Treponema pallidum

ANS: B, D, E Penicillin G is the first drug of choice for N. meningitidis. It is a drug of first choice for infections caused by sensitive gram-positive cocci, including S. pyogenes. It is a drug of choice for T. pallidum. It is not effective against methicillin-resistant S. aureus or P. aeruginosa. REF: pp. 1018-1019

1. The parents of a newborn question the nurse about the need for vaccinations: "Why does our baby need all those shots? He's so small, and they have to cause him pain." The nurse can explain to the parents that which of the following are true about vaccinations? (Select all that apply.) a. Are only required for infants b. Are part of primary prevention for system disorders c. Prevent the child from getting childhood diseases d. Help protect individuals and communities e. Are risk free f. Are recommended by the Centers for Disease Control and Prevention (CDC)

ANS: B, D, F Immunizations are considered part of primary prevention, help protect individuals from contracting specific diseases and from spreading them to the community at large, and are recommended by the CDC. Immunizations are recommended for people at various ages from infants to older adults. Vaccination does not guarantee that the recipient won't get the disease, but it decreases the potential to contract the illness. No medication is risk free. Chapter 22 Giddens

2. While reviewing the complete blood count (CBC) of a patient on her unit, the nurse notes elevated basophil and eosinophil readings. The nurse realizes that this is most indicative of which type of infection? a. Bacterial b. Fungal c. Parasitic d. Viral

ANS: C Parasitic infections are frequently indicated on a CBC by elevated basophil and eosinophil levels. Bacterial infections do not lead to elevated basophil and eosinophil levels but elevated B and T lymphocytes, neutrophils, and monocytes. Fungal infections do not lead to elevated basophil and eosinophil levels. Viral infections create elevations in B and T lymphocytes, neutrophils, and monocytes. Chapter 24 Giddens

9. A 6-week-old infant who has not yet received immunizations develops a severe cough. While awaiting nasopharyngeal culture results, the nurse will expect to administer which antibiotic? a. Clindamycin [Cleocin] b. Doxycycline [Vibramycin] c. Erythromycin ethylsuccinate d. Penicillin G

ANS: C Erythromycin is the drug of first choice for infections caused by Bordetella pertussis, the causative agent of whooping cough. Infants who have not received their first set of immunizations are at increased risk of pertussis. Clindamycin, doxycycline, and penicillin are not recommended. REF: pp. 1040-1041

1. A patient who is receiving a final dose of intravenous (IV) cephalosporin begins to complain of pain and irritation at the infusion site. The nurse observes signs of redness at the IV insertion site and along the vein. What is the nurse's priority action? a. Apply warm packs to the arm, and infuse the medication at a slower rate. b. Continue the infusion while elevating the arm. c. Select an alternate intravenous site and administer the infusion more slowly. d. Request central venous access.

ANS: C These signs indicate thrombophlebitis. The nurse should select an alternative IV site and administer the infusion more slowly. The IV should not be continued in the same site, because necrosis may occur. A central line would be indicated only for long-term administration of antibiotics. REF: p. 1027

1. A sentinel event refers to which situation? a. An event that could have harmed a patient, but serious harm didn't occur because of chance. b. An event that harms a patient as a result of underlying disease or condition. c. An event that harms a patient by omission or commission, not an underlying disease or condition. d. An event that signals the need for immediate investigation and response.

ANS: D A sentinel event is an unexpected occurrence involving death or serious physical or psychologic injury or the risk thereof called sentinel, because they signal the need for immediate investigation and response. A near-miss refers to an error or commission or omission that could have harmed the patient, but serious harm did not occur as a result of chance. Harm that relates to an underlying disease or condition provides the rationale for the close monitoring and supervision provided in a health care setting. An adverse event is one that results in unintended harm because of the commission or omission of an act. Giddens Safety

8. The nurse planning care for a patient with hypothermia should consider what similar exemplar? a. Heat exhaustion b. Heat stroke c. Infection d. Prematurity

ANS: D Prematurity, frost bite, environmental exposure, and brain injury are considered exemplars of hypothermia. Heat exhaustion is an exemplar of hyperthermia. Heat stroke is an exemplar of hyperthermia. Infection is an exemplar of hyperthermia. Chapter 10 Giddens

5. A nurse preparing to administer intravenous gentamicin to a patient notes that the dose is half the usual dose for an adult. The nurse suspects that this is because this patient has a history of: a. antibiotic resistance. b. interpatient variation. c. liver disease. d. renal disease.

ANS: D The aminoglycosides are eliminated primarily by the kidneys, so in patients with renal disease, doses should be reduced or the dosing interval should be increased to prevent toxicity. Patients with antibiotic resistance would be given amikacin. Interpatient variation may occur but cannot be known without knowing current drug levels. Aminoglycosides are not metabolized by the liver, so liver disease would not affect drug levels. REF: p. 1052

2. A 6-year-old child who has chickenpox also has a fever of 102.9° F (39.4° C). The child's mother asks the nurse if she should use aspirin to reduce the fever. What is the best response by the nurse? a. "Acetaminophen (Tylenol) should be used to reduce his fever, not aspirin." b. "You can use the aspirin, but watch for worsening symptoms." c. "You can use aspirin, but be sure to follow the instructions on the bottle." d. "It's best to wait to see if the fever gets worse."

a. "Acetaminophen (Tylenol) should be used to reduce his fever, not aspirin." Quiz 3 Inflammation and Immunity

5. The nurse is teaching a patient prescribed felodipine [Plendil] for the treatment of hypertension. Which statement is the most appropriate for the nurse to include in the teaching session? a. "Avoid grapefruit juice while you are taking this medication." b. "Do not eat foods high in tyramine such as aged cheese." c, "Herbal agents can help this drug work more effectively." d. "This drug is free of dangerous drug interactions."

a. "Avoid grapefruit juice while you are taking this medication." Grapefruit juice can raise levels of felodipine by as much as 406% because of the effect grapefruit juice has on the CYP3A4 isoenzyme. Chapter 6

3. Which statement by the nursing student indicates a need for further instruction about drug selectivity? a. "Botulinum toxin is very selective and therefore very safe for administration." c. "The more selective a drug is, the fewer side effects it will produce." d. "Even though a drug is selective, it can produce nonselective results."

a. "Botulinum toxin is very selective and therefore very safe for administration." Selectivity does not guarantee safety. Botulinum toxin can cause paralysis of respiratory muscles, resulting in death from respiratory arrest. All of the remaining statements about receptors and selectivity of drug action are correct. Chapter 5

10. The nurse should instruct a patient complaining of pain to do what to reduce fluctuations in drug levels? a. "Take pain medication around the clock at specified intervals and doses." b. "Take pain medication when the pain level reaches an 8 or 9 on a 1 to 10 scale." c. "Take pain medication at night before bed and avoid daytime dosing because of drowsiness." d. "Take pain medication after breakfast and dinner to reduce stomach upset."

a. "Take pain medication around the clock at specified intervals and doses." One technique to reduce drug level fluctuations is to take a specified dose at reduced dosing intervals. A patient who waits for the pain to peak will have to wait longer for the pain medicine to reach a plateau level of pain control. Avoiding daytime dosing because of drowsiness and avoiding stomach upset does not address the question of how to reduce fluctuations in drug levels. Chapter 4

5. A nurse provides discharge instructions for a patient who is taking acetaminophen [Tylenol] after surgery. The nurse should instruct the patient to avoid which product while taking acetaminophen? a. Alcoholic drinks b. Leafy green foods c. Bananas d. Dairy products

a. Alcoholic drinks Through several mechanisms, regular alcohol consumption while taking acetaminophen [Tylenol] increases the risk of liver injury when dosages are excessive. Therapeutic doses of acetaminophen [Tylenol] may be safe for patients who drink alcohol; however, the U.S. Food and Drug Administration (FDA) requires that acetaminophen [Tylenol] labels state an alcohol warning for patients who consume three or more drinks a day to consult their prescriber to determine whether acetaminophen [Tylenol] can be taken safely. It is not necessary to avoid leafy green foods, bananas, or dairy products when taking acetaminophen. Chapter 71

7. The nurse is reading a genetic research study. The study discusses how genetic variants can directly affect the metabolism of clopidogrel [Plavix], reducing the antiplatelet response. The nurse understands reduced efficacy of clopidogrel can increase the risk of which cardiovascular event? (Select all that apply.) a. Stroke b. Cancer c. Myocardial infarction d. Pulmonary embolism e. Palpitations

a. Stroke c. Myocardial infarction A variant in the gene code for CYP2C19 reduces clopidogrel antiplatelet effects, which increases the patient's risk of cardiovascular events such as stroke and myocardial infarction. Chapter 8

5.The nurse should question an order for glucocorticoids in the treatment of a patient with what? a. Systemic fungal infection b. Diabetes mellitus c. Myasthenia gravis d. Glaucoma

a. Systemic fungal infection Glucocorticoids are contraindicated in the treatment of a patient with a systemic fungal infection or in patients receiving live vaccines. Glucocorticoids should be used with caution in patients with diabetes mellitus, myasthenia gravis, and glaucoma. Chapter 72

3. A patient is receiving glucocorticoids for the treatment of rheumatoid arthritis. The patient complains of having a headache. Which ordered medication should the nurse administer? a. Aspirin [Bayer] b. Acetaminophen [Tylenol] c. Ibuprofen [Advil] d. Naproxen sodium [Aleve]

b. Acetaminophen [Tylenol] The risk of gastrointestinal irritation and ulceration for a patient taking glucocorticoids is increased by concurrent use of other medications, such as aspirin and nonsteroidal anti-inflammatory drugs (NSAIDs). Chapter 72

2. The healthcare provider prescribes a medication that is renally eliminated for a patient with acute renal failure. The nurse recognizes that the patient is at risk for which altered drug response? a. Increased drug excretion b. Decreased drug levels in the blood c. Development of drug toxicity d. Increased tolerance to the medication

c. Development of drug toxicity Kidney disease can reduce drug excretion, causing drugs to accumulate in the body. If the dosage is not lowered, the drug may accumulate to toxic levels Chapter 8

A health care professional is caring for patient who takes an oral contraceptive and is about to begin rifampin therapy to treat tuberculosis. The health care professional should include which of the following instructions? a. increase the rifampin dose b. increase the oral contraceptive dose c. allow 2 hr between taking the two drugs d. use additional contraception

d. use additional contraception Rational: Rifampin, can increase the metabolism of oral contraceptives, reducing their effectiveness. Patients taking oral contraceptives and rifampin should use additional contraceptive methods to prevent an unwanted pregnancy. ATI Infection

1. A nurse is preparing to administer nafcillin IM to an adult client who has an infection. Which of the following actions should the nurse plan to take? (Select all that apply.) A. Select a 25-gauge, ½-inch needle for the injection. B. Administer the medication deeply into the ventrogluteal muscle. C. Ask the client about an allergy to penicillin before administering the medication. D. Monitor the client for 30 min following the injection. E. Tell the client to expect a temporary r

1. A. A 25-gauge, ½-inch needle is too small and short for an IM injection of nafcillin to an adult client. Choose the needle size and length that is best for each specific client. A needle for an adult client's IM injection should be 19- to 22-gauge and 1½ inches long. B. CORRECT: It is important to administer nafcillin IM into a deep muscle mass (the ventrogluteal site). C. CORRECT: It is important to ask the client about an allergy to penicillin or other antibiotics before administering nafcillin. An allergy to another penicillin or to a cephalosporin is a contraindication for administering nafcillin. D. CORRECT: When administering a penicillin or other antibiotic parenterally, it is important to monitor the client for 30 min for an allergic reaction. E. A rash is not an expected reaction after nafcillin administration. A rash can be a manifestation of an allergy to the medication. CHAPTER 44 ANTIBIOTICS AFFECTING THE BACTERIAL CELL WALL

1. A nurse is caring for a client who has diabetes mellitus, pulmonary tuberculosis, and a new prescription for isoniazid. Which of the following supplements should the nurse expect to administer to prevent an adverse effect of INH? A. Ascorbic acid B. Pyridoxine C. Folic acid D. Cyanocobalamin

1. A. Ascorbic acid is given to clients who have a vitamin C deficiency. It is not a supplement administered to prevent an adverse effect of INH. B. CORRECT: Pyridoxine is frequently prescribed along with INH to prevent peripheral neuropathy for clients who have increased risk factors (diabetes mellitus or alcohol use disorder.) C. Folic acid is administered to clients who have hepatic disease and folic acid deficiency. It is not administered to prevent an adverse effect of INH. D. Cyanocobalamin is administered to clients who have malabsorption syndrome. It is not administered to prevent an adverse effect of INH CHAPTER 47 MYCOBACTERIAL, FUNGAL, AND PARASITIC INFECTIONS

1. A nurse reviewing a client's medication history notes an allergy to sulfonamides. This allergy is a contraindication for taking which of the following medications? (Select all that apply.) A. Hydrochlorothiazide B. Metoprolol C. Acetaminophen D. Glipizide E. Furosemide

1. A. CORRECT: A sulfonamide allergy is a contraindication for taking hydrochlorothiazide. Hypersensitivity, including Stevens-Johnson syndrome, can result from taking hydrochlorothiazide and a sulfonamide concurrently. B. A sulfonamide allergy is not a contraindication for taking metoprolol. It is a contraindication for taking chlorpropamide, glimepiride, metolazone, and ethacrynic acid. C. A sulfonamide allergy is not a contraindication for taking acetaminophen. It is a contraindication for taking chlorpropamide, glimepiride, metolazone, and ethacrynic acid. D. CORRECT: A sulfonamide allergy is a contraindication for taking some oral antidiabetes medications, including glipizide and glyburide. Hypersensitivity, including Stevens-Johnson syndrome, can result from taking glipizide and a sulfonamide concurrently. E. CORRECT: A sulfonamide allergy is a contraindication for taking loop diuretics (furosemide). Hypersensitivity, including Stevens-Johnson syndrome, can result from taking furosemide and a sulfonamide concurrently. CHAPTER 46 URINARY TRACT INFECTIONS

1. A nurse is implementing a plan of care for a client who has a wound infection. Which of the following actions is the nurse's priority? A. Administer antibiotic medication. B. Obtain a wound specimen for culture. C. Monitor the client for a superinfection. D. Teach the client about wound care.

1. A. Plan to administer antibiotics, but this is not the priority action. B. CORRECT: When using the urgent vs. nonurgent approach to care, the nurse's priority action is to obtain a culture of the wound before initiating antibiotic therapy. C. Monitor the client for a superinfection during antibiotic therapy; however, this is not the priority action. D. Teach the client about wound care before the client is discharged; however, this is not the priority action. ATI CHAPTER 43 PRINCIPLES OF ANTIMICROBIAL THERAPY

1. A nurse is teaching a client who has a new prescription for combination oral NRTIs (abacavir, lamivudine, and dolutegravir) for treatment of HIV. Which of the following statements should the nurse include? A. "These medications work by blocking HIV entry into cells." B. "These medications work by weakening the cell wall of the HIV virus." C. "These medications work by inhibiting enzymes to prevent HIV replication." D. "These medications work by preventing protein synthesis within the HIV cell."

1. A. The fusion/entry inhibitor enfuvirtide and the CCRS antagonist maraviroc are newer antiretroviral medications that work by blocking HIV entry into cells. B. Some bactericidal antibiotics (penicillin) work by weakening the cell walls of bacteria. C. CORRECT: The NRTI antiretroviral medications this client takes work by inhibiting DNA synthesis and thus viral replication. D. Some antibiotics (aminoglycosides) kill bacteria by preventing protein synthesis within the cell. CHAPTER 48 VIRAL INFECTIONS, HIV, AND AIDS

8. A nurse is preparing to administer dextrose 5% in water ( D5W) 750 mL IV to infuse over 6 hours. The nurse should set the IV pump to deliver how many mL/hr? brown the answer to the nearest whole number. Do not use a trailing zero.

125 ATI Chapter 3

7. A nurse is preparing to administer acetaphetamine 650 mg PO every 6 hr PRN for pain. The amount available is acetaphetamine liquid 500 mg/5mL. How many mL should the nurse administer per dose? Round the answer to the nearest tenth. Use a leading zero if it applies. Do not use a trailing zero

6.5 ATI Chapter 3

4. A nurse is preparing to administer haloperidol 2 mg PO every 12 hr. available is haloperidol 1 mg/ tablet. How many tablets should the nurse administer? Round to the nearest whole number. Do not use a trailing zero

2 ATI Chapter 3

2. A nurse is teaching a client who has manifestations of a urinary tract infection. The nurse should instruct the client that which of the following tests is needed to determine which microorganism is causing the infection? A. Blood WBC B. Blood creatinine C. Urine culture D. Urine specific gravity

2. A. An increase in the blood WBC count indicates the presence of an infection. A WBC does not identify the organism causing the infection. B. Blood creatinine is used to evaluate kidney function. C. CORRECT: A urine culture is used to identify the causative micro-organism and the sensitivity is used to identify an effective antibiotic to prescribe. D. A urine specific gravity evaluates the concentration of the urine to dehydration or overhydration. ATI CHAPTER 43 PRINCIPLES OF ANTIMICROBIAL THERAPY

2. A nurse is teaching a client who has a new prescription for nitrofurantoin. Which of the following information should the nurse include? (Select all that apply.) A. Observe for bruising on the skin. B. Take the medication with milk or meals. C. Expect brown discoloration of urine. D. Crush the medication if it is difficult to swallow. E. Expect insomnia when taking it.

2. A. CORRECT: Bruising can indicate a blood dyscrasia, and the client should notify the provider if this occurs. B. CORRECT: Taking the medication with milk or meals minimizes GI discomfort from nausea, vomiting, anorexia, and diarrhea. C. CORRECT: A brown discoloration of urine is a common adverse effect of nitrofurantoin. D. Crushing the medication can cause staining of the teeth. E. Nitrofurantoin is more likely to cause drowsiness than insomnia. CHAPTER 46 URINARY TRACT INFECTIONS

2. A nurse is caring for a client who takes several antiretroviral medications, including the NRTI zidovudine, to treat HIV infection. The nurse should monitor for which of the following adverse effects of zidovudine? (Select all that apply.) A. Fatigue B. Blurred vision C. Ataxia D. Hyperventilation E. Vomiting

2. A. CORRECT: Fatigue is a manifestation of anemia, an adverse effect of zidovudine. Neutropenia can also occur, causing a high risk for infection. B. Zidovudine can cause hearing loss and photophobia. C. Zidovudine can cause vertigo. D. CORRECT: Hyperventilation is a finding that can occur if the client develops lactic acidosis, a serious adverse effect of zidovudine. E. CORRECT: Vomiting and other GI effects are adverse effects of zidovudine CHAPTER 48 VIRAL INFECTIONS, HIV, AND AIDS

3. A nurse is teaching a client who has a severe UTI about ciprofloxacin. Which of the following information about adverse reactions should the nurse include? (Select all that apply.) A. Observe for pain and swelling of the Achilles tendon. B. Watch for a vaginal yeast infection. C. Expect excessive nighttime perspiration. D. Inspect the mouth for cottage cheese-like lesions. E. Take the medication with a dairy product.

3. A. CORRECT: Pain and swelling of the Achilles tendon indicate an adverse effect of ciprofloxacin to report to the provider. B. CORRECT: A vaginal yeast infection is an overgrowth of Candida albicans, which commonly occurs when taking ciprofloxacin. C. An alteration in perspiration is not an adverse effect of this medication. Common adverse effects include headache, tremors, dizziness, and dysphagia. D. CORRECT: Cottage cheese-like lesions in the mouth indicate an overgrowth of Candida albicans, a common adverse effect when taking ciprofloxacin. E. Milk and other dairy products contain calcium ions that reduce the effect of ciprofloxacin. The client should take the medication 6 hr before or 2 hr after ingesting dairy products. CHAPTER 46 URINARY TRACT INFECTIONS

3. A nurse is caring for a client who is starting a course of gentamicin IV for a serious respiratory infection. For which of the following manifestations should the nurse monitor as an adverse effect of this medication? (Select all that apply.) A. Pruritus B. Hematuria C. Muscle weakness D. Difficulty swallowing E. Vertigo

3. A. CORRECT: Paresthesias of the hands and feet, urticaria, rash, and pruritus are indications of a hypersensitivity reaction that can occur in clients taking gentamicin. B. CORRECT: Hematuria is an indication of acute kidney toxicity due to gentamicin. C. CORRECT: Muscle weakness and respiratory depression can occur in clients taking gentamicin as a result of neuromuscular blockade. D. Difficulty swallowing is not an adverse effect of gentamicin. E. CORRECT: Vertigo, ataxia, and hearing loss are indications of ototoxicity that can occur in clients taking gentamicin. CHAPTER 45 ANTIBIOTICS AFFECTING PROTEIN SYNTHESIS

3. A nurse is teaching a group of nurses about antimicrobial therapy. The nurse should instruct that effective penetration of antibiotics can be impeded by which of the following conditions? A. Meningitis B. An infected abscess C. Endocarditis D. Pneumonia E. Pyelonephritis

3. A. CORRECT: The blood brain barrier can block antibiotics from reaching the infective microorganisms in clients who have meningitis. B. CORRECT: Purulent drainage and poor vascularity can impede penetration of an antibiotic to an infected abscess. C. CORRECT: Clients who have endocarditis might develop a bacterial vegetation which can impede penetration of an antibiotic. D. Clients who have pneumonia should respond effectively to antimicrobial therapy because of the vascularity of pulmonary tissue. E. Clients who have pyelonephritis should respond to antimicrobial therapy effectively because of the vascularity of kidney tissue and the filtration system. ATI CHAPTER 43 PRINCIPLES OF ANTIMICROBIAL THERAPY

3. A nurse is caring for a client who is taking ritonavir, a protease inhibitor, to treat HIV infection. The nurse should monitor for which of the following adverse effects of this medication? A. Increased TSH level B. Decreased ALT level C. Hypoglycemia D. Hyperlipidemia

3. A. Increased TSH and T4 levels indicate hyperthyroidism, which is not an adverse effect of ritonavir. B. An increase in liver function tests, including AST and ALT levels, can occur as an adverse effect of ritonavir. C. Hyperglycemia indicating a possible onset or worsening of diabetes mellitus can occur as an adverse effect of ritonavir. D. CORRECT: Hyperlipidemia with increased cholesterol and triglyceride levels can occur as an adverse effect of ritonavir CHAPTER 48 VIRAL INFECTIONS, HIV, AND AIDS

3. A nurse is obtaining a medication history from a client who is to receive imipenem-cilastatin IV to treat an infection. Which of the following medications the client also receives increases the risk for a medication interaction? A. Regular insulin B. Furosemide C. Valproic acid D. Ferrous sulfate

3. A. Regular insulin, an antidiabetes medication, does not interact with imipenem-cilastatin. Medications that interact with imipenem-cilastatin include probenecid, aminophylline, theophylline, ganciclovir, and cyclosporine. B. Furosemide, a loop diuretic, does not interact with imipenem-cilastatin. C. CORRECT: Imipenem-cilastatin decreases the blood levels of valproic acid, an antiseizure medication, putting the client at risk for increased seizure activity. If the client must take these two medications concurrently, monitor for seizures. D. Ferrous sulfate does not interact with imipenem-cilastatin. CHAPTER 44 ANTIBIOTICS AFFECTING THE BACTERIAL CELL WALL

4. A nurse is teaching a client who is beginning a course of metronidazole to treat an infection. For which of the following findings of an adverse effect should the nurse instruct the client as a priority to stop taking metronidazole and notify the provider? A. Metallic taste B. Nausea C. Ataxia D. Dark-colored urine

4. A. A metallic taste in the mouth is an expected adverse effect of metronidazole and is nonurgent for the client to report to the provider. Another adverse effect is the priority to report. B. Nausea is an expected adverse effect of metronidazole and is nonurgent for the client to report to the provider. Another adverse effect is the priority to report. C. CORRECT: Using the urgent vs. nonurgent approach to client care, the priority adverse effect to report to the provider is ataxia, tremors, paresthesias of the extremities, and seizures, which are manifestations of CNS toxicity. D. Dark-colored urine is expected adverse effect of metronidazole and is nonurgent for the client to report to the provider. Another adverse effect is the priority to report. CHAPTER 47 MYCOBACTERIAL, FUNGAL, AND PARASITIC INFECTIONS

4. A nurse is caring for a group of clients who are receiving antimicrobial therapy. The nurse should identify that which of the following clients is at risk for medication toxicity? A. A young adult client who has a sinus infection B. An older adult client who has prostatitis C. A client who is postpartum and has mastitis D. A middle adult client who has a urinary tract infection

4. A. A young adult client who has a sinus infection should be able to metabolize and excrete the medication without developing antibiotic toxicity. B. CORRECT: An older adult client who has prostatitis and is receiving antibiotics is at risk for toxicity due to the age-related reduction in medication metabolism and excretion. C. A client who is postpartum and has mastitis should be able to metabolize and excrete the medication without developing antibiotic toxicity. D. A middle adult client who has a urinary tract infection should be able to metabolize and excrete the medication without developing antibiotic toxicity. ATI CHAPTER 43 PRINCIPLES OF ANTIMICROBIAL THERAPY

4. A nurse is caring for a client who has subacute bacterial endocarditis and is receiving several antibiotics, including streptomycin IM. For which of the following manifestations should the nurse monitor as an adverse effect of this medication? A. Ataxia B. Urinary retention C. Constipation D. Complex partial seizures

4. A. CORRECT: Ataxia of the client's movement and coordination of the body is a common adverse effect of streptomycin. This medication treats infections in combination with other antibiotics or to treat severe infections when other antibiotics failed. B. Urinary retention is not an adverse effect of streptomycin, but this medication can cause nephrotoxicity. C. Constipation is not an adverse effect of streptomycin. Common adverse effects include headache, ototoxicity, angioedema, muscle weakness, stomatitis, and optic nerve toxicity. D. Complex partial seizures are not an adverse effect of streptomycin. CHAPTER 45 ANTIBIOTICS AFFECTING PROTEIN SYNTHESIS

4. A nurse is caring for a client who has a cerebrospinal fluid infection with gram-negative bacteria. Which of the following cephalosporin antibiotics should the nurse expect to administer IV to treat this infection? A. Cefaclor B. Cefazolin C. Cefepime D. Cephalexin

4. A. Cefaclor, a second-generation cephalosporin, is unlikely to be effective against gram-negative bacteria in cerebrospinal fluid. B. Cefazolin, a first-generation cephalosporin, is unlikely to be effective against gram-negative bacteria in cerebrospinal fluid. C. CORRECT: Cefepime, a fourth-generation cephalosporin, is more likely to be effective against this infection than the other medications, which are from the first or second generation. Medications from each progressive generation of cephalosporins are more effective against gram-negative bacteria, more resistant to destruction by beta-lactamase, and more able to reach cerebrospinal fluid. D. Cephalexin, a first-generation cephalosporin, is unlikely to be effective against gram-negative bacteria in cerebrospinal fluid. CHAPTER 44 ANTIBIOTICS AFFECTING THE BACTERIAL CELL WALL

4. A nurse is planning discharge teaching for a female client who has a new prescription for trimethoprim-sulfamethoxazole. Which of the following information should the nurse include? A. Take the medication even if pregnant. B. Maintain a fluid restriction while taking it. C. Take it on an empty stomach. D. Stop taking it when manifestations subside.

4. A. Trimethoprim-sulfamethoxazole can cause birth defects and fetal kernicterus, especially when taking it during the first trimester or near term B. The client should take trimethoprim-sulfamethoxazole with at least eight 8 oz glasses of water each day to prevent crystalluria, which results in kidney damage. C. CORRECT: Inform the client that the medication can be taken the medication with or without food. D. The client should take the entire course of medication the provider prescribed to destroy all the bacteria and prevent a rebound infection. CHAPTER 46 URINARY TRACT INFECTIONS

5. A nurse is administering IV acyclovir to a client who has varicella. Which of the following actions should the nurse take? A. Administer a stool softener. B. Decrease fluid intake following infusion. C. Infuse acyclovir over 1 hr. D. Monitor for a for hypotension

5. A. Acyclovir can cause diarrhea. B. Increase fluids during and for 2 hr following acyclovir infusion to prevent nephrotoxicity. C. CORRECT: Administer IV acyclovir slowly, over at least 1 hr, to prevent nephrotoxicity. D. Acyclovir can cause thrombocytopenia in clients who are immunocompromised. CHAPTER 48 VIRAL INFECTIONS, HIV, AND AIDS

5. A nurse is planning to administer ciprofloxacin IV to a client who has cystitis. Which of the following actions should the nurse take? A. Administer a concentrated solution. B. Infuse the medication over 60 min. C. Infuse the solution through the primary IV fluid's tubing. D. Choose a small peripheral vein for administration.

5. A. Administer ciprofloxacin IV in a dilute solution to minimize irritation of the vein. B. CORRECT: Administer ciprofloxacin IV over 60 min to minimize irritation of the vein. C. Do not infuse ciprofloxacin with any other IV medication or solution. The nurse should use another infusion site. D. Infuse ciprofloxacin into a large vein to minimize the risks of irritation and phlebitis. CHAPTER 46 URINARY TRACT INFECTIONS

5. A nurse is teaching a group of nurses about the importance of prophylactic antimicrobial therapy. Which of the following information should the nurse include in the teaching? (Select all that apply.) A. Administer prophylactic antimicrobial therapy to clients who report exposure to a sexually transmitted infection. B. Administer prophylactic antimicrobial therapy to clients who are having orthopedic surgery. C. Instruct clients who have a prosthetic heart valve about the need for prophylactic antimicrobial therapy before dental work. D. Consult the provider for prophylactic antimicrobial therapy for clients who have recurrent urinary tract infections. E. Instruct clients to request prophylactic antimicrobial therapy immediately when they have an upper respiratory infection.

5. A. CORRECT: Clients who suspect exposure to a sexually transmitted infection require prophylactic antimicrobial therapy to prevent an infection. B. CORRECT: Clients who are having orthopedic surgery require prophylactic antimicrobial therapy to prevent an infection. C. CORRECT: Clients who are having dental work and have a prosthetic heart valve should receive prophylactic antimicrobial therapy to prevent an infection. D. CORRECT: Clients who have recurrent urinary tract infections should receive prophylactic antimicrobial therapy to prevent an infection. E. Upper respiratory infections can be viral in origin. Resistance to antimicrobial therapy can occur when clients take antibiotics needlessly. ATI CHAPTER 43 PRINCIPLES OF ANTIMICROBIAL THERAPY

5. A nurse is preparing to administer penicillin V to a client who has a streptococcal infection. The client reports difficulty swallowing tablets and doesn't "do well" with liquid or chewable medications because of the taste, even when the nurse mixes the medication with food. The nurse should request a prescription for which of the following medications? A. Fosfomycin B. Amoxicillin C. Nafcillin D. Cefaclor

5. A. Fosfomycin is available only in a PO formulation. Acceptable alternatives to penicillin V within the penicillin classification include penicillin G, ampicillin, ticarcillin-clavulanate, and piperacillin tazobactam. B. Amoxicillin is available only in a PO formulation. C. CORRECT: Nafcillin is an acceptable alternative within the penicillin classification because it is available for IM or IV use. D. Cefaclor is available only in a PO formulation and is not a penicillin. CHAPTER 44 ANTIBIOTICS AFFECTING THE BACTERIAL CELL WALL

5. A nurse is caring for a client who is undergoing preparation for extensive colorectal surgery. Which of the following oral antibiotics should the nurse expect to administer specifically to suppress normal flora in the GI tract? A. Kanamycin B. Gentamicin C. Neomycin D. Tobramycin

5. A. To rid the large intestine of normal flora, the nurse has to administer an antibiotic the client can take orally, so that it passes through the GI tract. Kanamycin, an aminoglycoside, is only available in parenteral formulations. B. To rid the large intestine of normal flora, the nurse has to administer an antibiotic the client can take orally, so that it passes through the GI tract. Gentamicin, an aminoglycoside, is only available in parenteral formulations. C. CORRECT: Expect to administer neomycin, an aminoglycoside antibiotic, orally prior to GI surgery to rid the large intestine of normal flora. D. To rid the large intestine of normal flora, the nurse has to administer an antibiotic the client can take orally, so that it passes through the GI tract. Tobramycin, an aminoglycoside, is only available in parenteral and inhalation formulations. CHAPTER 45 ANTIBIOTICS AFFECTING PROTEIN SYNTHESIS

6. A nurse is teaching a client who is beginning highly active antiretroviral therapy (HAART) for HIV infection about ways to prevent medication resistance. Which of the following information should the nurse teach the client about resistance? A. Taking low dosages of antiretroviral medication minimizes resistance. B. Taking one antiretroviral medication at a time minimizes resistance. C. Taking medication at the same times daily without missing doses minimizes resistance. D. Changing the medication regimen when adverse effects occur minimizes resistance.

6. A. Taking low dosages of the medication can cause medication resistance. B. Taking a combination of antiretroviral medications helps prevent resistance to each medication. Resistance occurs quickly if only one medication is taken. C. CORRECT: Emphasize the importance of taking each dose of medication exactly as prescribed. Missing even a few doses of antiretroviral medication can promote medication resistance, which can cause treatment failure. D. Changing the medication regimen when adverse effects occur can promote medication resistance. CHAPTER 48 VIRAL INFECTIONS, HIV, AND AIDS

3. A nurse is preparing to administer Furosemide 80 mg PO daily. The amount available of furosemide oral solution 10 mg/ 1 mL . how many mL should the nurse administer? Round to the nearest whole number. Do not use a trailing zero

8 ATI Chapter 3

6. An adolescent is brought to the emergency department by a parent who reports that the patient took a whole bottle of extended-release acetaminophen tablets somewhere between 8 and 10 hours ago. The nurse will anticipate administering which of the following? a. Acetylcysteine [Mucomyst] b. Activated charcoal c. Hemodialysis d. Respiratory support

ANS: A The nurse should anticipate giving acetylcysteine, because it is the specific antidote for acetaminophen overdose. It is 100% effective when given within 8 to 10 hours after ingestion and may still have some benefit after this interval. Activated charcoal is effective only if given before the medication is absorbed, so it must be given much sooner. Hemodialysis is not indicated. Respiratory support is used for ASA overdose. REF: p. 863

A provider has written an order for a medication: drug 100 mg PO every 6 hours. The half-life for the drug is approximately 6 hours. The nurse is preparing to administer the first dose at 8:00 AM on Tuesday. On Wednesday, when will the serum drug level reach plateau? a. 2:00 AM b. 8:00 AM c. 2:00 PM d. 8:00 PM

ANS: B It takes four half-lives for a drug to reach plateau. Total body stores reach their peak at the beginning of the fifth dose of a drug if all doses are equal in amount; in this case, this will be at 8:00 AM the following day. page 40-41

6. A 30-year-old male is admitted to the hospital with acute pancreatitis. He is in acute pain described as a 10/10, which is localized to the abdomen, periumbilical area, and some radiation to his back. The abdomen is grossly distended so it is difficult to assess. He is restless and agitated, with elevated pulse and blood pressure. An appropriate pain management plan of care may include which medication(s)? a. IV Dilaudid q 4 hours prn, hydrocodone 5/500 PO q 6 hours prn, and acetaminophen b. Norco 5/500 q 4 hours PO and Benadryl 25 mg PO q 6 hours c. Phenergan 25 mg IM q 6 hours d. Tylenol 325 mg q 6 hours

ANS: A A variety of routes of administration are used to deliver analgesics. A principle of pain management is to use the oral route of administration whenever feasible. All of the first-line analgesics used to manage pain are available in short-acting and long-acting formulations. For patients who have continuous pain, a long-acting analgesic, such as modified-release oral morphine, oxycodone, or hydromorphone, or transdermal fentanyl, is used to treat the persistent baseline pain. A fast-onset, short-acting analgesic (usually the same drug as the long-acting) is used to treat breakthrough pain if it occurs. When the oral route is not possible, such as in patients who cannot swallow or are NPO or nauseated, other routes of administration are used, including intravenous (IV), subcutaneous, transdermal, and rectal. Norco, Benadryl, Phenergan, and Tylenol are not appropriate solo choices for acute pancreatitis with pain reported as 10/10. Chapter 28 Giddens

7. Aspects of safety culture that contribute to a culture of safety in a health care organization include which component? a. Communication b. Fear of punishment c. Malpractice implications d. Team nursing

ANS: A Aspects that contribute to a culture of safety include leadership, teamwork, an evidence base, communication, learning, a just culture, and patient-centered care. Fear of professional or personal punishment and concern about malpractice implications are considered barriers to a culture of safety. No model of nursing care has been related to a culture of safety. Giddens Safety

A patient is taking sertraline [Zoloft] for depression, and the provider orders azithromycin [Zithromax] to treat an infection. What will the nurse do? a. Contact the provider to discuss an alternative to azithromycin. b. Request an order for a different antidepressant medication. c. Request an order to reduce the dose of sertraline. d. Withhold the sertraline while giving the azithromycin.

ANS: A Both sertraline and azithromycin prolong the QT interval, and when taken together, they increase the risk of fatal dysrhythmias. Because the antibiotic is used for a short time, it is correct to consider using a different antibiotic. Reducing the dose of sertraline does not alter the combined effects of two drugs that lengthen the QT interval. Sertraline should not be stopped abruptly, so withholding it during antibiotic therapy is not indicated. page 64

4. On admission to the clinic, the nurse notes a moderate amount of serous exudate leaking from the patient's wound. The nurse realizes what information about this fluid? a. Contains the materials used by the body in the initial inflammatory response. b. Indicates that the patient has an infection at the site of the wound. c. Is destroying healthy tissue. d. Results from ineffective cleansing of the wound area.

ANS: A Exudate is fluid moved from the vascular spaces to the area around a wound. It contains the proteins, fluid, and white blood cells (WBCs) needed to contain possible pathogens at the site of injury. Exudate appears as part of all inflammatory responses and does not mean an infection is present. Exudate is part of normal inflammatory responses which contain self-monitoring mechanisms to help prevent damage to healthy tissue. Exudate appears at wound sites regardless of cleaning done to the area of injury. Chapter 23 Giddens

1. A nurse is discussing glucocorticoids with a group of nursing students. Which statement by a student indicates understanding of the teaching? a. "Glucocorticoids have both endocrine and nonendocrine uses." b. "Patients treated for adrenocortical insufficiency receive pharmacologic doses." c. "Pharmacologic effects are achieved with low doses of glucocorticoids." d. "Physiologic doses are used to treat inflammatory disorders."

ANS: A Glucocorticoids have applications for both endocrine and nonendocrine disorders. Adrenocortical insufficiency requires physiologic, or low-dose, treatment. Pharmacologic doses are used when large doses are required, such as to suppress inflammation. Physiologic doses are used to treat endocrine disorders. REF: p. 869

5. A provider has ordered ceftriaxone 4 gm once daily for a patient with renal impairment. What will the nurse do? a. Administer the medication as prescribed. b. Contact the provider to ask about giving the drug in divided doses. c. Discuss increasing the interval between doses with the provider. d. Discuss reducing the dose with the provider.

ANS: A Unlike other cephalosporins, ceftriaxone is eliminated largely by the liver, so dosage reduction is unnecessary in patients with renal impairment. Giving the drug in divided doses, increasing the interval between doses, and reducing the dose are not necessary. REF: pp. 1025-1026

8. A staff nurse reports a medication error due to failure to administer a medication at the scheduled time. What is the charge nurse's best response? a. "We'll conduct a root cause analysis." b. "That means you'll have to do continuing education." c. "Why did you let that happen?" d. "You'll need to tell the patient and family."

ANS: A In a just culture the nurse is accountable for their actions and practice, but people are not punished for flawed systems. Through a strategy such as root cause analysis the reasons for errors in medication administration can be identified and strategies developed to minimize future occurrences. Requiring continued education may be an appropriate recommendation but not until data is collected about the event. Telling the patient is part of transparency and the sharing and disclosure among stakeholders, but it is generally the role of risk management staff, not the staff nurse. Giddens Safety

2. A 19-year-old male has sustained a transaction of C-7 in a motor vehicle crash rendering him a quadriplegic. He describes his pain as burning, sharp, and shooting. What type of pain is this patient describing? a. Neuropathic pain b. Ghost pain c. Mixed pain syndrome d. Nociceptive pain

ANS: A Neuropathic pain results from the abnormal processing of sensory input by the nervous system as a result of damage to the brain, spinal cord, or peripheral nerves. Simply put, neuropathic pain is pathologic. Examples of neuropathic pain include postherpetic neuralgia, diabetic neuropathy, phantom pain, and post stroke pain syndrome. Patients with neuropathic pain use very distinctive words to describe their pain, such as "burning," "sharp," and "shooting." Ghost pain is pain associated with loss of a limb or digit. Mixed pain syndrome is not easily recognized, is unique with multiple underlying and poorly understood mechanisms like fibromyalgia and low back pain. Nociceptive pain refers to the normal functioning of physiological systems that leads to the perception of noxious stimuli (tissue injury) as being painful. Patients describe this type of pain as aching, cramping, or throbbing. Neuropathic pain is pathologic and results from abnormal processing of sensory input by the nervous system as a result of damage to the brain, spinal cord, or peripheral nerves. Patients describe this type of pain as burning, sharp, and shooting. Chapter 28 Giddens

3. Controlling pain is important to promoting wellness. Unrelieved pain has been associated with complication? a. Prolonged stress response and a cascade of harmful effects system-wide b. Large tidal volumes and decreased lung capacity c. Decreased tumor growth and longevity d. Decreased carbohydrate, protein, and fat destruction

ANS: A Pain triggers a number of physiologic stress responses in the human body. Unrelieved pain can prolong the stress response and produce a cascade of harmful effects in all body systems. The stress response causes the endocrine system to release excessive amounts of hormones, such as cortisol, catecholamines, and glucagon. Insulin and testosterone levels decrease. Increased endocrine activity in turn initiates a number of metabolic processes, in particular, accelerated carbohydrate, protein, and fat destruction, which can result in weight loss, tachycardia, increased respiratory rate, shock, and even death. The immune system is also affected by pain as demonstrated by research showing a link between unrelieved pain and a higher incidence of nosocomial infections and increased tumor growth. Large tidal volumes are not associated with pain while decreased lung capacity is associated with unrelieved pain. Decreased tumor growth and longevity are not associated with unrelieved pain. Decreased carbohydrate, protein, and fat are not associated with pain or stress response. Chapter 28 Giddens

2. An 18-month-old female patient is diagnosed with her fifth ear infection in the past 10 months. The physician notes that the child's growth rate has decreased from the 60th percentile for height and weight to the 15th percentile over that same time period. The child has been treated for thrush consistently since the third ear infection. The nurse understands that the patient is at risk for which condition? a. Primary immunodeficiency b. Secondary immunodeficiency c. Cancer d. Autoimmunity

ANS: A Primary immunodeficiency is a risk for patients with two or more of the listed problems. Secondary immunodeficiency is induced by illness or treatment. Cancer is caused by abnormal cells that will trigger an immune response. Autoimmune diseases are caused by hyperimmunity. Chapter 22 Giddens

A patient is receiving digoxin twice daily. When assessing the patient before giving a dose, the nurse counts a pulse of 60 beats per minute and learns that the patient is experiencing nausea. The nurse consults a drug manual and verifies that the ordered dose is correct. What should the nurse do? a. Contact the prescriber to report the symptoms. b. Delay the dose so the drug can clear from receptor sites. c. Give the medication as ordered, because the dose is correct. d. Request an antinausea medication from the prescriber.

ANS: A The symptoms indicate toxicity, and even though the dose is safe and effective in most cases, an individual patient may have toxic effects with a standard dose. The nurse should contact the prescriber to discuss the next steps. Delaying a dose without a change in order is not within the scope of practice for a nurse. The nurse should not give a dose of a medication when toxicity is suspected, because additional drug will compound the symptoms. Antiemetics are useful for counteracting drug side effects, but they should not be used when the patient's symptoms indicate toxicity. page 51-52

3. The nurse is caring for a postoperative patient who had an open appendectomy. The nurse understands that this patient should have some erythema and edema at the incision site 12 to 24 hours post operation dependent on which condition? a. His immune system is functioning properly. b. He is properly vaccinated. c. He has an infection. d. The suppressor T-cells in his body are activated.

ANS: A Tissue integrity is closely associated with immunity. Openings in the integumentary system allow for the entrance of pathogens. If the immune response is functioning optimally, the body responds to the insult to the tissue by protecting the area from invasion of microorganisms and pathogens with inflammation. Routine vaccinations have no bearing on the body's response to intentional tissue impairment. The redness and swelling at the incision site in the first 12 to 24 hours is part of optimal immune functioning. A patient with erythema and edema that persist or worsen should be evaluated for infection. Suppressor T-cells help to control the immune response in the body. Chapter 22 Giddens

A patient is given a prescription for azithromycin [Zithromax] and asks the nurse why the dose on the first day is twice the amount of the dose on the next 4 days. Which reply by the nurse is correct? a. "A large initial dose helps to get the drug to optimal levels in the body faster." b. "The first dose is larger to minimize the first pass effect of the liver." c. "The four smaller doses help the body taper the amount of drug more gradually." d. "Tubular reabsorption is faster with initial doses, so more is needed at first."

ANS: A A large initial dose is often used as a loading dose to help get serum drug levels to plateau levels more quickly. Larger doses do not prevent first pass effects in drugs susceptible to this type of metabolism. Tapering of doses sometimes is used to prevent rebound or withdrawal effects and is done by stepping down the amount of drug with each dose. Tubular reabsorption is a process that allows drugs to be reabsorbed from the urine into the blood. page 41

7. A patient who has been receiving intravenous gentamicin for several days reports having had a headache for 2 days. The nurse will request an order to: a. discontinue the gentamicin. b. obtain a gentamicin trough before the next dose is given c. give an analgesic to control headache discomfort. d. obtain renal function tests to evaluate for potential nephrotoxicity.

ANS: A A persistent headache may be a sign of developing ototoxicity, and since ototoxicity is largely irreversible, gentamicin should be withdrawn at the first sign of developing ototoxicity. A gentamicin trough should be obtained before the next dose is given when high gentamicin levels are suspects. Analgesics are not indicated until a serious cause of the headache has been ruled out. A headache is an early sign of ototoxicity, not nephrotoxicity. REF: pp. 1052-1053

15. A patient who reports regular consumption of two or three alcoholic beverages per day asks about taking acetaminophen when needed for occasional recurrent pain. What will the nurse tell the patient? a. "Do not take more than 2 gm of acetaminophen a day." b. "Do not take more than 3 gm of acetaminophen a day." c. "Do not take more than 4 gm of acetaminophen a day." d. "Do not take a fixed-dose preparation with opioid analgesics."

ANS: A Acetaminophen poses a risk of liver toxicity when taken chronically or in larger doses, especially when combined with alcohol. Patients who drink regularly should be advised to take acetaminophen in low doses and not to exceed 2 gm per day. REF: p. 862

A nurse educator is conducting a continuing education class on pharmacology. To evaluate the learning of the nurses in the class, the nurse educator asks, "Which drug name gives information about the drug's pharmacologic classification?" Which is the correct response? a . Amoxicillin b . Tylenol c . Cipro d . Motrin

ANS: A Amoxicillin is the generic name, and the suffix "-cillin" indicates that it belongs to the penicillin class of antibiotics. Tylenol, Cipro, and Motrin are all trade names without segments that indicate their pharmacologic class. page 18

The nurse receives an order to give morphine 5 mg IV every 2 hours PRN pain. Which action is not part of the six rights of drug administration? a . Assessing the patient's pain level 15 to 30 minutes after giving the medication b . Checking the medication administration record to see when the last dose was administered c . Consulting a drug manual to determine whether the amount the prescriber ordered is appropriate d . Documenting the reason the medication was given in the patient's electronic medical record

ANS: A Assessing the patient's pain after administering the medication is an important part of the nursing process when giving medications, but it is not part of the six rights of drug administration. Checking to see when the last dose was given helps ensure that the medication is given at the right time. Consulting a drug manual helps ensure that the medication is given in the right dose. Documenting the reason for a pain medication is an important part of the right documentation—the sixth right. page 9

A patient newly diagnosed with diabetes is to be discharged from the hospital. The nurse teaching this patient about home management should begin by doing what? a . Asking the patient to demonstrate how to measure and administer insulin b . Discussing methods of storing insulin and discarding syringes c . Giving information about how diet and exercise affect insulin requirements d . Teaching the patient about the long-term consequences of poor diabetes control

ANS: A Because insulin must be given correctly to control symptoms and because an overdose can be fatal, it is most important for the patient to know how to administer it. Asking for a demonstration of technique is the best way to determine whether the patient has understood the teaching. When a patient is receiving a lot of new information, the information presented first is the most likely to be remembered. The other teaching points are important as well, but they are not as critical and can be taught later. page 9

6. A prescriber has ordered cefoxitin for a patient who has an infection caused by a gram-negative bacteria. The nurse taking the medication history learns that the patient experienced a maculopapular rash when taking amoxicillin [Amoxil] several years earlier. What will the nurse do? a. Administer the cefoxitin and observe for any side effects. b. Give the cefoxitin and have epinephrine and respiratory support available. c. Request an order for a different, nonpenicillin, noncephalosporin antibiotic. d. Request an order to administer a skin test before giving the cefoxitin

ANS: A Because of structural similarities between penicillins (PCNs) and cephalosporins, a few patients allergic to one drug type will be allergic to the other drug type, although this is rare. For patients with mild PCN allergy, such as rash, cephalosporins can be used with minimal concern, so it is correct to administer the drug and monitor for side effects. It is unnecessary to prepare for anaphylaxis, to give another class of drug, or to administer a skin test. REF: p. 1027

10. A patient taking a glucocorticoid for arthritis reports feeling bloated. The nurse notes edema of the patient's hands and feet. Which action by the nurse is correct? a. Ask the patient about sodium intake. b. Obtain a blood glucose level. c. Suggest the patient limit potassium intake. d. Tell the patient to stop taking the drug.

ANS: A Because of their mineralocorticoid activity, glucocorticoids can cause sodium and water retention and potassium loss. Asking about the sodium intake can help the nurse evaluate this patient. Although glucocorticoids can affect glucose tolerance, this patient does not have signs of hyperglycemia. Patients with sodium and water retention should be encouraged to increase their potassium intake. Telling a patient to stop taking the drug is incorrect, because a glucocorticoid must be withdrawn slowly to allow time for recovery of adrenal function. REF: p. 873

5. A child with an ear infection is not responding to treatment with amoxicillin [Amoxil]. The nurse will expect the provider to order: a. amoxicillin-clavulanic acid [Augmentin]. b. ampicillin. c. nafcillin. d. penicillin G [Benzylpenicillin].

ANS: A Beta-lactamase inhibitors are drugs that inhibit bacterial beta-lactamases. These drugs are always given in combination with a penicillinase-sensitive penicillin. Augmentin contains amoxicillin and clavulanic acid and is often used when patients fail to respond to amoxicillin alone. Ampicillin is similar to amoxicillin, but amoxicillin is preferred and, if drug resistance occurs, ampicillin is equally ineffective. Pharmaceutical chemists have developed a group of penicillins that are resistant to inactivation by beta-lactamases (eg, nafcillin), but these drugs are indicated only for penicillinase-producing strains of staphylococci. Penicillin G would be as ineffective as amoxicillin if beta-lactamase is present. REF: pp. 1021-1023

14. Which side effect of clindamycin [Cleocin] causes the most concern and may warrant discontinuation of the drug? a. Diarrhea b. Headache c. Nausea d. Vomiting

ANS: A CDAD is a serious, sometimes fatal suprainfection associated with clindamycin. Patients with diarrhea should notify their prescriber immediately and discontinue the drug until this condition has been ruled out. Headache, nausea, and vomiting do not warrant discontinuation of the drug and are not associated with severe side effects. REF: p. 1043

12. A nurse is explaining to nursing students why a cephalosporin is used in conjunction with an aminoglycoside for a patient with an infection. Which statement by a student indicates understanding of the teaching? a. "Cephalosporins enhance the actions of aminoglycosides by weakening bacterial cell walls." b. "Cephalosporins prevent neuromuscular blockade associated with aminoglycosides." c. "Cephalosporins prolong the postantibiotic effects of the aminoglycosides so doses can be decreased." d. "Cephalosporins reduce bacterial resistance to aminoglycosides."

ANS: A Cephalosporins, penicillins, and vancomycin can be used in conjunction with aminoglycosides; these drugs weaken the bacterial cell wall and enhance the bactericidal actions of aminoglycosides. Cephalosporins do not prevent neuromuscular blockade. They do not prolong the postantibiotic effects of aminoglycosides. They do not affect bacterial resistance. P REF: p. 1053

12. A nurse is providing teaching for a patient who will begin taking clarithromycin ER [Biaxin XL] to treat an Helicobacter pylori infection. Which statement by the patient indicates understanding of the teaching? a. "I may experience distorted taste when taking this medication." b. "I should take 1 tablet twice daily for 10 days." c. "I should take this medication on an empty stomach." d. "This medication does not interact with other drugs."

ANS: A Clarithromycin is available in an extended-relief preparation as Biaxin XL. Biaxin can cause distortion of taste, so patients should be warned of this side effect. Biaxin XL should be taken once and not twice daily. Biaxin should be taken with food. Biaxin interacts with other drugs by inhibiting hepatic metabolism of those drugs. REF: p. 1042

10. A child with an upper respiratory infection caused by B. pertussis is receiving erythromycin ethylsuccinate. After 2 days of treatment, the parent asks the nurse why the child's symptoms have not improved. Which response by the nurse is correct? a. "Erythromycin eliminates the bacteria that causes the infection, but not the toxin that causes the symptoms." b. "We may need to add penicillin or another antibiotic to increase the antimicrobial spectrum." c. "We will need to review the culture sensitivity information to see whether a different antibiotic is indicated." d. "Your child may have developed a suprainfection that we need to culture and treat."

ANS: A Erythromycin is the drug of first choice for treating pertussis infections. Because symptoms are caused by a bacterial toxin and not by the bacteria itself, the drug eliminates the bacteria but does little to alter the course of the disease. It is given to lower infectivity. It is not necessary to add another antibiotic, review the sensitivity information, or look for a suprainfection. REF: pp. 1040-1041

9. A patient is to undergo orthopedic surgery, and the prescriber will order a cephalosporin to be given preoperatively as prophylaxis against infection. The nurse expects the provider to order which cephalosporin? a. First-generation cephalosporin b. Second-generation cephalosporin c. Third-generation cephalosporin d. Fourth-generation cephalosporin

ANS: A First-generation cephalosporins are widely used for prophylaxis against infection in surgical patients, because they are as effective, less expensive, and have a narrower antimicrobial spectrum than second-, third-, and fourth-generation cephalosporins. REF: pp. 1025-1027

13. A patient who is taking gentamicin and a cephalosporin for a postoperative infection requests medication for mild postsurgical pain. The nurse will expect to administer which of the following medications? a. Acetaminophen b. Aspirin c. Ibuprofen d. Morphine

ANS: A Gentamicin and cephalosporins are both nephrotoxic. This patient should avoid taking other potentially nephrotoxic drugs. Acetaminophen is not nephrotoxic and may be given for mild pain. Aspirin and ibuprofen are both nephrotoxic. Morphine is not nephrotoxic but is not indicated for mild pain. REF: pp. 1053-1054

1. A nurse transcribes a new prescription for potassium penicillin G given intravenously (IV) every 8 hours and gentamicin given IV every 12 hours. Which is the best schedule for administering these drugs? a. Give the penicillin at 0800, 1600, and 2400; give the gentamicin [Garamycin] at 1800 and 0600. b. Give the penicillin at 0800, 1600, and 2400; give the gentamicin [Garamycin] at 1200 and 2400. c. Give the penicillin at 0600, 1400, and 2200; give the gentamicin [Garamycin] at 0600 and 1800. d. Give the penicillin every 8 hours; give the gentamicin [Garamycin] simultaneously with two of the penicillin doses.

ANS: A Gentamicin should never be administered concurrently with penicillin, because they will interact, and the penicillin may inactivate the aminoglycoside. All the other options show concurrent administration. REF: p. 1015 / p. 1021

1. Besides the cost of administering a given drug, which are considerations when a provider selects a cephalosporin to treat an infection? (Select all that apply.) a. Adverse effects b. Antimicrobial spectrum c. Brand name d. Manufacturer e. Pharmacokinetics

ANS: A, B, E Cephalosporins tend to be more alike than different, but some differences make some preferable to others. Adverse effects, antimicrobial spectrum, and drug pharmacokinetics affect a prescriber's decision to select one drug over another. The agent's brand name and manufacturer should not have anything to do with drug selection. REF: p. 1028

2. A patient is diagnosed with an infection caused by Staphylococcus aureus, and the prescriber orders intravenous gentamicin and penicillin (PCN). Both drugs will be given twice daily. What will the nurse do? a. Administer gentamicin, flush the line, and then give the penicillin. b. Give the gentamicin intravenously and the penicillin intramuscularly. c. Infuse the gentamicin and the penicillin together to prevent fluid overload. d. Request an order to change the penicillin to vancomycin.

ANS: A Gentamicin should not be infused with penicillins in the same solution, because PCN inactivates gentamicin; therefore, the nurse should give one first, flush the line, and then give the other. The nurse cannot give a drug IM when it is ordered IV without an order from the prescriber. These two drugs should not be infused in the same solution. There is no indication for changing the PCN to vancomycin; that should be done for serious infections. REF: pp. 1054-1055

13. A nurse is teaching a patient who has taken glucocorticoids for over a year about glucocorticoid withdrawal. Which statement by the patient indicates a need for further teaching? a. "I should reduce the dose by half each day until I stop taking the drug." b. "I will need to have cortisol levels monitored during the withdrawal process." c. "The withdrawal schedule may take several months." d. "If I have surgery, I may need to take the drug for a while, even after I have stopped."

ANS: A Glucocorticoid therapy can suppress adrenal function, so withdrawal should be done slowly to allow recovery of adrenal function. Reducing the dose of a glucocorticoid by half each day is not recommended. Patients should have their cortisol levels monitored to determine when therapy can be stopped. The withdrawal schedule may take several months. Patients who have stopped the drug may still experience adrenal insufficiency in times of physiologic stress, such as surgery. REF: p. 874

14. A patient who has arthritis has been taking ibuprofen [Motrin] and a glucocorticoid medication. The patient reports having tarry stools but denies gastric pain. Which action by the nurse is correct? a. Contact the provider to discuss ordering an antiulcer medication. b. Counsel the patient to use over-the-counter antacids. c. Reassure the patient not to worry unless there is gastric pain. d. Tell the patient to stop taking the glucocorticoid immediately.

ANS: A Glucocorticoid therapy, especially when combined with NSAIDs, can increase the risk of gastric ulcer and possibly GI bleeding. Treatment with antiulcer medications is indicated, but not with OTC antacids, since the provider needs to be aware of this adverse effect. Gastric pain is usually decreased because of the glucocorticoids, so absence of gastric pain is not reassuring. The glucocorticoid should be withdrawn slowly, not immediately. REF: p. 873

14. A patient has been receiving intravenous ketorolac 30 mg every 6 hours for postoperative pain for 4 days. The patient will begin taking oral ketorolac 10 mg every 4 to 6 hours to prepare for discharge in 1 or 2 days. The patient asks the nurse whether this drug will be prescribed for management of pain after discharge. The nurse will respond by telling the patient that the provider will prescribe a(n): a. different nonsteroidal anti-inflammatory drug for home management of pain. b. fixed-dose opioid analgesic/nonsteroidal anti-inflammatory medication. c. lower dose of the oral ketorolac for long-term pain management. d. intranasal preparation of ketorolac for pain management at home.

ANS: A Ketorolac is not indicated for chronic or minor pain and should not be used longer than 5 days. Patients discharged home will be instructed to use other NSAIDs for pain. A fixed-dose opioid/NSAID is not indicated. Low-dose ketorolac would not be used, because 5 days would have passed. The intranasal therapy would not be indicated after 5 days. REF: p. 859

12. A woman who has arthritis is breast-feeding her infant and asks the nurse if it is safe to take NSAIDs while nursing. What will the nurse tell this patient? a. NSAIDs are safe to take while breast-feeding. b. NSAIDs are not safe, even in small amounts. c. She should take only COX-2 inhibitors while breast-feeding. d. She should request a prescription for narcotic analgesics.

ANS: A NSAIDs are safe and may be taken while breast-feeding. It is not necessary to use a COX-2 inhibitor while breast-feeding. Narcotics cross breast milk and should be used cautiously while breast-feeding. REF: p. 855

11. A nurse is providing teaching for an adult patient with arthritis who has been instructed to take ibuprofen [Motrin] for discomfort. Which statement by the patient indicates a need for further teaching? a. "I may experience tinnitus with higher doses of this medication." b. "I may take up to 800 mg 4 times daily for pain." c. "I should limit alcohol intake to fewer than three drinks a day." d. "I will take this medication with meals to help prevent stomach upset."

ANS: A NSAIDs do not cause salicylism and therefore do not cause tinnitus with higher doses. The maximum dose for adults is 3200 mg/day, or 800 mg 4 times/day. Patients taking NSAIDs should be cautioned to limit alcohol intake. Taking NSAIDs with meals helps prevent GI upset. REF: pp. 856-857

In a discussion of drug-drug interactions, which would be the best example of a beneficial inhibitory interaction? a. Naloxone [Narcan] blocking morphine sulfate's actions b. Antacids blocking the action of tetracycline [Sumycin] c. Propanolol [Inderal] blocking the effects of albuterol d. Cholestyramine blocking the actions of antihypertensive drugs

ANS: A Naloxone is used when a narcotic overdose has occurred. As a narcotic antagonist, it provides a beneficial inhibitory interaction. An antacid blocking tetracycline's antibiotic effects would not be beneficial, but rather detrimental to the desired effects of the tetracycline. Alcohol would not block the effects of opiates, but would contribute to CNS depression. Cholestyramine and certain other adsorbent drugs, which are administered orally but do not undergo absorption, can adsorb other drugs onto themselves, thereby preventing absorption of page 58

8. A patient with no known drug allergies is receiving amoxicillin [Amoxil] PO twice daily. Twenty minutes after being given a dose, the patient complains of shortness of breath. The patient's blood pressure is 100/58 mm Hg. What will the nurse do? a. Contact the provider and prepare to administer epinephrine. b. Notify the provider if the patient develops a rash. c. Request an order for a skin test to evaluate possible PCN allergy. d. Withhold the next dose until symptoms subside.

ANS: A This patient is showing signs of an immediate penicillin allergy, that is, one that occurs within 2 to 30 minutes after administration of the drug. The patient is showing signs of anaphylaxis, which include laryngeal edema, bronchoconstriction, and hypotension; these must be treated with epinephrine. This is an emergency, and the provider must be notified immediately, not when other symptoms develop. It is not necessary to order skin testing. The patient must be treated immediately, and subsequent doses should not be given. REF: p. 1020

A patient is given a new medication and reports nausea within an hour after taking the drug. The nurse consults the drug information manual and learns that nausea is not an expected adverse effect of this drug. When the next dose is due, what will the nurse do? a. Administer the drug and tell the patient to report further nausea. b. Hold the drug and notify the provider of the patient's symptoms. c. Report the symptoms of nausea to the MEDWATCH program. d. Request an order for an antiemetic to counter this drug's effects.

ANS: A Not all adverse drug reactions (ADRs) can be detected during clinical trials, and nurses should be alert to any effects that may result from drug administration. Because nausea is not a serious effect and because it is not yet known whether the drug is the cause of this patient's nausea, the nurse should administer the medication and observe the patient for recurrence of the symptom. It is not necessary to hold the drug, because nausea is not a serious side effect. The MEDWATCH program should be notified when there is a greater suspicion that the drug may have caused the nausea if the nausea occurs with subsequent doses. Until there is greater suspicion that the drug actually caused this patient's nausea, giving an antiemetic is not indicated. page 65-66

2. A pregnant adolescent patient asks the nurse whether she should continue to take her prescription for tetracycline [Sumycin] to clear up her acne. Which response by the nurse is correct? a. "Tetracycline can be harmful to the baby's teeth and should be avoided." b. "Tetracycline is safe to take during pregnancy." c. "Tetracycline may cause allergic reactions in pregnant women." d. "Tetracycline will prevent asymptomatic urinary tract infections."

ANS: A Tetracyclines can cause discoloration of deciduous teeth of infants if taken by the mother after the fourth month of gestation. Tetracyclines should not be given to pregnant women. Tooth discoloration can be prevented if the drugs are not taken by pregnant women or by children under 8 years of age. Tetracycline is not appropriate for a pregnant patient. Pregnancy does not precipitate an allergic response to tetracycline. Tetracycline should not be used to prevent urinary tract infections (UTIs), especially in pregnant women. REF: pp. 1038-1039

A nurse consults a drug manual before giving a medication to an 80-year-old patient. The manual states that older-adult patients are at increased risk for hepatic side effects. Which action by the nurse is correct? a . Contact the provider to discuss an order for pretreatment laboratory work. b . Ensure that the drug is given in the correct dose at the correct time to minimize the risk of adverse effects. c . Notify the provider that this drug is contraindicated for this patient. d . Request an order to give the medication intravenously so that the drug does not pass through the liver.

ANS: A The drug manual indicates that this drug should be given with caution to elderly patients. Getting information about liver function before giving the drug establishes baseline data that can be compared with post-treatment data to determine whether the drug is affecting the liver. Giving the correct dose at the correct interval helps to minimize risk, but without baseline information, the effects cannot be determined. The drug is not contraindicated. page 7

A nurse is preparing to administer medications. Which patient would the nurse consider to have the greatest predisposition to an adverse reaction? a . A 30-year-old man with kidney disease b . A 75-year-old woman with cystitis c . A 50-year-old man with an upper respiratory tract infection d . A 9-year-old boy with an ear infection

ANS: A The individual with impaired kidney function would be at risk of having the drug accumulate to a toxic level because of potential excretion difficulties. Cystitis is an infection of the bladder and not usually the cause of excretion problems that might lead to an adverse reaction from a medication. A respiratory tract infection would not predispose a patient to an adverse reaction, because drugs are not metabolized or excreted by the lungs. A 9-year-old boy would not have the greatest predisposition to an adverse reaction simply because he is a child; nor does an ear infection put him at greater risk. page 9

A nurse is preparing to give an antibiotic to a patient who reports being allergic to antibiotics. Before giving the medication, what will the nurse do first? a. Ask whether the patient has taken this antibiotic for other infections b. Question the patient about allergies to other medications c. Request an order for a lower dose of the antibiotic d. Request an order for an antihistamine

ANS: A The nurse needs to assess whether the patient is truly allergic to this drug. Allergic reactions require previous exposure to the drug, so the nurse should ask whether the patient has taken this antibiotic before. If a patient is allergic to a drug, lowering the dose will not decrease the risk of allergic reaction. Antihistamines sometimes are given when patients must take a drug to which they are allergic. page 62-63

10. A nurse is preparing a pediatric patient for surgery and is teaching the patient and the child's parents about the use of the patient-controlled analgesia pump. The parents voice concern about their child receiving an overdose of morphine. What will the nurse do? a. Instruct the parents not to activate the device when their child is sleeping. b. Reassure the parents that drug overdose is not possible with PCA. c. Suggest that the child use the PCA sparingly. d. Tell the patient that the pump can be programmed for PRN dosing only.

ANS: A The nurse should instruct parents not to activate the PCA when their child is sleeping because that can lead to drug overdose. Postoperative pain should be treated appropriately with medications that are effective. Nonopioid medications are not sufficient to treat postoperative pain. Patients should be encouraged to use the PCA as needed so that pain can be controlled in a timely fashion. PRN dosing is not as effective as dosing that is continuous, so a basal dose should be given as well as a PRN dose. REF: p. 264 / pp. 276-277

1. A nurse is providing education about tetracycline [Sumycin]. Which statement by the patient best demonstrates understanding of the administration of this medication? a. "I should not take this medication with milk or other dairy products." b. "I should not worry if I experience an acnelike rash with this medication." c. "I should take an antacid, such as Tums, if I experience gastrointestinal distress." d. "I should take this antibiotic with a calcium supplement to improve absorption."

ANS: A The patient should avoid taking the medication with dairy products to help prevent chelation. An acnelike reaction would indicate an allergic response. Taking the medication with calcium-containing antacids or supplements should be avoided, because this also leads to chelation. REF: pp. 1038-1039

6. The prescriber orders 20 mg of hydrocortisone orally once each day. The nurse will make sure the drug is scheduled to be administered at what time? a. 8:00 AM b. 12:00 PM c. 4:00 PM d. 9:00 PM

ANS: A To allow the adrenals to recover, the hydrocortisone dose should be administered before 9:00 AM. The other times are incorrect for oral administration of daily hydrocortisone. REF: p. 876

14. A patient is receiving gentamicin once daily. A nursing student asks the nurse how the drug can be effective if given only once a day. The nurse explains drug dosing schedules for aminoglycosides. Which statement by the student indicates a need for further teaching? a. "Gentamicin has a longer half-life than other aminoglycosides." b. "Large doses given once daily yield higher peak levels." c. "The postantibiotic effect lasts for several hours." d. "There is less risk of ototoxicity and nephrotoxicity with large daily doses."

ANS: A When a daily dose is given once daily instead of divided into 2 or 3 doses, a higher peak level can be achieved. The higher peak, along with the fact that aminoglycosides have a postantibiotic effect, means that the bacterial kill is just as great with one dose as with 2 or 3 doses per day. When a single daily dose is given, the risk of toxicity is reduced. Gentamicin does not have a longer half-life than other aminoglycosides. REF: p. 1054

6. A patient is admitted to the unit for treatment for an infection. The patient receives IV amikacin [Amikin] twice a day. When planning for obtaining a peak aminoglycoside level, when should the nurse see that the blood is drawn? a. 30 minutes after the IV infusion is complete b. 1 hour after the IV infusion is complete c. 1 hour before administration of the IV infusion d. A peak level is not indicated with twice-daily dosing.

ANS: A When divided daily doses are used, blood samples for measurement of peak levels are drawn 1 hour after IM injection and 30 minutes after completion of an IV infusion. This medication is administered IV, so blood draws must follow 30 minutes after infusion to obtain peak levels. Measurement of peak levels is unnecessary only when a single daily dose is used. REF: p. 1054

A patient tells a nurse that a medication prescribed for recurrent migraine headaches is not working. What will the nurse do? a . Ask the patient about the number and frequency of tablets taken. b . Assess the patient's headache pain on a scale from 1 to 10. c . Report the patient's complaint to the prescriber. d . Suggest biofeedback as an adjunct to drug therapy.

ANS: A When evaluating the effectiveness of a drug, it is important to determine whether the patient is using the drug as ordered. Asking the patient to tell the nurse how many tablets are taken and how often helps the nurse determine compliance. Assessing current pain does not yield information about how well the medication is working unless the patient is currently taking it. The nurse should gather as much information about compliance, symptoms, and drug effectiveness as possible before contacting the prescriber. Biofeedback may be an effective adjunct to treatment, but it should not be recommended without complete information about drug effectiveness. page 9

2. A patient has an infection caused by Pseudomonas aeruginosa. The prescriber has ordered piperacillin and amikacin, both to be given intravenously. What will the nurse do? a. Make sure to administer the drugs at different times using different IV tubing. b. Suggest giving larger doses of piperacillin and discontinuing the amikacin. c. Suggest that a fixed-dose combination of piperacillin and tazobactam [Zosyn] be used. d. Watch the patient closely for allergic reactions, because this risk is increased with this combination.

ANS: A When penicillins are present in high concentrations, they interact with aminoglycosides and inactivate the aminoglycoside; therefore, these two drugs should never be mixed in the same IV solution. The drugs should be given at different times with different tubing. In the treatment of Pseudomonas infections, extended-spectrum penicillins, such as piperacillin, usually are given in conjunction with an antipseudomonal aminoglycoside, such as amikacin; therefore, suggesting a larger dose of piperacillin and discontinuation of the amikacin is incorrect. Zosyn is not recommended. The risk of allergic reactions does not increase with this combination of drugs. REF: p. 1015 / p. 1021 / p. 1023

4. A patient is diagnosed with a lung infection caused by P. aeruginosa. The culture and sensitivity report shows sensitivity to all aminoglycosides. The nurse knows that the rate of resistance to gentamicin is common in this hospital. The nurse will expect the provider to order which medication? a. Amikacin [Amikin] b. Gentamicin c. Paromomycin d. Tobramycin

ANS: A When resistance to gentamicin and tobramycin is common, amikacin is the drug of choice for initial treatment of aminoglycoside-sensitive infections. Gentamicin would not be indicated, because resistance is more likely to develop. Paromomycin is used only for local effects within the intestine and is given orally. Tobramycin is not indicated, because organisms can more readily develop resistance. REF: p. 1050 | p. 1055

15. A woman who is breastfeeding is prescribed a low pharmacologic dose of a glucocorticoid and asks the nurse about potential effects on her infant. What will the nurse tell her about this medication? a. "At this dose, the concentration in your breast milk is safe." b. "Contact your provider to discuss lowering the dose." c. "There will be reversible side effects for your baby." d. "This drug is likely to cause growth retardation in your baby."

ANS: A Women who are nursing may take physiologic or low pharmacologic doses of glucocorticoids without achieving concentrations in breast milk that affect the nursing infant. It is not necessary to lower the dose. Since there are negligible levels, there are no effects on the infant. Large pharmacologic doses can cause growth retardation and other adverse effects in the infant. REF: pp. 874-875

A nurse is caring for a woman with breast cancer who is receiving tamoxifen. A review of this patient's chart reveals a deficiency of the CYP2D6 gene. The nurse will contact the provider to suggest: a. a different medication. b. an increased dose. c. a reduced dose. d. serum drug levels.

ANS: A Women with a deficiency of the CYP2D6 gene lack the ability to convert tamoxifen to its active form, endoxifen, and will not benefit from this drug. Another drug should be used to treat this patient's breast cancer. Increasing the dose, reducing the dose, or monitoring serum drug levels will not make this drug more effective in these women. page 76-77

Which groups of people are especially sensitive to medication effects? (Select all that apply.) a. Older adults b. Caucasians c. Infants d. Minorities e. Women

ANS: A, C Older adults and infants are the two groups most sensitive to drugs because of differences in organs that absorb, metabolize, and excrete drugs. In the older adult, organ degeneration accounts for these differences, whereas in infants the differences are related to organ immaturity. Racial and gender differences tend to be related to genetic differences and not race and gender per se. These groups are more sensitive to drug effects in some cases and less sensitive in other cases. page 78-79

2. A nursing student wants to know the differences between hospital-associated methicillin-resistant Staphylococcus aureus (HA-MRSA) and community-associated methicillin-resistant Staphylococcus aureus (CA-MRSA). Which statements about CA-MRSA are true? (Select all that apply.) a. 20% to 30% of the general population are colonized with CA-MRSA. b. Boils caused by CA-MRSA can be treated without antibiotics. c. CA-MRSA is less dangerous than HA-MRSA. d. CA-MRSA does not cause necrotizing fasciitis. e. CA-MRSA is transmitted by airborne droplets.

ANS: A, B, C CA-MRSA is thought to be present in 20% to 30% of the population, and many of these individuals are asymptomatic carriers. Boils caused by CA-MRSA can often be treated by surgical drainage alone. CA-MRSA is less dangerous than HA-MRSA but more dangerous than methicillin-sensitive Staphylococcus aureus (MSSA). CA-MRSA generally causes mild skin infections but can cause more serious infections, such as necrotizing fasciitis. CA-MRSA is transmitted by skin-to-skin contact and by contact with contaminated objects. REF: pp. 1017-1018

1. Which patients may receive gentamicin safely? (Select all that apply.) a. A 6-day-old newborn b. A 5-year-old child c. A pregnant woman d. A woman breast-feeding an infant e. An elderly patient with renal disease

ANS: A, B, D Aminoglycosides may be safely given to infants younger than 8 days old and to children and adolescents. Use during pregnancy is not recommended because they may harm the fetus. Gentamicin is probably safe during lactation. Gentamicin is used with caution in older patients with reduced renal function. REF: p. 1052

Which drugs will not be affected by interpatient variability? (Select all that apply.) a. Antiseptics applied to the skin to slow bacterial growth b. Antacids to help with the discomfort of heartburn c. Broad-spectrum antibiotics that are effective against many organisms d. Chelating agents that remove metal compounds from the body e. Topical analgesics used to treat localized pain

ANS: A, B, D Antiseptics, antacids, and chelating agents are all receptorless drugs that do not depend on the body's processes for effects; these agents react with other molecules. Broad-spectrum antibiotics and topical analgesics bind with receptors to produce desired effects, and these processes can be influenced by individual patient variables. page 51

A nurse is caring for a patient who is taking multiple medications. To help ensure that adverse drug reactions are prevented or minimized, the nurse will do which of the following? (Select all that apply.) a. Ask the patient about over-the-counter medications used. b. Contact the prescriber to request cytochrome P450 levels. c. Limit the patient's calcium intake. d. Obtain a thorough diet history. e. Request orders for PRN medications to treat any anticipated symptoms of drug interactions.

ANS: A, B, D Over-the-counter medications add to drug interactions, and a thorough history of all medications taken by the patient is essential to minimize adverse drug reactions. Cytochrome P450 levels yield important information about a patient's ability to metabolize drugs and can help predict whether drugs will reach toxic levels or be ineffective. A diet history allows providers to anticipate significant known food-drug interactions. Limiting calcium intake is necessary only if the patient is taking drugs known to interact with calcium, such as tetracycline. Asking for PRN medications to treat drug reactions may only compound the risk, because the risk of drug interactions increases with the number of medications taken. page 59-60

1. A nurse is administering morphine sulfate to a postoperative patient. Which are appropriate routine nursing actions when giving this drug? (Select all that apply.) a. Counting respirations before and after giving the medication b. Encouraging physical activity and offering increased fluids c. Monitoring the patient's blood pressure closely for hypertension d. Palpating the patient's lower abdomen every 4 to 6 hours e. Requesting an order for methylnaltrexone [Relistor] to prevent constipation

ANS: A, B, D Respiratory depression, constipation, and urinary retention are common adverse effects of opioid analgesics. It is important to count respirations before giving the drug and periodically thereafter to make sure that respiratory depression has not occurred. Increased physical activity, increased fluid intake, and increased fiber help alleviate constipation. It is important to assess the patient's abdomen and palpate the bladder to make sure that urinary retention has not occurred. Patients taking morphine often experience hypotension, not hypertension. Methylnaltrexone is given as a last resort to treat constipation, because it blocks mu receptors in the intestine. REF: pp. 261-264

The nurse is administering morning medications. The nurse gives a patient multiple medications, two of which compete for plasma albumin receptor sites. As a result of this concurrent administration, the nurse can anticipate that what might occur? (Select all that apply.) a. Binding of one or both agents will be reduced. b. Plasma levels of free drug will rise. c. Plasma levels of free drug will fall. d. The increase in free drug will intensify effects. e. The increase in bound drug will intensify effects.

ANS: A, B, D When two drugs bind to the same site on plasma albumin, coadministration of those drugs produces competition for binding. As a result, binding of one or both agents is reduced, causing plasma levels of free drug to rise. The increase in free drug can intensify the effect, but it usually undergoes rapid elimination. The increase in plasma levels of free drug is rarely sustained. page 56

1. The focus of quality health care should be on which of the following items? (Select all that apply.) a. Excellent services b. Comprehensive communication c. Private hospital rooms d. Health team collaboration e. Culturally competent care

ANS: A, B, D, E Excellent services, communication, collaboration, and culturally competent care brings quality to the health care delivered to the patient. Private hospital rooms may be a preference by some patients, but they do not add to the quality of care. Giddens Health Care Quality

2. What are the major attributes of health care quality? (Select all that apply.) a. Conforms to standards b. Sound decision making c. High acuity patients d. Low health care costs e. Identifies adverse events

ANS: A, B, E Major attributes of health care quality include conformation to standards set by regulatory agencies, sound decision making regarding care, and identifying potential adverse events. High acuity of patients does not contribute to quality health care, because the care demand is increased, and low health care costs mean fewer services may be available. Giddens Health Care Quality

2. Individuals of low socioeconomic status are at an increased risk for infection because of which of the following? (Select all that apply.) a. Uninsured or underinsured status b. Easy access to health screenings c. High cost of medications d. Inadequate nutrition e. Mostly female gender

ANS: A, C, D Individuals of low socioeconomic status tend to be part of the underinsured or uninsured population. Lack of insurance decreases accessibility to health care in general and health screening services specifically. High costs of medication and nutritious food also make this population at higher risk for infection. Gender has not been shown to be an increased risk factor for infection in the lower socioeconomic population. Chapter 24 Giddens

3. The nurse is teaching a pharmacology refresher course to a group of nurses. A student asks what host factors affect the choice of agents in antimicrobial therapy. The nurse will tell the students that such host factors include what? (Select all that apply.) a. Age b. Gender c. Immune system status d. Infection site e. Weight

ANS: A, C, D Host factors, such as the patient's age, immune status, and the site of the infection, affect the choice of antimicrobial therapy used to treat an infection. Gender and weight do not affect the choice of antimicrobial but may affect the dose and route of administration. REF: p. 1010

Which patients are at increased risk for adverse drug events? (Select all that apply.) a. A 2-month-old infant taking a medication for gastroesophageal reflux disease b. A 23-year-old female taking an antibiotic for the first time c. A 40-year-old male who is intubated in the intensive care unit and taking antibiotics and cardiac medications d. A 7-year-old female receiving insulin for diabetes e. An 80-year-old male taking medications for COPD

ANS: A, C, E Patients at increased risk for adverse drug events include the very young, the very old, and those who have serious illnesses. Females, children, and young adults taking single medications do not have increased risks for adverse events. page 62

Which are effective ways to help prevent medication errors? (Select all that apply.) a. Developing nonpunitive approaches to track errors b. Focusing on caregivers who make errors c. Helping patients to be active, informed members of the healthcare team d. Naming, blaming, and shaming those who make errors e. Using electronic medical order entry systems

ANS: A, C, E To help prevent medication errors, it is important to create an environment for tracking errors that is nonpunitive so that caregivers can learn from mistakes and work together to change systems appropriately. Helping patients be active, informed members of the healthcare team is a useful tool in this process. Using electronic order entry helps eliminate confusion from poor handwriting and allows built-in systems to warn caregivers about possible overdoses, side effects, and drug interactions; it also helps ensure the right dose at the right time to the right patient. An approach that focuses on those who make mistakes by naming, blaming, and shaming is not productive and often results in personnel who cover up mistakes instead of working to make things better. page 67-68

The prescriber has ordered an antibiotic for a patient with a bacterial infection. The nurse provides patient education at discharge and instructs the patient to take the drug on an empty stomach. When should the patient take the drug? (Select all that apply.) a. 1 hour or more before a meal b. Only after an 8-hour fast c. Only after the patient has missed a meal d. At least 2 hours after a meal e. Shortly before a meal

ANS: A, D The absorption of some drugs can be significantly reduced by food; these drugs should be taken on an empty stomach, which is 1 hour or more before a meal or at least 2 hours after a meal. An 8-hour fast is not necessary; the patient does not need to miss a meal to take the medication; and it is not reasonable to have the patient on thin liquids for 12 hours. page 61

7. An 80-year-old male patient is in the intensive care unit has suffered a fractured femur. You are making rounds and notice he is somnolent, with no response to verbal or physical stimulation. He has been on round the clock opioid doses q 4 hours. What is the nurse's first action? a. Call the rapid response team to care for the patient immediately. b. Discontinue the opioids on the medication administration record. c. Assess the patient's blood pressure and pain level. d. Start a second intravenous line with a large bore catheter.

ANS: B After establishing unresponsiveness, the next action is to call a Rapid Response. The patient is not able to subjectively describe pain if unresponsive. Another IV line may be needed, but first the nurse should call for help. The opioids should be discontinued on the MAR; however the priority action is to call for help. Chapter 28 Giddens

6. A patient comes to the clinic with a complaint of painful, itchy feet. On interview, the patient tells the nurse that he is a college student living in a dormitory apartment that he shares with five other students. What teaching should the nurse provide for this patient? a. "Don't eat with the other students." b. "Avoid sharing razors and other personal items." c. "Have a complete blood count (CBC) checked monthly." d. "Disinfect showers and bathroom floors weekly after use."

ANS: B Avoidance of sharing personal items like razors and hairbrushes can decrease the spread of pathogens that cause inflammation and infection. Not eating with the others in his college apartment won't relieve or prevent the spread of infection. A CBC monthly will not treat or prevent inflammation. Showers should be disinfected before and after each use Chapter 23 Giddens

2. A patient is diagnosed with a sprain to her right ankle after a fall. The patient asks the nurse about using ice on her injured ankle. What is the nurse's best response? a. "Use ice only when the ankle hurts." b. "Ice should be applied for 15 to 20 minutes every 2 to 3 hours over the next 1 to 2 days." c. "Wrap an ice pack around the injured ankle for the next 24 to 48 hours." d. "Ice is not recommended for use on the sprain because it would inhibit the inflammatory response."

ANS: B Ice is used on areas of injury during the first 24 to 48 hours after the injury occurs to prevent damage to surrounding tissues from excessive inflammation. Ice should be used for a maximum of 20 minutes at a time every 2 to 3 hours. Ice must be used according to a schedule for it to be effective and not be overused. Using ice more often or for longer periods of time can cause additional tissue damage. Ice is recommended to inhibit the inflammatory process from damaging surrounding tissue. Chapter 23 Giddens

1. The nursery nurse should identify which newborn at significant risk for hypothermic alteration in thermoregulation? a. Large for gestational age b. Low birth weight c. Born at term d. Well nourished

ANS: B Low birth weight and poorly nourished infants (particularly premature infants) and children are at greatest risk for hypothermia. A large for gestational age infant would not be malnourished. An infant born at term is not considered at significant risk. A well nourished infant is not at significant risk. Chapter 10 Giddens

5. A 70-year-old retired nurse is interested in nondrug, mind-body therapies, self-management, and alternative strategies to deal with joint discomfort from rheumatoid arthritis. What options should you consider in her plan of care considering her expressed wishes? a. Stationary exercise bicycle, free weights, and spinning class b. Mind-body therapies such as music therapy, distraction techniques, meditation, prayer, hypnosis, guided imagery, relaxation techniques, and pet therapy c. Chamomile tea and IcyHot gel d. Acupuncture and attending church services

ANS: B Mind-body therapies are designed to enhance the mind's capacity to affect bodily function and symptoms and include music therapy, distraction techniques, meditation, prayer, hypnosis, guided imagery, relaxation techniques, and pet therapy, among many others. Stationary exercise bicycle, free weights, and spinning are not mind-body therapies. They are classified as exercise therapies. Chamomile tea and IcyHot gel are not mid-body therapies per se. They are classified as herbal and topical thermal treatments. Acupuncture is an ancient Chinese complementary therapy, while attending church services is a religious prayer mind-body therapy capable of enhancing the mind's capacity to affect bodily function and symptoms. Chapter 28 Giddens

1. A 62-year-old male has fallen while trimming tree branches sustaining tissue injury. He describes his condition as an aching, throbbing back. This is characteristic of what type of pain? a. Neuropathic pain b. Nociceptive pain c. Chronic pain d. Mixed pain syndrome

ANS: B Nociceptive pain refers to the normal functioning of physiological systems that leads to the perception of noxious stimuli (tissue injury) as being painful. Patients describe this type of pain as aching, cramping, or throbbing. Neuropathic pain is pathologic and results from abnormal processing of sensory input by the nervous system as a result of damage to the brain, spinal cord, or peripheral nerves. Patients describe this type of pain as burning, sharp, and shooting. Chronic pain is constant and unrelenting such as pain associated with cancer. Mixed pain syndrome is not easily recognized, is unique with multiple underlying and poorly understood mechanisms like fibromyalgia and low back pain. Chapter 28 Giddens

A nurse is giving an enteral medication. The patient asks why this method is preferable for this drug. How will the nurse reply? a. "This route allows more rapid absorption of the drug." b. "This route is safer, less expensive, and more convenient." c. "This route is the best way to control serum drug levels." d. "This route prevents inactivation of the drug by digestive enzymes."

ANS: B Parenteral routes include the intravenous, intramuscular, and subcutaneous routes. Enteral routes include oral administration, including pills and liquid suspensions. Enteral routes are safer, cheaper, and easier to use. Parenteral routes are used when rapid absorption, precise control of plasma drug levels, and prevention of digestive inactivation are important. page 31-32

3. A student nurse is talking with his instructor. The student asks how quality of care is evaluated. What is the best response by the instructor? a. "Quality of care is evaluated by the patient getting well." b. "Quality of care is evaluated on the basis of process and outcomes." c. "Quality of care is evaluated by the physician's assessment." d. "Quality of care is evaluated by the patient's satisfaction."

ANS: B Quality of care is evaluated by process and outcomes. If the outcomes are achieved, then the care has achieved what is was designed to do. The patient getting well may be an action of the body doing what it is supposed to do and not quality of care; the same can be said of the physician's assessment. The patient's satisfaction is subjective according to his or her perceptions and not the quality of care. Giddens Health Care Quality

5. Which strategies should the nurse include in a community program for senior citizens related to dealing with cold winter temperatures? a. Avoiding hot beverages b. Shopping at an indoor mall c. Using a fan at low speed d. Walking slowly in the park

ANS: B Shopping indoors where there is protection from the elements and temperature control is one strategy to avoid cold temperatures. Hot beverages can help an individual deal with cold weather. Avoiding breezes and air currents is recommended to conserve body temperature. Physical activity can increase body temperature, and if the senior is going to walk in the park, weather-appropriate (warm) clothing and a usual or brisk pace, not a slow pace, would be recommended. Chapter 10 Giddens

4. What are two major foci of The Joint Commission (TJC) in the delivery of health care? a. Cost containment; safety b. Safety; quality c. Quality; assessment d. Assessment; evaluation

ANS: B The focus of TJC is quality and safety for patient care. TJC does not address cost containment, assessment of care, or evaluation of care. Giddens Health Care Quality

7. What is the most appropriate measure for a nurse to use in assessing core body temperature when there are suspected problems with thermoregulation? a. Oral thermometer b. Rectal thermometer c. Temporal thermometer scan d. Tympanic membrane sensor

ANS: B The most reliable means available for assessing core temperature is a rectal temperature, which is considered the standard of practice. An oral temperature is a common measure but not the most reliable. A temporal thermometer scan has some limitations and is not the standard. The tympanic membrane sensor could be used as a second source for temperature assessment. Chapter 10 Giddens

A patient is taking a drug that does not bind to albumin. Which aspect of renal drug excretion is affected by this characteristic? a. Active tubular secretion b. Glomerular filtration c. Passive tubular reabsorption d. pH-dependent ionization

ANS: B As blood flows through the glomerular capillaries, fluids and small molecules are forced through the pores of the capillary wall. Large molecules, such as drugs bound to albumin, remain behind in the blood. Active tubular secretion requires active transport systems to pump molecules from one side of the membrane to another. Passive tubular reabsorption allows substances in high concentrations in the tubule to be reabsorbed into the blood, and lipid-soluble drugs readily cross this membrane by passive reabsorption. pH-dependent ionization affects drugs that ionize in either acid or alkaline environments. page 37

3. Which set of assessment data is consistent for a patient with severe infection that could lead to system failure? a. Blood pressure (BP) 92/52, pulse (P) 56 beats/min, respiratory rate (RR) 10 breaths/min, urine output 1200 mL in past 24 hours b. BP 90/48, P 112 beats/min, RR 26 breaths/min, urine output 240 mL in past 24 hours c. BP 112/64, P 98 beats/min, RR 18 breaths/min, urine output 2400 mL in past 24 hours d. BP 152/90, P 52 beats/min, RR 12 breaths/min, urine output 4800 mL in past 24 hours

ANS: B The patient with severe infection presents with low BP and compensating elevations in pulse to move lower volumes of blood more rapidly and respiration to increase access to oxygen. Urine output decreases to counteract the decreased circulating blood volume and hypotension. These vital signs are all too low: Blood pressure (BP) 92/52, pulse (P) 56 beats/min, respiratory rate (RR) 10 breaths/min, urine output 1200 mL in past 24 hours. The patient with severe infection does have a low BP, but the pulse and respiratory rate increase to compensate. This data is all within normal limits: BP 112/64, P 98 beats/min, RR 18 breaths/min, urine output 2400 mL in past 24 hours. This set of data reflects an elevated BP with a decrease in pulse and respiratory rates along with normal urine output: BP 152/90, P 52 beats/min, RR 12 breaths/min, urine output 4800 mL in past 24 hours. None of these is a typical response to severe infection. Chapter 24 Giddens

1. The nurse is caring for a patient who is being discharged home after a splenectomy. What information on immune function needs to be included in this patient's discharge planning? a. The mechanisms of the inflammatory response b. Basic infection control techniques c. The importance of wearing a face mask in public d. Limiting contact with the general population

ANS: B The spleen is one of the major organs of the immune system. Without the spleen, the patient is at higher risk for infection; so, the nurse must be sure that the patient understands basic principles of infection control. The patient with a splenectomy does not need to understand the mechanisms of inflammatory response. The patient with a splenectomy does not need to wear a face mask in public as long as the patient understands and maintains the basic principles of infection control. The patient who has had a splenectomy does not need to limit contact with the general population as long as the patient understands and maintains the basic principles of infection control. Chapter 22 Giddens

A nurse is preparing to administer a drug. Upon reading the medication guide, the nurse notes that the drug has been linked to symptoms of Parkinson's disease in some patients. What will the nurse do? a. Ask the patient to report these symptoms, which are known to be teratogenic effects. b. Observe the patient closely for such symptoms and prepare to treat them if needed. c. Request an order to evaluate the patient's genetic predisposition to this effect. d. Warn the patient about these effects and provide reassurance that this is expected.

ANS: B A drug that causes disease-like symptoms is known to be iatrogenic. Nurses should be prepared for this possibility and be prepared to withdraw the drug if necessary and treat the symptoms. Such effects are not teratogenic. Patients with a genetic predisposition to respond differently to drugs are known to have idiosyncratic effects. Iatrogenic effects, even when known, are not typically expected side effects. page 62-63

10. A patient is receiving an intraperitoneal aminoglycoside during surgery. To reverse a serious side effect of this drug, the nurse may expect to administer which agent? a. Amphotericin B b. Calcium gluconate c. Neuromuscular blocker d. Vancomycin

ANS: B Aminoglycosides can inhibit neuromuscular transmission, especially during intraperitoneal or intrapleural instillation, and this risk is increased when neuromuscular blocking agents and general anesthetics are given. Calcium can reverse neuromuscular blockade. Amphotericin B, additional neuromuscular blockers, and vancomycin are not indicated. REF: p. 1053

6. A patient has a viral sinus infection, and the provider tells the patient that antibiotics will not be prescribed. The patient wants to take an antibiotic and asks the nurse what possible harm could occur by taking an antibiotic. Which response by the nurse is correct? a. "Antibiotics are mutagenic and can produce changes that cause resistance." b. "Even normal flora can develop resistance and transfer this to pathogens." c. "Host cells become resistant to antibiotics with repeated use." d. "Patients who overuse antibiotics are more likely to have nosocomial infections."

ANS: B Antibiotics make conditions favorable for the overgrowth of microbes with acquired resistance. Normal flora, present at all times, can develop resistance and can transfer this resistance to pathogens if they occur. Even when pathogens are not present, antibiotic use can promote resistance in the future. Antibiotics are not mutagenic. Host cells are not affected. Antibiotic use does not increase the risk of nosocomial infection in a particular patient but does increase resistance in resident organisms in a particular hospital. REF: p. 1005

8. A pregnant patient in her third trimester asks the nurse whether she can take aspirin for headaches. Which response by the nurse is correct? a. "Aspirin is safe during the second and third trimesters of pregnancy." b. "Aspirin may cause premature closure of the ductus arteriosus in your baby." c. "Aspirin may induce premature labor and should be avoided in the third trimester." d. "You should use a first-generation nonsteroidal anti-inflammatory medication."

ANS: B Aspirin poses risks to the pregnant patient and her fetus, including premature closure of the ductus arteriosus. ASA is not safe, especially in the third trimester, because it can cause anemia and can contribute to postpartum hemorrhage. ASA does not induce labor but can prolong labor by inhibiting prostaglandin synthesis. NSAIDs have similar effects and also should be avoided. REF: p. 855

1. A nurse is teaching a group of nursing students about cyclooxygenase (COX) inhibitors. A student asks the nurse about characteristics of COX-1 inhibitors. Which statement by the nurse is true? a. "COX-1 inhibitors protect against colorectal cancer." b. "COX-1 inhibitors protect against myocardial infarction and stroke." c. "COX-1 inhibitors reduce fever." d. "COX-1 inhibitors suppress inflammation."

ANS: B COX-1 inhibitors have beneficial effects of reducing platelet aggregation and thus reducing the risk of myocardial infarction and stroke. COX-2 inhibitors protect against colorectal cancer, reduce inflammation, and reduce fever. REF: p. 849

8. A 60-year-old female patient is about to begin long-term therapy with a glucocorticoid. Which of the following will be important for minimizing the risk of osteoporosis? a. Baseline vitamin D level b. Calcium and vitamin D supplements c. Estrogen therapy d. Skeletal x-rays before treatment

ANS: B Calcium and vitamin D supplements can help minimize the patient's risk of developing osteoporosis. A baseline vitamin D level is not recommended. Estrogen therapy can help in postmenopausal women, but its risks outweigh its benefits. Patients should undergo evaluation of the bone mineral density of the lower spine, not skeletal x-rays. REF: p. 872

13. A nurse is providing medication teaching for a patient who will begin taking diclofenac [Voltaren] gel for osteoarthritis in both knees and elbows. Which statement by the patient indicates understanding of the teaching? a. "Because this is a topical drug, liver toxicity will not occur." b. "I should cover areas where the gel is applied to protect them from sunlight." c. "I will apply equal amounts of gel to all affected areas." d. "The topical formulation has the same toxicity as the oral formulation."

ANS: B Diclofenac is available in topical and oral preparations. Patients should be warned to protect treated areas from sunlight. Side effects occur, such as liver toxicity, even with topical dosing. Patients should apply smaller amounts to the upper extremities. Systemic toxicity is lower with topical formulations. REF: p. 858

9. A patient who takes the loop diuretic ethacrynic acid is given intravenous gentamicin for an infection. After several days of treatment with gentamicin, the nurse reviews the patient's most recent laboratory results and notes a gentamicin trough of 2.1 mcg/mL and normal blood urea nitrogen (BUN) and serum creatinine levels. The nurse will question the patient about: a. gastrointestinal (GI) symptoms. b. headache, dizziness, or vertigo. c. presence of rash. d. urine output.

ANS: B Ethacrynic acid has ototoxic properties, and patients who take this drug with an aminoglycoside have an increased risk of ototoxicity, especially when trough levels of the aminoglycoside are elevated. A trough level of 2.1 mcg/mL is above normal limits for gentamicin, so this patient should be asked about early signs of ototoxicity. There is no indication to evaluate for GI symptoms, rash, or urine output. REF: pp. 1052-1053 / p. 1054

3. A woman complains of burning on urination and increased frequency. The patient has a history of frequent urinary tract infections (UTIs) and is going out of town in 2 days. To treat the infection quickly, the nurse would expect the healthcare provider to order: a. aztreonam [Azactam]. b. fosfomycin [Monurol]. c. trimethoprim/sulfamethoxazole [Bactrim]. d. vancomycin [Vancocin].

ANS: B Fosfomycin has been approved for single-dose therapy of UTIs in women. Vancomycin and aztreonam are not indicated for UTIs. Bactrim is indicated for UTIs, but administration of a single dose is not therapeutic. REF: p. 1034

5. 5. A patient who is a long-distance runner has been diagnosed with rheumatoid arthritis in both knees and will begin glucocorticoid therapy. When teaching the patient about the medication, the nurse will include what information? a. "By reducing inflammation, this drug will slow the progression of your disease." b. "Glucocorticoids are used as adjunctive therapy during acute flare-ups." c. "Oral glucocorticoids cause less toxicity than intra-articular injections." d. "You may resume running when the pain and swelling improve."

ANS: B Glucocorticoids are used as adjunctive therapy to treat acute exacerbations of rheumatoid arthritis. Glucocorticoids reduce pain and inflammation but do not alter the course of the disease. Oral glucocorticoids produce side effects similar to those of all glucocorticoids, but they are absorbed more rapidly and completely than intra-articular injections. Patients should be warned against overactivity even though their pain has been reduced, because they can further injure their joints. REF: p. 871

4. A nurse is teaching a group of nursing students why glucocorticoids are preferred over nonsteroidal anti-inflammatory drugs in the treatment of inflammation. Which statement by a student indicates a need for further teaching? a. "Glucocorticoids act by multiple mechanisms and have more anti-inflammatory effects than NSAIDs." b. "Glucocorticoids have fewer side effects than nonsteroidal anti-inflammatory drugs." c. "Glucocorticoids help avert damage to tissues from lysosomal enzymes." d. "Glucocorticoids reduce the immune component of inflammation."

ANS: B Glucocorticoids have many side effects. They are used because of their multiple mechanisms of action, including their ability to limit tissue damage caused by the inflammatory process and suppression of the immune component of inflammation. REF: pp. 871-872

11. A child is to begin long-term glucocorticoid therapy. The parents ask the nurse about the effects of this drug on the child's growth. Which response by the nurse is correct? a. "A smaller dose may be indicated for your child." b. "Ask your provider about every other day dosing." c. "Long-acting glucocorticoid preparations should prevent growth suppression." d. "Oral glucocorticoids rarely cause growth suppression."

ANS: B Growth retardation can be minimized with alternate day dosing of glucocorticoids. Giving smaller doses does not prevent this effect with long-term therapy. Long-acting glucocorticoids do not prevent this effect. Oral glucocorticoids have the same side effects. REF: p. 873

A nurse is teaching nursing students about the use of nonproprietary names for drugs. The nurse tells them which fact about nonproprietary names? a . They are approved by the FDA and are easy to remember. b . They are assigned by the U.S. Adopted Names Council. c . They clearly identify the drug's pharmacologic classification. d . They imply the efficacy of the drug and are less complex.

ANS: B Nonproprietary, or generic, names are assigned by the U.S. Adopted Names Council, which ensures that each drug has only one name. Trade names, or brand names, are approved by the FDA and are easier to remember. Some nonproprietary names contain syllables that identify the classification, although not all do. Drug names are not supposed to identify the use for the drug, although some brand names do so. page 18

3. A patient with moderate to severe chronic pain has been taking oxycodone [OxyContin] 60 mg every 6 hours PRN for several months and tells the nurse that the medication is not as effective as before. The patient asks if something stronger can be taken. The nurse will contact the provider to discuss: a. administering a combination opioid analgesic/acetaminophen preparation. b. changing the medication to a continued-release preparation. c. confronting the patient about drug-seeking behaviors. d. withdrawing the medication, because physical dependence has occurred.

ANS: B Oxycodone is useful for moderate to severe pain, and a continued-release preparation may give more continuous relief. Dosing is every 12 hours, not PRN. A combination product is not recommended with increasing pain, because the nonopioid portion of the medication cannot be increased indefinitely. This patient does not demonstrate drug-seeking behaviors. Physical dependence is not an indication for withdrawing an opioid, as long as it is still needed; it indicates a need for withdrawing the drug slowly when the drug is discontinued. REF: p. 271

3. A nurse assisting a nursing student with medications asks the student to describe how penicillins (PCNs) work to treat bacterial infections. The student is correct in responding that penicillins: a. disinhibit transpeptidases. b. disrupt bacterial cell wall synthesis. c. inhibit autolysins. d. inhibit host cell wall function.

ANS: B PCNs weaken the cell wall, causing bacteria to take up excessive amounts of water and subsequently rupture. PCNs inhibit transpeptidases and disinhibit autolysins. PCNs do not affect the cell walls of the host. REF: p. 1015

17. A nurse is teaching a nursing student about dalfopristin/quinupristin [Synercid]. Which statement by the student indicates an understanding of the teaching? a. "Patients should stop taking the drug if they experience joint and muscle pain." b. "Patients taking this drug should have blood tests performed frequently." c. "Patients who are allergic to penicillin should not take this drug." d. "This drug will be administered intravenously over a 30- to 60-minute period."

ANS: B Patients taking dalfopristin/quinupristin should have blood levels measured twice the first week and then weekly thereafter to assess for hepatotoxicity. Joint and muscle pain are not an indication for withdrawing the drug. There is no cross-sensitivity to penicillin. The drug is given intravenously over a period of at least 1 hour. REF: p. 1045

A patient taking oral contraceptives thinks she may be pregnant. As part of this patient's history, what will the nurse ask the patient? a. "Do you drink grapefruit juice?" b. "Do you take seizure medication?" c. "Do you take your contraception with milk?" d. "Do you use laxatives regularly?"

ANS: B Patients taking oral contraceptives along with phenobarbital, which is used to treat seizures, will have lower levels of the contraceptive, because phenobarbital is an inducing agent, which causes an increase in the metabolism of oral contraceptives. Grapefruit juice inhibits the metabolism of some drugs, leading to toxic effects. Dairy products interfere with the absorption of tetracyclines, because the calcium binds with the drug to form an insoluble complex. Laxatives reduce the absorption of some drugs by speeding up the transit time through the gut. page 59-60

10. A patient receiving a cephalosporin develops a secondary intestinal infection caused by Clostridium difficile. What is an appropriate treatment for this patient? a. Adding an antibiotic, such as vancomycin [Vancocin], to the patient's regimen b. Discontinuing the cephalosporin and beginning metronidazole [Flagyl] c. Discontinuing all antibiotics and providing fluid replacement d. Increasing the dose of the cephalosporin and providing isolation measures

ANS: B Patients who develop C. difficile infection (CDI) as a result of taking cephalosporins or other antibiotics need to stop taking the antibiotic in question and begin taking either metronidazole or vancomycin. Adding one of these antibiotics without withdrawing the cephalosporin is not indicated. CDI must be treated with an appropriate antibiotic, so stopping all antibiotics is incorrect. Increasing the cephalosporin dose would only aggravate the CDI. REF: pp. 1031-1032

10. An older adult patient with chronic obstructive pulmonary disease (COPD) develops bronchitis. The patient has a temperature of 39.5°C. The nurse will expect the provider to: a. obtain a sputum culture and wait for the results before prescribing an antibiotic. b. order empiric antibiotics while waiting for sputum culture results. c. treat symptomatically, because antibiotics are usually ineffective against bronchitis. d. treat the patient with more than one antibiotic without obtaining cultures.

ANS: B Patients with severe infections should be treated while culture results are pending. If a patient has a severe infection or is at risk of serious sequelae if treatment is not begun immediately, it is not correct to wait for culture results before beginning treatment. Until a bacterial infection is ruled out, treating symptomatically is not indicated. Treating without obtaining cultures is not recommended. REF: pp. 1005-1006

The U.S. Food and Drug Administration (FDA) recommends genetic testing of patients receiving certain medications. Genetic testing helps prescribers: a. better establish a drug's therapeutic index. b. determine whether a patient is a rapid or slow metabolizer of the drug. c. identify racial characteristics that affect psychosocial variation in drug response. d. produce a drug that is tailored to an individual patient's genetic makeup.

ANS: B Pharmacogenomics is the study of the ways genetic variations affect individual responses to drugs through alterations in genes that code for drug-metabolizing enzymes and drug receptors. For some drugs, the FDA requires genetic testing, and for others, this testing is recommended but not required. Genetic testing does not determine a drug's therapeutic index; this is a measure of a drug's safety based on statistics of the drug's use in the general population (see Chapter 5). Any distinct physiologic differences in drug response among various racial populations are related to genetic differences and do not affect psychosocial differences in drug responses. Genetic testing is recommended to identify how a patient will respond to a drug and not to design a drug specific to an individual. page 75-76

Someone asks a nurse about a new drug that is in preclinical testing and wants to know why it cannot be used to treat a friend's illness. Which statement by the nurse is correct? a . "A drug at this stage of development can be used only in patients with serious disease." b . "At this stage of drug development, the safety and usefulness of the medication is unknown." c . "Clinical trials must be completed to make sure the drug is safe to use in humans." d . "Until postmarketing surveillance data are available, the drug cannot be used."

ANS: B Preclinical testing must be completed before drugs can be tested in humans. In this stage, drugs are evaluated for toxicities, pharmacokinetic properties, and potentially useful effects. Some drugs can be used in patients before completion of Phase III studies, but this is after preclinical testing is complete. Clinical trials proceed in stages, and each stage has guidelines defining how a new drug may be used and which patients may receive it. Postmarketing surveillance takes place after a drug is in general use. page 16

8. The nurse is caring for a patient who is receiving vancomycin [Vancocin]. The nurse notes the patient is experiencing flushing, rash, pruritus, and urticaria. The patient's heart rate is 120 beats per minute, and the blood pressure is 92/57 mm Hg. The nurse understands that these findings are consistent with: a. allergic reaction. b. red man syndrome. c. rhabdomyolysis. d. Stevens-Johnson syndrome.

ANS: B Rapid infusion of vancomycin can cause flushing, rash, pruritus, urticaria, tachycardia, and hypotension, a collection of symptoms known as red man syndrome. Rhabdomyolysis is not associated with the administration of vancomycin. The patient's symptoms may seem to indicate an allergic reaction, but this is specifically red man syndrome. The symptoms are not those of Stevens-Johnson syndrome, which manifests as blisters or sores (or both) on the lips and mucous membranes after exposure to the sun. REF: p. 1033

19. A patient with severe community-acquired pneumonia has been prescribed telithromycin [Ketek]. Which aspect of the patient's medical history is of concern to the nurse? a. Anemia b. Myasthenia gravis c. Renal disease d. Strep. pneumoniae infection

ANS: B Telithromycin is a macrolide antibiotic used only for CAP. Patients with myasthenia gravis may experience rapid muscle weakness after taking the drug, and some have died from respiratory failure, so patients with MG should not take this drug. This drug does not have significant myelosuppression, so anemia is not a concern. The drug causes liver injury, so liver disease, and not renal disease, is a concern. Telithromycin is indicated for treatment of S. pneumonia. REF: p. 1044

The FDA Amendments Act (FDAAA) was passed in 2007 to address which aspect of drug safety? a . Allowing pharmaceutical companies to identify off-label uses of medications approved for other uses b. Evaluating drug safety information that emerges after a drug has been approved and is in use c . Expediting the approval process of the U.S. Food and Drug Administration (FDA) so that needed drugs can get to market more quickly d . Requiring manufacturers to notify patients before removing a drug from the market

ANS: B The FDAAA was passed to enable the Food and Drug Administration to continue oversight of a drug after granting it approval so that changes in labeling could be made as necessary and postmarketing risks could be tracked and identified. A provision of the FDA Modernization Act (FDAMA), passed in 1997, allows drug companies to promote their products for off-label uses as long as they promise to conduct studies to support their claims. Regulations to permit accelerated approval of drugs for life-threatening diseases were adopted in 1992 by the FDA. The requirement that drug companies notify patients 6 months before removing a drug from the market is a provision of the FDAMA. page 14-15

3. A patient who is taking acetaminophen for pain wants to know why it does not cause gastrointestinal upset, as do other over-the-counter pain medications. The nurse will explain that this is most likely because of which property of acetaminophen? a. It does not inhibit cyclooxygenase. b. It has minimal effects at peripheral sites. c. It is more similar to opioids than to nonsteroidal anti-inflammatory drugs (NSAIDs). d. It is selective for cyclooxygenase-2.

ANS: B The differences between the effects of acetaminophen and aspirin are thought to result from selective inhibition of cyclooxygenase; acetaminophen has only minimal effects on cyclooxygenase at peripheral sites, which may explain why acetaminophen does not have adverse GI, renal, and antiplatelet effects. Acetaminophen is a selective COX inhibitor. It is not more similar to opioids than NSAIDs. It is not selective for COX-2. REF: p. 861

A postoperative patient who is worried about pain control will be discharged several days after surgery. The nurse providing discharge teaching tells the patient that the prescribed Lortab is not as strong as the morphine the patient was given in the immediate postoperative period. Which response is the patient likely to experience? a. A decreased likelihood of filling the prescription for the drug b. A negative placebo effect when taking the medication c. An increased compliance with the drug regimen d. Optimistic, realistic expectations about the drug

ANS: B The full extent of placebo effects, if they truly occur, is not well documented or understood, although a decrease in pain as a placebo effect has been demonstrated to some extent. To foster a beneficial placebo effect, it is important for all members of the healthcare team to present an optimistic and realistic assessment of the effects of the drug the patient is taking. If the nurse tells an anxious patient that the medication being given is not as strong as what has been given, the patient is likely to have lowered expectations of the effectiveness of the drug, causing a negative placebo effect. Lowered expectations do not mean that the patient will give up on the drug entirely; in fact, the patient may actually fill the prescription and then take more drug than what is prescribed to get a better effect. page 75 & 80

A postoperative patient reports pain, which the patient rates as an 8 on a scale from 1 to 10 (10 being the most extreme pain). The prescriber has ordered acetaminophen [Tylenol] 650 mg PO every 6 hours PRN pain. What will the nurse do? a . Ask the patient what medications have helped with pain in the past. b . Contact the provider to request a different analgesic medication. c . Give the pain medication and reposition the patient to promote comfort. d . Request an order to administer the medication every 4 hours.

ANS: B The nursing diagnosis for this patient is severe pain. Acetaminophen is given for mild to moderate pain, so the nurse should ask the prescriber to order a stronger analgesic medication. Asking the patient to tell the nurse what has helped in the past is part of an initial assessment and should be done preoperatively and not when the patient is having severe pain. Because the patient is having severe pain, acetaminophen combined with nondrug therapies will not be sufficient. Increasing the frequency of the dose of a medication for mild pain will not be effective. page 9

4. A patient is diagnosed with periodontal disease, and the provider orders oral doxycycline [Periostat]. The patient asks the purpose of the drug. What is the nurse's response? a. "It is used because of its anti-inflammatory effects." b. "It inhibits collagenase to protect connective tissue in the gums." c. "It reduces bleeding and the pocket depth of oral lesions." d. "It suppresses bacterial growth in the oral mucosa."

ANS: B Two tetracyclines are used for periodontal disease. Doxycycline inhibits collagenase, which destroys connective tissue in the gums. It is not used for anti-inflammatory effects. Minocycline is used to reduce bleeding and pocket depth and to inhibit bacterial growth. REF: p. 1038

A postoperative patient is being discharged home with acetaminophen/hydrocodone [Lortab] for pain. The patient asks the nurse about using Tylenol for fever. Which statement by the nurse is correct? a . "It is not safe to take over-the-counter drugs with prescription medications." b . "Taking the two medications together poses a risk of drug toxicity." c . "There are no known drug interactions, so this will be safe." d . "Tylenol and Lortab are different drugs, so there is no risk of overdose."

ANS: B Tylenol is the trade name and acetaminophen is the generic name for the same medication. It is important to teach patients to be aware of the different names for the same drug to minimize the risk of overdose. Over-the-counter (OTC) medications and prescription medications may be taken together unless significant harmful drug interactions are possible. Even though no drug interactions are at play in this case, both drugs contain acetaminophen, which could lead to toxicity. page 8

A nurse is teaching a group of women about medications. The women want to know why so many drugs have unpredictable effects in women. The nurse will tell them that: a. drugs usually have more toxic effects in women. b. most known drug effects are based on drug trials in men. c. women have varying responses to drugs during menstrual cycles. d. women metabolize drugs more slowly.

ANS: B Until 1997 almost all clinical drug trials were performed in men. Women may have more toxic effects with some drugs and fewer toxic effects with others. Not all drugs are influenced by hormonal changes. Women metabolize some drugs more slowly and other drugs more quickly. Unless drug trials are performed in both women and men, the effects of drugs in women will not be clear. page 78

What occurs when a drug binds to a receptor in the body? a. It alters the receptor to become nonresponsive to its usual endogenous molecules. b. It increases or decreases the activity of that receptor. c. It gives the receptor a new function. d. It prevents the action of the receptor by altering its response to chemical mediators.

ANS: B When a drug binds to a receptor, it mimics or blocks the actions of the usual endogenous regulatory molecules, either increasing or decreasing the rate of the physiologic activity normally controlled by that receptor. It does not alter the activity of the receptor and does not give the receptor a new function. page 46-47

15. A nurse is preparing to administer a dose of gentamicin to a patient who is receiving the drug 3 times daily. The nurse will monitor ____ levels. a. peak b. peak and trough c. serum drug d. trough

ANS: B When divided doses of aminoglycosides are given, it is important to measure both peak and trough levels of the drug, because it is more difficult to achieve therapeutic peaks in lower doses without causing toxicity. Trough levels are drawn when single-dosing regimens are used, because high peak levels are guaranteed. REF: p. 1054

4. A nurse is teaching a nursing student what is meant by "generations" of cephalosporins. Which statement by the student indicates understanding of the teaching? a. "Cephalosporins are assigned to generations based on their relative costs to administer." b. "Cephalosporins have increased activity against gram-negative bacteria with each generation." c. "First-generation cephalosporins have better penetration of the cerebrospinal fluid." d. "Later generations of cephalosporins have lower resistance to destruction by beta-lactamases."

ANS: B With each progression from first-generation agents to fifth-generation agents, the cephalosporins show increased activity against gram-negative organisms, increased resistance to destruction by beta-lactamases, and increased ability to reach the CSF. Cost is not a definitive factor. First-generation drugs have less penetration of the CSF. Resistance to destruction by beta-lactamases increases with increasing generations. REF: pp. 1025-1026

8. A patient who has been taking gentamicin for 5 days reports a headache and dizziness. What will the nurse do? a. Request an order for a gentamicin peak level. b. Suspect ototoxicity and notify the prescriber. c. Tell the patient to ask for help with ambulation. d. Tell the patient to report any tinnitus.

ANS: B Headache and dizziness are signs of ototoxicity, and the prescriber should be notified. A peak level is not indicated; it is more important to know the trough level. Telling the patient to ask for help with ambulation and to report tinnitus should both be done but neither one is the priority nursing action. REF: pp. 1052-1053

12. A patient who has been taking a glucocorticoid for several months arrives in the clinic. The nurse notes that the patient's cheeks appear full and that a prominent hump of fat is present on the upper back. The nurse will ask the provider to order which test(s)? a. Liver function tests b. Serum electrolytes c. Tuberculin skin test d. Vitamin D levels

ANS: B This patient shows signs of iatrogenic Cushing's syndrome, which may include serum electrolyte disturbances; therefore, the electrolyte levels should be monitored. Liver function tests, tuberculin skin testing, and vitamin D levels are not indicated. REF: p. 873

What are the properties of an ideal drug? (Select all that apply.) A. Irreversible action B. Predictability C. Ease of administration D. Chemical stability E. A recognizable trade name

ANS: B, C, D In addition to predictability, ease of administration, and chemical stability, other properties include a reversible action so that any harm the drug may cause can be undone and a simple generic name, because generic names are usually complex and difficult to remember and pronounce. page 1-2

A nurse is reviewing a patient's medical record before administering a medication. Which factors can alter the patient's physiologic response to the drug? (Select all that apply.) a . Ability to swallow pills b . Age c . Genetic factors d . Gender e . Height

ANS: B, C, D Age, genetic factors, and gender influence an individual patient's ability to absorb, metabolize, and excrete drugs; therefore, these factors must be assessed before a medication is administered. A patient's ability to swallow pills, although it may determine the way a drug is administered, does not affect the physiologic response. Height does not affect response; weight and the distribution of adipose tissue can affect the distribution of drugs. page 6-7

When administering medications to infants, it is important to remember which of the following? (Select all that apply.) a. Breast-feeding infants are more likely to develop toxicity when given lipid-soluble drugs. b. Immaturity of renal function in infancy causes infants to excrete drugs less efficiently. c. Infants have immature livers, which slows drug metabolism. d. Infants are more sensitive to medications that act on the central nervous system(CNS). e. Oral medications are contraindicated in infants, because PO administration requires a cooperative patient.

ANS: B, C, D Immature renal function causes infants to excrete drugs more slowly, and infants are at risk for toxicity until renal function is well developed. Infants' livers are not completely developed, and they are less able to metabolize drugs efficiently. Because the blood-brain barrier is not well developed in infants, caution must be used when administering CNS drugs. Lipid-soluble drugs may be excreted in breast milk if the mother is taking them, but breast-feeding does not affect medications given directly to the infant. Oral medications may be given safely to infants as long as they are awake and can swallow the drug. page 32-33

Two nurses are discussing theories of drug-receptor interaction. Which statements are true regarding the affinity of a drug and its receptor? (Select all that apply.) a. Affinity and intrinsic activity are dependent properties. b. Affinity refers to the strength of the attraction between a drug and its receptor. c. Drugs with high affinity are strongly attracted to their receptors. d. Drugs with low affinity are strongly attracted to their receptors. e. The affinity of a drug for its receptors is reflected in its potency.

ANS: B, C, E Affinity refers to the strength of the attraction between a drug and its receptor. Drugs with high affinity are strongly attracted to their receptors, and the affinity of a drug and its receptors is reflected in its potency. Affinity and intrinsic activity are independent properties. Drugs with low affinity are weakly attracted to their receptors. page 48-49

2. Which side effects of opioid analgesics can have therapeutic benefits? (Select all that apply.) a. Biliary colic b. Cough suppression c. Suppression of bowel motility d. Urinary retention e. Vasodilation

ANS: B, C, E Individual effects of morphine may be beneficial, detrimental, or both. Cough suppression is usually beneficial; suppression of bowel motility and vasodilation can be either beneficial or detrimental. Biliary colic and urinary retention are always detrimental side effects. REF: pp. 262-262

Which actions occur in 90% of fatal medication errors? (Select all that apply.) a. Confusing drugs with similar packaging b. Giving a drug intravenously instead of intramuscularly c. Giving Nasarel instead of Nizoral d. Using an infusion device that malfunctions e. Writing a prescription illegibly

ANS: B, C, E Ninety percent of fatal medication errors fall into three categories: human factors, communication mistakes, and name confusion. Giving a drug IV (intravenously) instead of IM (intramuscularly) is an example of a human factor; writing a prescription so that it is illegible is an example of a communication mistake; and giving a drug with a name that sounds like the name of another drug is an example of name confusion. Confusion of drugs with similar packaging and using a faulty device also can cause fatal drug errors, but these factors do not fall into the categories that account for 90% of fatal errors. page 68

A patient tells the nurse that an analgesic he will begin taking may cause drowsiness and will decrease pain up to 4 hours at a time. Based on this understanding of the drug's effects by the patient, the nurse will anticipate which outcome? A. Decreased chance of having a placebo effect B. Decreased motivation to take the drug C. Improved compliance with the drug regimen D. Increased likelihood of drug overdose

ANS: C A drug is effective if it produces the intended effects, even if it also produces side effects. Patients who understand both the risks and benefits of taking a medication are more likely to comply with the drug regimen. page 2

What is a desired outcome when a drug is described as easy to administer? A. It can be stored indefinitely without need for refrigeration. B. It does not interact significantly with other medications. C. It enhances patient adherence to the drug regimen. D. It is usually relatively inexpensive to produce.

ANS: C A major benefit of drugs that are easy to administer is that patients taking them are more likely to comply with the drug regimen. Drugs that are easy to give may have the other attributes listed, but those properties are independent of ease of administration. page 2

1. The nurse is working on a plan of care with her patient which includes turning and positioning and adequate nutrition to help the patient maintain intact skin integrity. The nurse helps the patient to realize that this breaks the chain of infection by eliminating which element? a. Host b. Mode of transmission c. Portal of entry d. Reservoir

ANS: C Broken or impaired skin creates a portal of entry for pathogens. By maintaining intact tissue, the patient and the nurse have broken the chain of infection by eliminating a portal of entry. Host is incorrect because you are not eliminating the person or organism. Intact tissue does not eliminate the mode of transmission. Skin can still be used to transfer pathogens regardless of it being intact or broken. Intact skin does not eliminate the location for pathogens to live and grow. Chapter 24 Giddens

5. The nurse reviews the patient's complete blood count (CBC) results and notes that the neutrophil levels are elevated, but monocytes are still within normal limits. This indicates what type of inflammatory response? a. Chronic b. Resolved c. Early stage acute d. Late stage acute

ANS: C Elevated neutrophils and monocytes within normal limits are findings indicative of early inflammatory response. Neutrophils increase in just a few hours, while it takes the body days to increase the monocyte levels. Chronic inflammation results in varying elevations in WBCs dependent on multiple issues. Elevated neutrophils are not indicative of resolved inflammation. Elevations in monocytes occur later in the inflammatory response. Chapter 23 Giddens

1. A nurse is instructing her patient with ulcerative colitis regarding the need to avoid enteric coated medications. The nurse knows that the patient understands the reason for this teaching when he states which of the following? a. "The coating on these medications is irritating to my intestines." b. "I need a more immediate response from my medications than can be obtained from enteric coated medications." c. "Enteric coated medications are absorbed lower in the digestive tract and can be irritating to my intestines or inadequately absorbed by my inflamed tissue." d. "I don't need to use these medications because they cause diarrhea, and I have had enough trouble with diarrhea and rectal bleeding over the past weeks."

ANS: C Enteric coatings on medications are designed to prevent breakdown and absorption of the medication until lower in the digestive tract, usually to prevent stomach irritation or to reach a certain point in the digestive tract for optimal absorption. For the patient with ulcerative colitis, the intestinal lining is inflamed or susceptible to inflammation and can have impaired absorption; therefore, enteric coated medications should be avoided. The coating is not irritating, but the medication can be. The response time of the medication is not a concern in this instance. Enteric coated medicines do not cause diarrhea simply because they are enteric coated. Chapter 23 Giddens

4. While caring for a patient preparing for a kidney transplant, the nurse knows that the patient understands teaching on immunosuppression when she makes which statement? a. "My body will treat the new kidney like my original kidney." b. "I will have to make sure that I avoid being around people." c. "The medications that I take will help prevent my body from attacking my new kidney." d. "My body will only have a problem with my new kidney if the donor is not directly related to me."

ANS: C Immunosuppressant therapy is initiated to inhibit optimal immune response. This is necessary in the case of transplantation, because the normal immune response would cause the body to recognize the new tissue as foreign and attack it. The body will identify the new kidney as foreign and will not treat it as the original kidney. While patients with transplants must be careful about exposure to others, especially those who are or might be ill, and practice adequate and consistent infection control techniques, they don't have to avoid people or social interaction. The new kidney brings foreign cells regardless of relationship between donor and recipient. Chapter 22 Giddens

6. The nurse is preparing to administer medications to a patient with rheumatoid arthritis (RA). The nurse should explain which goal of treatment to the patient? a. Eradicate the disease b. Enhance immune response c. Control inflammation d. Manage pain

ANS: C Medications for RA are intended to control the inflammation that results from the body's hyperimmune response. Autoimmune diseases like RA are chronic and currently have no curative treatments. Autoimmune diseases like RA are caused by hyperimmune response. The immune system needs to be suppressed, not enhanced. While the medications used for RA might help with pain management, the goal of medication intervention is to manage the inflammation. Chapter 22 Giddens

2. Two nurses are discussing health care quality. They agree which event contributes to increased health care quality? a. Magnet status hospitals b. Fewer adverse events c. Collaboration of multiple health care agencies d. Increased patient education

ANS: C Multiple health care agencies are able to collaborate and provide better outcomes for health care personnel and patients. Magnet status hospitals may be good, but the status does not always mean a quality outcome. Fewer adverse events and increased patient education are good, but they may be the result of other variables and not just quality of care rendered Giddens Health Care Quality

4. An elderly Chinese woman is interested in biologically based therapies to relieve osteoarthritis (OA) pain. You are preparing a plan of care for her OA. Options most conducive to her expressed wishes may include which actions or activities? a. Pilates, breathing exercises, and aloe vera b. Guided imagery, relaxation breathing, and meditation c. Herbs, vitamins, and tai chi d. Alternating ice and heat to relieve pain and inflammation

ANS: C Nonpharmacologic strategies encompass a wide variety of nondrug treatments that may contribute to comfort and pain relief. These include the body-based (physical) modalities, such as massage, acupuncture, and application of heat and cold, and the mind-body methods, such as guided imagery, relaxation breathing, and meditation. There are also biologically based therapies which involve the use of herbs and vitamins, and energy therapies such as reiki and tai chi. Pilates, breathing exercises, aloe vera, guided imagery, relaxation breathing, meditation, and alternating ice and heat are multimodal therapies for pain management. They are not exclusively biologically based, which involves the use of herbs and vitamins. Chapter 28 Giddens

4. A nurse is teaching a group of business people about disease transmission. He knows that he needs to reeducate when one of the participants states which of the following? a. "When traveling outside of the country, I need to be sure that I receive appropriate vaccinations." b. "Food and water supplies in foreign countries can contain microorganisms to which my body is not accustomed and has no resistance." c. "If I don't feel sick, then I don't have to worry about transmitted diseases." d. "I need to be sure to have good hygiene practices when traveling in crowded planes and trains."

ANS: C People can transmit pathogens even if they don't currently feel ill. Some carriers never experience the full symptoms of a pathogen. Travelers may need different vaccinations when traveling to countries outside their own because of variations in prevalent microorganisms. Food and water supplies in foreign countries can contain microorganisms that will affect a body unaccustomed to their presence. Adequate hygiene is essential when in crowded, public spaces like planes and other forms of public transportation. Chapter 24 Giddens

6. During orientation to an emergency department, the nurse educator would be concerned if the new nurse listed which of the following as a risk factor for impaired thermoregulation? a. Impaired cognition b. Occupational exposure c. Physical agility d. Temperature extremes

ANS: C Physical agility is not a risk factor for impaired thermoregulation. The nurse educator would use this information to plan additional teaching to include medical conditions and gait disturbance as risk factors for hypothermia, because their bodies have a reduced ability to generate heat. Impaired cognition is a risk factor. Recreational or occupational exposure is a risk factor. Temperature extremes are risk factors for impaired thermoregulation. Chapter 10 Giddens

5. A student nurse and clinical instructor are discussing quality in health care. The instructor knows the student understands when the student makes which statement? a. "Quality is apparent in all health care." b. "Quality is an outcome of health care." c. "Quality is seen and unseen in health care." d. "Quality is achieved by collaboration in health care."

ANS: C Quality in health care is tangible and intangible. Quality in health care is not apparent in all health care, as many areas of health care are lacking. Quality of care does not always affect the outcome of care; the patient may recover no matter what care is given. Quality is not always achieved by collaboration. Giddens Health Care Quality

5. Which is an essential element of a standard order set to verify a medication order? a. Volume only b. Number of tablets c. Metric dose/strength d. Hour of administration

ANS: C The ISMP recommendations for standardized medication order sets include such elements as the drug name (generic followed by brand when appropriate), metric dose/strength, frequency and duration, route, and indication. Although a prescription may include volume or number of tablets, the essential component is dose or strength, because the volume or number of tablets may vary by manufacturer. The exact hour of administration can be based on factors such as the frequency, agency protocols, and patient preferences. Giddens Safety

An adult male patient is 1 day postoperative from a total hip replacement. On a pain scale of 0 to 10, with 10 being the greatest pain, the patient reports a pain level of 10. Which medication would be most appropriate for the nurse to administer to this patient? a. 60 mg morphine sulfate PO b. 75 mg meperidine [Demerol] intramuscularly c. 6 mg morphine sulfate intravenously d. Fentanyl [Duragesic] patch 50 mcg transdermally

ANS: C The intravenous route is the fastest route of absorption and the one most appropriate for a patient in extreme pain. With the oral route, the medication would take at least 45 minutes to be effective, too long for a patient in extreme pain. With the intramuscular route, the medication would take at least 15 minutes to be effective; although faster than the oral route, this is not as fast as the intravenous route. A Duragesic patch would be the most inappropriate route because of the long drug half-life. This is a more appropriate route for long-term use. page 31-32

3. The nurse admitting a patient to the emergency department on a very hot summer day would suspect hyperthermia when the patient demonstrates which assessment finding? a. Decreased respirations b. Low pulse rate c. Red, sweaty skin d. Slow capillary refill

ANS: C With hyperthermia, vasodilatation occurs causing the skin to appear flushed and warm or hot to touch. There is an increased respiration rate with hyperthermia. The heart rate increases with hyperthermia. With hypothermia there is slow capillary refill. Chapter 10 Giddens

A patient receives a drug that has a narrow therapeutic range. The nurse administering this medication will expect to do what? a. Administer the drug at intervals longer than the drug half-life. b. Administer this medication intravenously. c. Monitor plasma drug levels. d. Teach the patient that maximum drug effects will occur within a short period.

ANS: C A drug with a narrow therapeutic range is more difficult to administer safely, because the difference between the minimum effective concentration and the toxic concentration is small. Patients taking these medications must have their plasma drug levels monitored closely to ensure that they are getting an effective dose that is not toxic. Administering medications at longer intervals only increases the time required to reach effective plasma drug levels. Drugs that have a narrow therapeutic range may be given by any route and do not differ from other medications in the amount of time it takes for them to take effect, which is a function of a drug's half-life and dosing frequency. page 39-40

A patient has been receiving an antibiotic with a small therapeutic index for 10 days. Upon assessment, the nurse notes an increase in the drug's side effects. What would be the nurse's priority action? a. Call the prescriber and have the antibiotic changed. b. Suspect an allergic reaction and administer a PRN antihistamine. c. Ask the prescriber to order a plasma drug level test d. Set up oxygen and obtain an order for an antagonist

ANS: C A narrow therapeutic index indicates that a drug is relatively unsafe and should be monitored closely. The nurse should have a blood level drawn to confirm suspicions of toxicity. The nurse would not have the antibiotic changed, because there is no cause at this time. The patient is unlikely to be experiencing an allergic reaction, because the antibiotic has been in the system for 10 days. The patient shows no signs of anaphylaxis, so oxygen and an antagonist are not indicated. page 53

A young adult postoperative patient is receiving morphine 2 to 4 mg IV every 2 hours PRN pain. The last dose was 3 mg given 2 hours ago. The patient is asleep, and the nurse notes a heart rate of 86 beats per minute and a respiratory rate of 8 breaths per minute. Which PRN medication will the nurse give this patient? a. Diphenhydramine [Benadryl] to counter morphine side effects b. Morphine 4 mg for increased pain, as indicated by tachycardia c. Naloxone [Narcan] to block the effects of the morphine d. Nothing at this time, because the patient is resting comfortably

ANS: C A respiratory rate of 8 breaths per minute indicates respiratory depression, which is a significant adverse effect indicating morphine toxicity. Naloxone blocks the actions of morphine at cell receptor sites and is given to quickly reverse the effects. This patient does not have signs of an allergic response, which would include shortness of breath, a rapid respiratory rate, and wheezing. The tachycardia might be a sign of worsening pain, but the toxic effects must be treated first. Patients who are sleeping are not always pain free. page 58

7. A patient who is taking doxycycline for a serious infection contacts the nurse to report anal itching. The nurse will contact the provider to discuss: a. adding an antihistamine to the patient's drug regimen. b. ordering liver function tests to test for hepatotoxicity. c. prescribing an antifungal drug to treat a superinfection. d. testing the patient for a C. difficile secondary infection

ANS: C A superinfection occurs secondary to suppression of drug-sensitive organisms. Overgrowth with fungi, especially Candida albicans, is common and may occur in the mouth, pharynx, vagina, and bowel. Anal itching is a sign of such an infection, not a sign of hepatotoxicity. Antihistamines will not treat the cause. C. difficile is characterized by profuse, watery diarrhea. REF: p. 1039

6. A patient is to begin taking doxycycline to treat a rickettsial infection. Which statement by the patient indicates a need for teaching about this drug? a. "I should consult my provider before using laxatives or antacids while taking this drug." b. "I should not take a calcium supplement or consume dairy products with this drug." c. "I should take this drug with food to ensure more complete absorption." d. "If I get diarrhea, I should stop taking the drug and let my provider know immediately."

ANS: C Absorption of tetracyclines is reduced in the presence of food. The tetracyclines form insoluble chelates with calcium, iron, magnesium, aluminum, and zinc, so patients should not take tetracyclines with dairy products, calcium supplements, or drugs containing these minerals. Patients who experience diarrhea should stop taking the drug and notify the provider so they can be tested for C. difficile infection. REF: p. 1038

7. A patient is about to receive penicillin G for an infection that is highly sensitive to this drug. While obtaining the patient's medication history, the nurse learns that the patient experienced a rash when given amoxicillin [Amoxil] as a child 20 years earlier. What will the nurse do? a. Ask the provider to order a cephalosporin. b. Reassure the patient that allergic responses diminish over time. c. Request an order for a skin test to assess the current risk. d. Suggest using a desensitization schedule to administer the drug.

ANS: C Allergy to penicillin can decrease over time; therefore, in patients with a previous allergic reaction who need to take penicillin, skin tests can be performed to assess the current risk. Until this risk is known, changing to a cephalosporin is not necessary. Reassuring the patient that allergic responses will diminish is not correct, because this is not always the case; the occurrence of a reaction must be confirmed with skin tests. Desensitizing schedules are used when patients are known to be allergic and the drug is required anyway. REF: p. 1020

6. A patient is receiving intravenous potassium penicillin G, 2 million units to be administered over 1 hour. At 1900, the nurse notes that the dose hung at 1830 has infused completely. What will the nurse do? a. Assess the skin at the infusion site for signs of tissue necrosis. b. Observe the patient closely for confusion and other neurotoxic effects. c. Request an order for serum electrolytes and cardiac monitoring. d. Watch the patient's actions and report any bizarre behaviors.

ANS: C Although penicillin G is the least toxic of all antibiotics, certain adverse effects may be caused by compounds coadministered with penicillin. When large doses of potassium penicillin G are administered rapidly, hyperkalemia can occur, which can cause fatal dysrhythmias. When penicillin G is administered IM, tissue necrosis occurs with inadvertent intra-arterial injection. Confusion, seizures, and hallucinations can occur if blood levels of the drug are too high. Bizarre behaviors result with large IV doses of procaine penicillin G. REF: p. 1020

3. A nurse is reviewing the culture results of a patient receiving an aminoglycoside. The report reveals an anaerobic organism as the cause of infection. What will the nurse do? a. Contact the provider to discuss an increased risk of aminoglycoside toxicity. b. Continue giving the aminoglycoside as ordered. c. Request an order for a different class of antibiotic. d. Suggest adding a penicillin to the patient's drug regimen.

ANS: C Aminoglycosides are not effective against anaerobic microbes, so another class of antibiotics is indicated. There is no associated increase in aminoglycoside toxicity with anaerobic infection. The aminoglycoside will not be effective, so continuing to administer this drug is not indicated. Adding another antibiotic is not useful, because the aminoglycoside is not necessary. REF: p. 1050

A patient is taking drug X and receives a new prescription for drug Y, which is listed as an inducing agent. The nurse caring for this patient understands that this patient may require ____ doses of drug ____. a. lower; X b. lower; Y c. higher; X d. higher; Y

ANS: C An inducing agent stimulates the synthesis of CYP isoenzymes, which may increase the metabolism of other drugs as much as two- to threefold, thereby lowering the level of those drugs in the body and requiring higher doses to maintain drug effectiveness. page 56-58

3. The nurse is teaching a nursing student about the mechanism by which antimicrobial agents achieve selective toxicity. Which statement by the student indicates a need for further teaching? a. "Some agents disrupt the bacterial cell wall." b. "Some agents act to block the conversion of para-aminobenzoic acid (PABA) to folic acid." c. "Some agents cause phagocytosis of bacterial cells." d. "Some agents weaken the cell wall, causing cell wall lysis."

ANS: C Antimicrobial agents do not cause phagocytosis of bacterial cells. They do disrupt and weaken the bacterial cell wall; because human cells do not have cell walls, antimicrobial agents are not toxic to human cells. Some agents block the conversion of PABA to folic acid; humans do not synthesize folic acid and are not harmed by this process. REF: pp. 1001-1002

4. A patient tells the nurse that she takes aspirin for menstrual cramps, but she does not feel that it works well. What will the nurse suggest? a. The patient should avoid any type of COX inhibitor because of the risk of Reye's syndrome. b. The patient should increase the dose to a level that suppresses inflammation. c. The patient should use a first-generation nonsteroidal anti-inflammatory medication instead. d. The patient should use acetaminophen because of its selective effects on uterine smooth muscle.

ANS: C Aspirin (ASA) has analgesic effects for joint pain, muscle pain, and headache, but it is relatively ineffective against visceral pain, including uterine smooth muscle pain, for which NSAIDs are indicated. The risk of Reye's syndrome is associated with the use of ASA in children to treat fever. Increasing the ASA dose to anti-inflammatory levels is useful for rheumatic fever, tendonitis, and bursitis. Acetaminophen is not effective for dysmenorrhea. REF: p. 850 / pp. 851-852

2. A patient who takes daily doses of aspirin is scheduled for surgery next week. The nurse should advise the patient to: a. continue to use aspirin as scheduled. b. reduce the aspirin dosage by half until after surgery. c. stop using aspirin immediately. d. stop using aspirin 3 days before surgery.

ANS: C Aspirin must be withdrawn at least 1 week before surgery. Aspirin cannot be continued as scheduled, because the risk for bleeding is too great. An interval of 3 days is not long enough for the bleeding effects of aspirin to be reversed. Cutting the dose in half would not reduce the effects of bleeding associated with aspirin use. REF: p. 853

3. A clinic patient who has been taking a glucocorticoid for arthritis for several months remarks to the nurse, "It's a good thing my symptoms are better, because my mother has been quite ill, and I have to take care of her." The patient's blood pressure is 100/60 mm Hg. The nurse will report this to the provider and ask about: a. reducing the patient's dose. b. using every other day dosing. c. increasing the patient's dose. d. tapering the dose.

ANS: C Because of their adrenal suppression, patients taking glucocorticoids long term require increased doses at times of stress and even for a time after stopping the drug until adrenal function returns. This patient's lower blood pressure is an indication that glucocorticoid levels may be depleted. Reducing the dose would only exacerbate the patient's problems. Every other day dosing is used early in glucocorticoid therapy to reduce adrenal suppression, but it would not be useful now. Tapering of doses is used to allow adrenal function to recover as the drug is discontinued. REF: p. 874

A nursing student is preparing to give a medication that has a boxed warning. The student asks the nurse what this means. What will the nurse explain about boxed warnings? a. They indicate that a drug should not be given except in life-threatening circumstances. b. They provide detailed information about the adverse effects of the drug. c. They alert prescribers to measures to mitigate potential harm from side effects. d. They provide information about antidotes in the event that toxicity occurs.

ANS: C Boxed warnings (also known as black box warnings) are used to alert providers to potential side effects and to ways to prevent or reduce harm from these side effects. A boxed warning is placed on any drug that, although useful, has serious side effects; this is a way to keep drugs on the market while protecting patients. Many of these drugs are used in situations that are not life threatening. The boxed warning provides a concise summary and not a detailed explanation of drug side effects. The boxed warning does not include antidotes to toxicity. page 66

3. A patient recently began receiving clindamycin [Cleocin] to treat an infection. After 8 days of treatment, the patient reports having 10 to 15 watery stools per day. What will the nurse tell this patient? a. The provider may increase the clindamycin dose to treat this infection. b. This is a known side effect of clindamycin, and the patient should consume extra fluids. c. The patient should stop taking the clindamycin now and contact the provider immediately. d. T he patient should try taking Lomotil or a bulk laxative to minimize the diarrheal symptoms.

ANS: C Clostridium difficile-associated diarrhea (CDAD) is the most severe toxicity of clindamycin; if severe diarrhea occurs, the patient should be told to stop taking clindamycin immediately and to contact the provider so that treatment with vancomycin or metronidazole can be initiated. Increasing the dose of clindamycin will not treat this infection. Consuming extra fluids while still taking the clindamycin is not correct, because CDAD can be fatal if not treated. Taking Lomotil or bulk laxatives only slows the transit of the stools and does not treat the cause. REF: pp. 1042-1043

A patient is being discharged after surgery. During the admission history, the nurse learned that the patient normally consumes two or three glasses of wine each day. The prescriber has ordered hydrocodone with acetaminophen [Lortab] for pain. What will the nurse do? a. Request an order for acetaminophen without hydrocodone for pain. b. Suggest that the patient use ibuprofen for pain. c. Tell the patient not to drink wine while taking Lortab. d. Tell the patient to limit wine intake to one or two glasses per day.

ANS: C Combining a hepatotoxic drug with certain other drugs may increase the risk of hepatotoxicity. When even therapeutic doses of acetaminophen are taken with alcohol, the acetaminophen can cause liver damage. Patients should be cautioned not to drink alcohol; even two drinks with acetaminophen can produce this effect. Hydrocodone does not contribute to hepatotoxicity. Ibuprofen is not indicated for postoperative pain unless the pain is mild. Limiting wine to one or two glasses per day still increases the risk of hepatotoxicity. page 63-64

4. A child has received amoxicillin [Amoxil] for three previous ear infections, but a current otitis media episode is not responding to treatment. The nurse caring for this child suspects that resistance to the bacterial agent has occurred by which microbial mechanism? a. Alteration of drug target molecules b. Antagonist production c. Drug inactivation d. Reduction of drug concentration at the site of action

ANS: C Drug inactivation can occur when microbes produce drug-metabolizing enzymes. Penicillin-resistant organisms, including many that cause otitis media, produce penicillinase. Alteration of drug target molecules, drug inactivation, and reduction of the drug concentration occur with other antimicrobials. REF: pp. 1003-1005

A nurse administers the same medication in the same preparation in the same dose to several patients and notes that some patients have a better response to the drug than others. What is the most likely explanation for this phenomenon? a. Altered bioavailability of the drug b. Patient compliance with the therapeutic regimen c. Pharmacogenomic differences among individuals d. Placebo effects enhancing expectations of drug efficacy

ANS: C Each patient's genetic makeup can determine how that patient responds to drugs quantitatively and qualitatively, and this is the most likely cause of individual variation when the same drug is given at the same dose. The bioavailability of a drug is determined by the drug's composition and varies across formulations of the drug. The patients in this example were given the same drug. The nurse was administering the medication to the patients, so compliance is not an issue. Nothing in this example indicates that a placebo effect was in play. page 75-76

A patient tells the nurse that the oral drug that has been prescribed has caused a lot of stomach discomfort in the past. What will the nurse ask the prescriber? a. Whether a sublingual form of the medication can be given b. Whether the medication can be given by a parenteral route instead c. Whether an enteric-coated form of the drug is available d. Whether the patient can receive a sustained-release preparation of the drug

ANS: C Enteric-coated drugs are preparations that have been coated with a material that dissolves in the intestines, not the stomach. This coating is used either to protect the drug from stomach acid and pepsin or to protect the stomach from a drug that can cause gastric upset. Sublingual forms often are used for drugs that undergo rapid inactivation during the first pass through the hepatic circulation so that the drug can be absorbed directly into the systemic circulation. Parenteral routes are more costly and less safe than oral administration and should not be used unless necessary. A sustained-release preparation is used to release the drug into the body over a specific period to reduce the number of daily doses required to sustain therapeutic drug levels. page 31

A patient claims to get better effects with a tablet of Brand X of a drug than with a tablet of Brand Y of the same drug. Both brands contain the same amount of the active ingredient. What does the nurse know to be most likely? a. Advertising by pharmaceutical companies can enhance patient expectations of one brand over another, leading to a placebo effect. b. Because the drug preparations are chemically equivalent, the effects of the two brands must be identical. c. Tablets can differ in composition and can have differing rates of disintegration and dissolution, which can alter the drug's effects in the body. d. The bioavailability of a drug is determined by the amount of the drug in each dose.

ANS: C Even if two brands of a drug are chemically equivalent (i.e., they have identical amounts of the same chemical compound), they can have different effects in the body if they differ in bioavailability. Tablets made by different manufacturers contain different binders and fillers, which disintegrate and dissolve at different rates and affect the bioavailability of the drug. Two brands may be chemically equivalent and still differ in bioavailability, which is not determined by the amount of drug in the dose. page 31-32

A patient is receiving intravenous gentamicin. A serum drug test reveals toxic levels. The dosing is correct, and this medication has been tolerated by this patient in the past. Which could be a probable cause of the test result? a. A loading dose was not given. b. The drug was not completely dissolved in the IV solution. c. The patient is taking another medication that binds to serum albumin. d. The medication is being given at a frequency that is longer than its half-life.

ANS: C Gentamicin binds to albumin, but only weakly, and in the presence of another drug that binds to albumin, it can rise to toxic levels in blood serum. A loading dose increases the initial amount of a drug and is used to bring drug levels to the desired plateau more quickly. A drug that is not completely dissolved carries a risk of causing embolism. A drug given at a frequency longer than the drug half-life will likely be at subtherapeutic levels and not at toxic levels. page 34

A nurse is explaining drug metabolism to a nursing student who asks about glucuronidation. The nurse knows that this is a process that allows drugs to be: a. excreted in hydrolyzed form in the feces to reduce drug toxicity. b. reabsorbed from the urine into the renal circulation to minimize drug loss. c. recycled via the enterohepatic recirculation to remain in the body longer. d. transported across the renal tubules to be excreted in the urine.

ANS: C Glucuronidation of some drugs in the liver allows drugs to enter the bile, pass into the duodenum, and then be hydrolyzed to release the free drug. This is a repeating cycle of enterohepatic recirculation, which allows drugs to remain in the body longer. Glucuronidated drugs that are more resistant to hydrolysis are excreted in the feces. Glucuronidation occurs in the enterohepatic circulation and not in the renal circulation. page 37

16. A patient who has been taking linezolid [Zyvox] for 6 months develops vision problems. What will the nurse do? a. Reassure the patient that this is a harmless side effect of this drug. b. Tell the patient that blindness is likely to occur with this drug. c. Tell the patient that this symptom is reversible when the drug is discontinued. d. Tell the patient to take tyramine supplements to minimize this effect.

ANS: C Linezolid is associated with neuropathy, including optic neuropathy. This is a reversible effect that will stop when the drug is withdrawn. Reassuring the patient that this is a harmless side effect is not correct. It is not an indication that blindness will occur. Tyramine supplements are not indicated. REF: p. 1044

The nurse administers naloxone [Narcan] to a patient who has received a toxic dose of morphine sulfate. The nurse understands that the naloxone is effective because of which action? a. Countering the effects of morphine sulfate by agonist actions b. Increasing the excretion of morphine sulfate by altering serum pH c. Preventing activation of opioid receptors through antagonist actions d. Regulating the sensitivity of opioid receptors by neurochemical alterations

ANS: C Naloxone acts by blocking the action of opioids at opioid receptors. An opioid agonist would increase the effects of morphine. Naloxone does not affect serum pH or excretion of opioids. Naloxone does not alter the sensitivity of opioid receptors. page 48-49

10. A nurse is teaching a nursing student who wants to know how aspirin and nonaspirin first-generation NSAIDs differ. Which statement by the student indicates a need for further teaching? a. "Unlike aspirin, first-generation NSAIDs cause reversible inhibition of cyclooxygenase." b. "NSAIDs do not increase the risk of myocardial infarction and stroke; however, unlike ASA, they do not provide protective benefits against those conditions." c. "Unlike aspirin, first-generation NSAIDs do not carry a risk of hypersensitivity reactions." d. "Unlike aspirin, first-generation NSAIDs cause little or no suppression of platelet aggregation."

ANS: C Nonaspirin first-generation NSAIDs carry a risk of hypersensitivity reactions similar to the risk posed by ASA. These agents cause reversible COX inhibition, whereas ASA causes irreversible COX inhibition. NSAIDs do not provide protective benefits for myocardial infarction or stroke, as does ASA. Nonaspirin first-generation NSAIDs do cause suppression of platelet aggregation, but the suppression is reversible. REF: p. 850

6. A patient who has biliary colic reports a pain level of 8 on a 1 to 10 pain scale with 10 being the most severe pain. The patient has an order for ibuprofen as needed for pain. Which action by the nurse is correct? a. Administer the ibuprofen as ordered. b. Contact the provider to discuss nonpharmacologic pain measures. c. Request an order for meperidine [Demerol]. d. Request an order for morphine sulfate.

ANS: C Opioids can induce spasm of the common bile duct and can cause biliary colic. For patients with existing biliary colic, morphine may intensify the pain. It is important to treat pain, however, and certain opioids, such as meperidine, which cause less smooth muscle spasm, may be given. Ibuprofen is used for mild to moderate pain and is not appropriate for this patient. Nonpharmacologic methods are appropriate when used as adjunctive therapy with an opioid. p. 263

11. A patient with an infection caused by Pseudomonas aeruginosa is being treated with piperacillin. The nurse providing care reviews the patient's laboratory reports and notes that the patient's blood urea nitrogen and serum creatinine levels are elevated. The nurse will contact the provider to discuss: a. adding an aminoglycoside. b. changing to penicillin G. c. reducing the dose of piperacillin. d. ordering nafcillin.

ANS: C Patients with renal impairment should receive lower doses of piperacillin than patients with normal renal function. Aminoglycosides are nephrotoxic. Penicillin G and nafcillin are not effective against Pseudomonas infections. REF: pp. 1022-1023

9. A patient has an infection caused by Streptococcus pyogenes. The prescriber has ordered dicloxacillin PO. What will the nurse do? a. Administer the medication as ordered. b. Contact the provider to suggest giving the drug IV. c. Question the need for a penicillinase-resistant penicillin. d. Suggest ordering vancomycin to treat this infection.

ANS: C Penicillinase-resistant penicillins have been developed for use against penicillinase-producing strains of staphylococci. These drugs have a very narrow antimicrobial spectrum and should be used only for such infections. S. pyogenes can be treated with penicillin G. The nurse should question the order. It is incorrect to contact the provider to ask for IV dosing. This infection can be treated with penicillin G and not with vancomycin. REF: pp. 1021-1022

A patient has been taking narcotic analgesics for chronic pain for several months. The nurse caring for this patient notes that the prescribed dose is higher than the recommended dose. The patient has normal vital signs, is awake and alert, and reports mild pain. What does the nurse recognize about this patient? a. This patient exhibits a negative placebo effect with a reduced response to the drug. b. This patient has developed a reaction known as tachyphylaxis because of repeated exposure to the drug. c. This patient has developed pharmacodynamic tolerance, which has increased the minimal effective concentration (MEC) needed for analgesic effect. d. This patient produces higher than normal hepatic enzymes as a result of prolonged exposure to the drug.

ANS: C Pharmacodynamic tolerance results when a patient takes a drug over a period of time. Adaptive processes occur in response to chronic receptor occupation. The result is that the body requires increased drug, or an increased MEC, to achieve the same effect. This patient is getting adequate pain relief, so there is no negative placebo effect. Tachyphylaxis is a form of tolerance that can be defined as a reduction in drug responsiveness brought on by repeated dosing over a short time; this occurs over several months. Barbiturates induce synthesis of hepatic enzymes that cause increased metabolism of the drug, but it does not increase the MEC. page 74-75

5. A nursing student asks a nurse to clarify the differences between the mechanisms of spontaneous mutation and conjugation in acquired resistance of microbes. What will the nurse say? a. Conjugation results in a gradual increase in resistance. b. Conjugation results in random changes in the microbe's DNA. c. Spontaneous mutation leads to resistance to only one antimicrobial agent. d. Spontaneous mutation can transfer DNA from one organism to another.

ANS: C Spontaneous mutation generally confers resistance to only one drug. Conjugation can occur quickly; spontaneous mutation is gradual. Spontaneous mutation is random; conjugation is not. Conjugation can occur with the transfer of DNA from one organism to another. REF: pp. 1004-1005

5. To prevent yellow or brown discoloration of teeth in children, tetracyclines should not be given: a. to children once the permanent teeth have developed. b. to patients taking calcium supplements. c. to pregnant patients after the fourth month of gestation. d. with dairy products or antacids.

ANS: C Tetracyclines bind to calcium in developing teeth, resulting in yellow or brown discoloration. They should not be given to pregnant women after the fourth month of gestation, because they will cause staining of deciduous teeth in the fetus. In children, discoloration occurs when tetracyclines are given between the ages of 4 and 8 years, because this is when permanent teeth are developing. Tetracycline binds with calcium, so absorption is diminished when the drug is given with calcium supplements, dairy products, or calcium-containing antacids; however, this does not affect tooth development. REF: p. 1038 | p. 1040

7. A recent campaign, initiated by the Centers for Disease Control (CDC), to delay the emergence of antibiotic resistance in hospitals, has what as one of its objectives? a. Allowing patients to stop antibiotics when symptoms subside b. Allowing prescribers to develop their own prescribing guidelines c. Increased adherence to prescribed antibiotics d. Increased use of antibiotics among parents of young children

ANS: C The CDC initiative has identified three objectives to help delay the emergence of resistance to antibiotics in hospitals. One of the objectives is to increase adherence to prescribed antibiotics among users. Allowing patients to stop using antibiotics before the prescription ends is not one of the objectives and may increase the risk of resistance. Other objectives include ensuring that prescribers adhere to appropriate prescribing guidelines instead of developing their own and decreasing antibiotic use among parents of small children. REF: pp. 1005-1006

The nurse is preparing to care for a patient who will be taking an antihypertensive medication. Which action by the nurse is part of the assessment step of the nursing process? a . Asking the prescriber for an order to monitor serum drug levels b . Monitoring the patient for drug interactions after giving the medication c . Questioning the patient about over-the-counter medications d . Taking the patient's blood pressure throughout the course of treatment

ANS: C The assessment part of the nursing process involves gathering information before beginning treatment, and this includes asking about other medications the patient may be taking. Monitoring serum drug levels, watching for drug interactions, and checking vital signs after giving the medication are all part of the evaluation phase. page 6

A patient asks a nurse why a friend who is taking the same drug responds differently to that drug. The nurse knows that the most common variation in drug response is due to differences in each patient's: a. drug receptor sites. b. hypersensitivity potential. c. metabolism of drugs. d. psychosocial response.

ANS: C The most common source of genetic variation in drug response is related to alterations in drug metabolism and is determined by genetic codes for various drug-metabolizing isoenzymes. There are known genetic differences in codes for drug target sites, but these are not as numerous as those for metabolic isoenzymes. Hypersensitivity potential is also genetically determined, but variations produce differences in adverse reactions to drugs and not in drug effectiveness. Psychosocial responses vary for many less measurable reasons, such as individual personalities and variations in cultures. page 76-77

13. A nurse is preparing to administer intramuscular penicillin to a patient who is infected with T. pallidum and notes that the order is for sodium penicillin G. Which action is correct? a. Administer the drug as prescribed. b. Contact the provider to discuss administering the drug intravenously. c. Contact the provider to discuss changing the drug to benzathine penicillin G. d. Request an order for piperacillin instead of penicillin G.

ANS: C The procaine and benzathine penicillin salts are absorbed slowly and are considered repository preparations. When benzathine penicillin G is injected IM, penicillin G is absorbed for weeks and is useful only against highly sensitive organisms such as T. pallidum. Sodium penicillin G is absorbed rapidly, with peak effects in 15 minutes. Administering the drug IV will not yield repository effects. Piperacillin is not used for T. pallidum IC. REF: p. 1019

4. A patient with cancer has been taking an opioid analgesic four times daily for several months and reports needing increased doses for pain. What will the nurse tell the patient? a. PRN dosing of the drug may be more effective. b. The risk of respiratory depression increases over time. c. The patient should discuss increasing the dose with the provider. d. The patient should request the addition of a benzodiazepine to augment pain relief.

ANS: C This patient is developing tolerance, which occurs over time and is evidenced by the need for a larger dose to produce the effect formerly produced by a smaller dose. This patient should be encouraged to request an increased dose. PRN dosing is less effective than scheduled, around-the-clock dosing. The risk of respiratory depression decreases over time as patients develop tolerance to this effect. Benzodiazepines are CNS depressants and should not be given with opioids, because they increase the risk of oversedation.

5. AA woman in labor receives meperidine [Demerol] for pain. The nurse caring for the infant willobserve the infant closely for: a. c ongenital anomalies. b. excessive crying and sneezing. c. respiratory depression. d. tremors and hyperreflexia

ANS: C Use of morphine or other opioids during delivery can cause respiratory depression in the neonate, because the drug crosses the placenta. Infants should be monitored for respiratory depression and receive naloxone if needed. Opioids given during delivery do not contribute to birth defects in the newborn. Excessive crying and sneezing and tremors and hyperreflexia are signs of neonatal opioid dependence, which occurs with long-term opioid use by the mother during pregnancy and not with short-term use of these drugs during labor. REF: p. 277 / p. 284

A patient is taking a drug that has known toxic side effects. What will the nurse do? a. Discontinue the drug at the first signs of toxicity. b. Ensure that complete blood counts are ordered periodically. c. Monitor the function of all organs potentially affected by the drug. d. Teach the patient how to treat the symptoms if they develop.

ANS: C When a drug is administered that has known toxic side effects, the nurse is responsible for monitoring all organ systems potentially affected by the drug. Not all toxic side effects warrant discontinuation of the drug, and a nurse cannot discontinue a drug without an order from the provider. Complete blood counts are indicated only for drugs that affect the blood. Some drugs need to be discontinued, so teaching a patient to treat symptoms is not correct in all cases. page 66

16. A patient is receiving tobramycin 3 times daily. The provider has ordered a trough level with the 8:00 AM dose. The nurse will ensure that the level is drawn at what time? a. 4:00 AM b. 7:00 AM c. 7:45 AM d. 8:45 AM

ANS: C When a patient is receiving divided doses of an aminoglycoside, the trough level should be drawn just before the next dose; therefore, 7:45 AM would be the appropriate time. It would not be appropriate to draw a trough at the other times listed. REF: p. 1054

1. A nurse is caring for a client experiencing IV extravasation. The facility requires the administration of an antidote for the prescribed IV solution. After stopping the IV infusion, which of the following actions should the nurse take first? a. Remove the IV catheter b. Withdrawal the solution from the IV access c. Administer the antidote to the vesicant d. Insert a new IV access in a different extremity

Answer: b ATI Chapter 4

2. A nurse tells a nursing student that the glucocorticoids given for rheumatoid arthritis are nearly identical to substances produced naturally by the body. The student remarks that the drug must be very safe. Which response by the nurse is correct? a. "As long as the drug is taken as prescribed, side effects usually do not occur." b. "By interrupting the inflammatory process, these drugs inhibit side effects." c. "Side effects can occur and are dependent on the dose and duration of treatment." d. "The negative feedback loop prevents side effects."

ANS: C When taken in pharmacologic doses, as for inflammatory disorders, glucocorticoids can cause an array of serious adverse effects. When glucocorticoids are given in pharmacologic doses, side effects can occur even when the drugs are taken as prescribed. Interruption of the inflammatory process causes some of the adverse effects. The negative feedback loop does not inhibit side effects. REF: pp. 869-870

A patient reports becoming "immune" to a medication because it no longer works to alleviate symptoms. The nurse recognizes that this decreased effectiveness is likely caused by: a. antagonists produced by the body that compete with the drug for receptor sites. b. decreased selectivity of receptor sites, resulting in a variety of effects. c. desensitization of receptor sites by continual exposure to the drug. d. synthesis of more receptor sites in response to the medication.

ANS: C Continual exposure to an agonist would cause the cell to become less responsive or desensitized. The body does not produce antagonists as a response to a medication. Receptor site selectivity is determined by physiologic factors and not by the substances that bind to them. Medications do not cause more receptors to be produced. page 51

1. Which patients should be given antibiotics prophylactically? (Select all that apply.) a. Children who attend day care with other children who have strep throat b. Children undergoing chemotherapy who have mild neutropenia c. Patients with certain congenital heart defects at risk for bacterial endocarditis d. Patients with compound fractures undergoing surgical repair e. Postoperative patients who have undergone emergency cesarean sections

ANS: C, D Antimicrobial prophylaxis is recommended for patients with congenital or other heart disease who have an increased risk for bacterial endocarditis and for patients undergoing certain surgeries in which the risk for infection is very high, including those who have compound fractures. Children who are exposed to strep throat will need to have confirmed strep infection prior to receiving antibiotics. Patients with milk neutropenia do not need antimicrobial prophylaxis. Patients undergoing emergency C-sections need antimicrobial prophylaxis prior to surgery, not after. REF: p. 1012

Before administering a medication, what does the nurse need to know to evaluate how individual patient variability might affect the patient's response to the medication? (Select all that apply.) A. Chemical stability of the medication B. Ease of administration C. Family medical history D. Patient's age E. Patient's diagnosis

ANS: C, D, E The family medical history can indicate genetic factors that may affect a patient's response to a medication. Patients of different ages can respond differently to medications. The patient's illness can affect how drugs are metabolized. The chemical stability of the medication and the ease of administration are properties of drugs. page 3-4

2. Which are benefits of using a combination of two or more antibiotics? (Select all that apply.) a. Reduced cost b. Reduced risk of superinfection c. Reduced toxicity d. Reduced resistance e. Reduced risk in severe infection

ANS: C, D, E In some situations, an antibiotic combination can reduce toxicity, because the dosage of a more toxic agent can be reduced. Drug combinations can be used to suppress the emergence of resistant bacteria in the treatment of tuberculosis. With a severe infection, a combination of antibiotics is useful until culture results are known to ensure that all possible sources of infection are treated. Using multiple antibiotics is more costly and poses a greater risk of superinfection. REF: pp. 1011-1012

11. A patient has a skin infection and the culture reveals methicillin-resistant Staphylococcus aureus (MRSA). What is an appropriate treatment for this patient? a. Cefaclor b. Cefazolin c. Cefotaxime d. Ceftaroline

ANS: D Ceftaroline is a fifth-generation cephalosporin with a spectrum similar to third-generation cephalosporins but also with activity against MRSA. Cefaclor is a second-generation cephalosporin. Cefazolin is a first generation cephalosporin. Cefotaxime is a third-generation cephalosporin. REF: p. 1028

9. To promote a safety culture, the nurse manager preparing the staff schedule considers the anticipated census in planning the number and experience of staff on any given shift. Which is the human factor primarily addressed with this consideration? a. Available supplies b. Interdisciplinary communication c. Interruptions in work d. Workload fluctuations

ANS: D Including an adequate number of staff members with experience caring for anticipated patients is a strategy to manage the workload and potential fluctuations. A safety culture requires organizational leadership (e.g., the nurse manager) that gives attention to human factors such as managing workload fluctuations. This strategy also applies principles of crew resource management in that it addresses workload distribution. Lack of supplies can create a challenge for safe care but could not be addressed with the schedule. Concerns with communication and coordination across disciplines, including power gradients, and excessive professional courtesy can create hazards but would not be the best answer. Strategies to minimize interruptions in work are essential but would not be the best answer in this situation. Giddens Safety

The nurse is assessing a newly admitted older patient who has recently lost 15 pounds. The nurse notes that the patient is taking warfarin (Coumadin). Which laboratory tests will the nurse discuss with this patient's provider? a. Blood glucose and C-reactive protein b. Complete blood count and hepatic function tests c. Renal function tests and serum electrolytes d. Serum albumin and coagulation studies

ANS: D Older patients and those who are malnourished are at increased risk for low serum albumin. Since warfarin binds to albumin, such patients are at increased risk for elevated warfarin levels, which can cause increased bleeding. The nurse should request albumin levels and coagulation studies. page 79

5. In order to provide the best intervention for a patient, the nurse is often responsible for obtaining a sample of exudate for culture. What information will this provide? a. Whether a patient has an infection. b. Where an infection is located. c. What cells are being utilized by the body to attack an infection. d. What specific type of pathogen is causing an infection.

ANS: D People can transmit pathogens even if they don't currently feel ill. Some carriers never experience the full symptoms of a pathogen. A CBC will identify that the patient has an infection. Inspection and radiography will help identify where an infection is located. The CBC with differential will identify the white blood cells being used by the body to fight an infection. The culture will grow the microorganisms in the sample for identification of the specific type of pathogen. Chapter 24 Giddens

The nurse is teaching a patient how a medication works to treat an illness. To do this, the nurse will rely on knowledge of which topic? A. Clinical pharmacology B. Drug efficacy C. Pharmacokinetics D. Pharmacotherapeutics

ANS: D Pharmacotherapeutics is the study of the use of drugs to diagnose, treat, and prevent conditions. Clinical pharmacology is concerned with all aspects of drug-human interactions. Drug efficacy measures the extent to which a given drug causes an intended effect. Pharmacokinetics is the study of the impact of the body on a drug. page 1

2. A volunteer at the senior center asks the visiting nurse why the senior citizens always seem to be complaining about the temperature. What is the nurse's best response? a. Older people have a diminished ability to regulate body temperature because of active sweat glands. b. Older people have a diminished ability to regulate body temperature because of increased circulation. c. Older people have a diminished ability to regulate body temperature because of peripheral vasoconstriction. d. Older people have a diminished ability to regulate body temperature because of slower metabolic rates.

ANS: D Slower metabolic rates are one factor that reduces the ability of older adults to regulate temperature and be comfortable when there are any temperature changes. As the body ages, the sweat glands decrease in number and efficiency. Older adults have reduced circulation. The body conserves heat through peripheral vasoconstriction, and older adults have a decreased vasoconstrictive response, which impacts ability to respond to temperature changes. Chapter 10 Giddens

3. The strategy to avoid medication errors endorsed by the Institute for Safe Medication Practices (ISMP) to differentiate products with look-alike names is referred to as which term? a. Automatic alerts b. Bar coding c. Computer order entry d. Tallman lettering

ANS: D Tallman lettering is a term coined by ISMP to describe the practice of using unique letter characteristics of similar drug names known to have been confused with one another. Tallman lettering is used to differentiate products with look-alike names such as BenaDRYL (antihistamine) and BenaZEPRIL (ace inhibitor). The other options are examples of safety-enhancing technologies strategies designed to minimize drug errors, but they are not directed at look-alike medications. Automatic alerts are computer-generated alarms that can be programmed to occur with such things as allergies and incompatible medications. Bar coding is used with medication administration systems that can be programmed to match patient identification bracelets with documentation. Computer order entry systems are designed to include components of a standard medication order. Giddens Safety

4. What is the priority nursing action for a patient suspected to be hypothermic? a. Assess vital signs. b. Hydrate with intravenous (IV) fluids. c. Provide a warm blanket. d. Remove wet clothes.

ANS: D The first thing to do with a patient suspected to be hypothermic is to remove wet clothes, because heat loss is five times greater when clothing is wet. Assessing vital signs is important, but the wet clothes should be removed first. Hydration is very important with hyperthermia and the associated danger of dehydration, but there is not a similar risk with hypothermia. A warm blanket over wet clothes would not be an effective warming strategy. Chapter 10 Giddens

6. To promote safety, the nurse manager sensitive to point of care (sharp end) and systems level (blunt end) exemplars works closely with staff to address which point of care exemplar? a. Care coordination b. Documentation c. Electronic records d. Fall prevention

ANS: D The most common safety issues at the sharp end include prevention of decubitus ulcers, medication administration, fall prevention, invasive procedures, diagnostic workup, recognition of/action on adverse events, and communication. These are the most common issues the staff nurse providing direct patient care encounters. Each of the other options are classified as systems level exemplars. Giddens Safety

2. The nurse is caring for a patient experiencing an allergic reaction to a bee sting who has an order for diphenhydramine (BenaDRYL). The only medication in the patient's medication bin is labeled BenaZEPRIL. The nurse contacts the pharmacy for the correct medication to avoid what type of error? a. Communication b. Diagnostic c. Preventive d. Treatment

ANS: D The nurse avoided a treatment error, giving the wrong medication. Benazepril is an ace inhibitor used to treat blood pressure. According to Leape, treatment errors occur in the performance of an operation, procedure, or test; in administering a treatment; in the dose or method of administering a drug; or in avoidable delay in treatment or in responding to an abnormal test. Communication errors refer to those that occur from a failure to communicate. Diagnostic errors are the result of a delay in diagnosis, failure to employ indicated tests, use of outmoded tests, or failure to act on results of monitoring or testing. Preventive errors occur when there is inadequate monitoring or failure to provide prophylactic treatment or follow-up of treatment. Giddens Safety

5. The nurse is caring for a patient who was started on intravenous antibiotic therapy earlier in the shift. As the second dose is being infused, the patient reports feeling dizzy and having difficulty breathing and talking. The nurse notes that the patient's respirations are 26 breaths/min with a weak pulse of 112 beats/min. The nurse suspects that the patient is experiencing which condition? a. Suppressed immune response b. Hyperimmune response c. Allergic reaction d. Anaphylactic reaction

ANS: D The patient is exhibiting signs and symptoms of an anaphylactic reaction to the medication. These signs and symptoms during administration of a medication do not correspond to a suppressed immune response but a type of hyperimmune response. While the patient is experiencing a hyperimmune response, the signs and symptoms allow for a more specific response. While the patient is experiencing an allergic reaction, the signs and symptoms presented in the scenario allow for a more specific response. Chapter 22 Giddens

1. The nurse is teaching a patient about taking warfarin and asks if the patient takes aspirin. This assessment by the nurse reflects a knowledge of which type of drug interaction? a. Creation of unique effects b. Increased therapeutic effects c. Inhibitory effects d. Potentiative effects

ANS: D A potentiative effect is one in which one drug intensifies the effects of another. Both warfarin and aspirin suppress blood clotting, and the combination may increase the risk of bleeding, which is an intensified adverse effect. Creation of a unique effect is a rare occurrence in which the combination of two drugs creates a response not seen with either drug when given alone. Increased therapeutic effects are a type of potentiative effect; however, in this case the combination of two drugs would increase the desired effects. An inhibitory effect is a type of pharmacodynamic effect that occurs when an antagonist drug inhibits the action of an agonist drug at the same receptor site. page 55-56

A patient is given a drug for the first time and develops shortness of breath. The patient's heart rate is 76 beats per minute, the respiratory rate is 20 breaths per minute, and the blood pressure is 120/70 mm Hg. The nurse checks a drug administration manual to make sure the correct dose was given and learns that some patients taking the drug experience shortness of breath. The nurse will contact the provider to report what? a. An allergic reaction b. An idiosyncratic effect c. An iatrogenic response d. A side effect

ANS: D A side effect is a secondary drug effect produced at therapeutic doses. This patient received the correct dose of the drug and developed shortness of breath, which, in this case, is a drug side effect. To experience an allergic reaction, a patient must have prior exposure to a drug and sensitization of the immune response. An idiosyncratic effect results from a genetic predisposition to an uncommon drug response. An iatrogenic response occurs when a drug causes symptoms of a disease. page 62-63

A patient asks a nurse why drugs that have been approved by the FDA still have unknown side effects. What will the nurse tell the patient? a . Testing for all side effects of a medication would be prohibitively expensive. b . Patients in drug trials often are biased by their preconceptions of a drug's benefits. c . Researchers tend to conduct studies that will prove the benefits of their new drugs. d . Subjects in drug trials do not always represent the full spectrum of possible patients.

ANS: D All drug trials are limited by a relatively small group of subjects who may not have all the characteristics of people who will be using the drug; therefore, some side effects go undetected until the drug is in use. Although drug trials are very expensive, this is only an indirect reason they do not detect all side effects before approval. In theory, well-designed drug trials, using blinded studies, minimize or eliminate subject bias. Designing studies to prove desired results is unethical. page 17

10. The parent of an infant with otitis media asks the nurse why the prescriber has ordered amoxicillin [Amoxil] and not ampicillin [Unasyn]. What will the nurse tell the parent? a. Amoxicillin is a broader spectrum antibiotic than ampicillin. b. Amoxicillin is not inactivated by beta-lactamases. c. Ampicillin is associated with more allergic reactions. d. Ampicillin is not as acid stable as amoxicillin.

ANS: D Amoxicillin and ampicillin are similar in structure and actions but differ primarily in acid stability. Amoxicillin is more acid stable and, when administered orally, results in higher blood levels than can be obtained with equivalent doses of ampicillin. The two drugs have the same spectrum, both are inactivated by beta-lactamases, and both can cause allergic reactions. REF: pp. 1022-1023

4. A child with otitis media has had three ear infections in the past year. The child has just completed a 10-day course of amoxicillin [Amoxil] with no improvement. The parent asks the nurse why this drug is not working, because it has worked in the past. What will the nurse tell the patient? a. "Amoxicillin is too narrow in spectrum." b. "The bacteria have developed a three-layer cell envelope." c. "The bacteria have developed penicillin-binding proteins (PBPs) that have a low affinity for penicillins." d. "The bacteria have synthesized penicillinase."

ANS: D Beta-lactamases are enzymes that cleave the beta-lactam ring and render the PCN inactive. This resistance is common with organisms that cause ear infections. Amoxicillin is a broad-spectrum antibiotic. A three-layer cell envelope occurs in gram-negative bacteria. Some bacterial strains, including methicillin-resistant Staphylococcus aureus (MRSA), develop PBPs with a low affinity for penicillins. MRSA is not a common cause of otitis media. REF: pp. 1015-1017

12. A patient who has cystic fibrosis has a Pseudomonas aeruginosa infection and the provider has ordered aztreonam [Cayston]. What will the nurse teach this patient about administration of this drug? a. Administer the drug intramuscularly twice daily. b. Give a daily dose every day for 28 days and then stop. c. Inhale the powdered drug as ordered three times each day. d. Use the nebulizer to administer the drug three times daily.

ANS: D Cayston is a form of aztreonam formulated for inhalation administration for patients with cystic fibrosis who have P. aeruginosa lung infections. The reconstituted powder is given using a nebulizer system three times daily for 28 days followed by 28 days off. This form of the drug is not given IM. The dose is three times daily. The drug is reconstituted and administered via a nebulizer. REF: p. 1033

8. A hospitalized patient who is taking demeclocycline [Declomycin] reports increased urination, fatigue, and thirst. What will the nurse do? a. Contact the provider to report potential toxic side effects. b. Notify the provider to discuss changing the medication to doxycycline. c. Perform bedside glucometer testing to evaluate the serum glucose level. d. Provide extra fluids and reassure the patient that these are expected side effects.

ANS: D Demeclocycline stimulates urine flow and sometimes is used to treat patients with syndrome of inappropriate antidiuretic hormone secretion (SIADH). The patient should be reassured that increased urination, fatigue, and thirst are known side effects of demeclocycline. It is not correct to notify the provider of toxic side effects or to request another tetracycline. Glucometer testing is not necessary, because the increased urination is not related to an elevated blood glucose level. REF: p. 1040

A nurse provides teaching to a patient who will begin taking a drug with a known risk of hepatotoxicity. Which statement by the patient indicates a need for further teaching? a. "I should avoid taking acetaminophen while taking this drug." b. "I will need periodic evaluation of aspartate aminotransferase and alanine aminotransferase levels." c. "If I experience nausea, vomiting, or abdominal pain, I should call my provider." d. "Routine testing and early detection of problems will prevent liver failure."

ANS: D Drug-induced liver injury can progress from undetectable to advanced between routine tests; therefore, routine testing does not always prevent liver failure. Patients taking known hepatotoxic drugs should avoid other drugs, such as acetaminophen, that can cause liver damage. Aspartate aminotransferase (AST) and alanine aminotransferase (ALT) are liver enzymes that are routinely monitored when a patient is taking hepatotoxic drugs. Nausea, vomiting, and abdominal pain are signs of liver injury and should be reported. page 63-64

Which statement about food and drug interactions is true? a. Foods alter drug absorption and metabolism but not drug action. b. Medications are best absorbed on an empty stomach. c. Patient discomfort is the food and drug interaction of most concern. d. Some foods can inhibit CYP isoenzymes and alter drug metabolism.

ANS: D Grapefruit juice inhibits CYP3A4, which lowers the metabolism of some drugs, leading to toxic effects of drugs affected by these isoenzymes. Foods can alter all pharmacokinetic and pharmacodynamic processes. Not all medications are absorbed better on an empty stomach; some require certain foods to enhance absorption. Patient comfort is a concern, but it is not as important as more severe and possibly life-threatening food and drug interactions. page 59-60

18. A child has been receiving chloramphenicol for a Neisseria meningitidis central nervous system (CNS) infection. The nurse administers the dose and subsequently notes that the child has vomited and appears dusky and gray in color. The child's abdomen is distended. What will the nurse do? a. Contact the provider for an order to obtain a chloramphenicol level. b. Notify the provider that the child's meningitis is worsening. c. Recognize this as initial signs of a C. difficile infection. d. Stop the infusion immediately and notify the provider.

ANS: D Gray syndrome is a potentially fatal toxicity associated with chloramphenicol use. When symptoms occur, the drug should be stopped immediately. Lower chloramphenicol levels may prevent gray syndrome, but lowering the dose will not stop symptoms once they have appeared. These are not signs of worsening meningitis or a C. difficile infection. REF: p. 1045

The nurse is teaching a patient about home administration of insulin to treat diabetes mellitus. As part of the teaching, the patient and nurse identify goals to maintain specific blood glucose ranges. This represents which aspect of the nursing process? a . Assessment b . Evaluation c . Implementation d . Planning

ANS: D In the planning step, the nurse delineates specific interventions directed at solving or preventing problems. When creating the care plan, the nurse defines goals, sets priorities, and establishes criteria for evaluating success. The assessment step involves collecting data about the patient. The evaluation step involves evaluating the medication effectiveness. The implementation step identifies actions that are taken to administer the drug. page 9

2. The nurse is caring for a patient on a medical-surgical unit who has a fever of unknown origin. The prescriber has ordered a broad-spectrum antibiotic. Which intervention is the priority? a. Administering the antibiotic immediately b. Administering antipyretics as soon as possible c. Delaying administration of the antibiotic until the culture results are available d. Obtaining all cultures before the antibiotic is administered

ANS: D It is essential that samples of exudates and body fluids (in this case, blood cultures) be obtained for culture before initiation of treatment. Administration of the antibiotic is important but not more important than obtaining specimens for culture. Antipyretics may be indicated, but the priority is obtaining specimens for culture. Treatment may be initiated before the test results are available. REF: pp. 1007-1008

A patient is using a metered-dose inhaler containing albuterol for asthma. The medication label instructs the patient to administer "2 puffs every 4 hours as needed for coughing or wheezing." The patient reports feeling jittery sometimes when taking the medication, and she doesn't feel that the medication is always effective. Which action is outside the nurse's scope of practice? a . Asking the patient to demonstrate use of the inhaler b . Assessing the patient's exposure to tobacco smoke c . Auscultating lung sounds and obtaining vital signs d . Suggesting that the patient use one puff to reduce side effects

ANS: D It is not within the nurse's scope of practice to change the dose of a medication without an order from a prescriber. Asking the patient to demonstrate inhaler use helps the nurse to evaluate the patient's ability to administer the medication properly and is part of the nurse's evaluation. Assessing tobacco smoke exposure helps the nurse determine whether nondrug therapies, such a smoke avoidance, can be used as an adjunct to drug therapy. Performing a physical assessment helps the nurse evaluate the patient's response to the medication. page 9-10

A patient who is taking morphine for pain asks the nurse how a pain medication can also cause constipation. What does the nurse know about morphine? a. It binds to different types of receptors in the body. b. It can cause constipation in toxic doses. c. It causes only one type of response, and the constipation is coincidental. d. It is selective to receptors that regulate more than one body process.

ANS: D Morphine is a medication that is selective to receptor type that regulates more than one process. Because it is selective to receptor type, it does not bind to different types of receptors. Constipation is a normal side effect and is not significant for toxicity. page 48-49

2. A patient who has had abdominal surgery has been receiving morphine sulfate via a patient-controlled analgesia (PCA) pump. The nurse assesses the patient and notes that the patient's pupils are dilated and that the patient is drowsy and lethargic. The patient's heart rate is 84 beats per minute, the respiratory rate is 10 breaths per minute, and the blood pressure is 90/50 mm Hg. What will the nurse do? a. Discuss possible opiate dependence with the patient's provider. b. Encourage the patient to turn over and cough and take deep breaths. c. Note the effectiveness of the analgesia in the patient's chart. d. Prepare to administer naloxone and possibly ventilatory support.

ANS: D Opioid toxicity is characterized by coma, respiratory depression, and pinpoint pupils. Although pupils are constricted initially, they may dilate as hypoxia progresses, which also causes blood pressure to drop. This patient has a respiratory rate of fewer than 12 breaths per minute, dilated pupils, and low blood pressure; the patient also is showing signs of central nervous system (CNS) depression. The nurse should prepare to give naloxone and should watch the patient closely for respiratory collapse. Patients with opioid dependence show withdrawal symptoms when the drug is discontinued. When postoperative patients have adequate analgesia without serious side effects, encouraging patients to turn, cough, and breathe deeply is appropriate. This patient is probably relatively pain free, but providing emergency treatment is the priority. REF: pp. 265-266

A nurse is preparing to care for a patient who is receiving digoxin. When screening for potential adverse effects from this drug, the nurse will review which of this patient's laboratory results? a. Albumin b. Blood urea nitrogen (BUN) and creatinine c. Hepatic enzymes d. Serum electrolytes

ANS: D Patients with low serum potassium are at risk for fatal cardiac dysrhythmias when taking digoxin, and it is essential to know this level before this medication is administered. Knowing a patient's albumin level would be important when giving drugs that are protein bound. The BUN and creatinine levels are indicators of renal function. Hepatic enzymes are important to know when drugs are metabolized by the liver. page 74-75

The nurse is preparing to administer penicillin G intramuscularly to a child. The child's parents ask why the drug cannot be given in an oral liquid form. What is the nurse's reply? a. "This drug causes severe gastric upset if given orally." b. "This drug has a narrow therapeutic range, and the dose must be tightly controlled." c. "This drug is absorbed much too quickly in an oral form." d. "This drug would be inactivated by enzymes in the stomach."

ANS: D Penicillin G is inactivated by digestive enzymes in the stomach and cannot be given orally. It does not have a narrow therapeutic range. page 27-28

8. A patient with chronic pain has been receiving morphine sulfate but now has decreased pain. The prescriber changes the medication to pentazocine [Talwin]. The nurse will monitor the patient for: a. euphoria. b. hypotension. c. respiratory depression. d. yawning and sweating.

ANS: D Pentazocine is an agonist-antagonist opioid, and when given to a patient who is physically dependent on morphine, it can precipitate withdrawal. Yawning and sweating are early signs of opioid withdrawal. Pentazocine does not produce euphoria, hypotension, or respiratory depression. REF: pp. 264-265 / pp. 272-273

A patient has been receiving intravenous penicillin for pneumonia for several days and begins to complain of generalized itching. The nurse auscultates bilateral wheezing and notes a temperature of 38.5°C (101°F). Which is the correct action by the nurse? a . Administer the next dose and continue to evaluate the patient's symptoms. b . Ask the prescriber if an antihistamine can be given to relieve the itching. c . Contact the prescriber to request an order for a chest radiograph. d . Hold the next dose and notify the prescriber of the symptoms.

ANS: D Pruritus and wheezing are signs of a possible allergic reaction, which can be fatal; therefore, the medication should not be given and the prescriber should be notified. When patients are having a potentially serious reaction to a medication, the nurse should not continue giving the medication. Antihistamines may help the symptoms of an allergic reaction, but the first priority is to stop the medication. Obtaining a chest radiograph is not helpful. page 10

A child ingests a parent's aspirin tablets, and the prescriber orders sodium bicarbonate to block the toxic effects of the aspirin. The nurse caring for this patient knows that sodium bicarbonate is effective against the aspirin because it: a. accelerates its passage through the intestine. b. alters urinary pH to enhance renal excretion. c. induces CYP isoenzymes to increase drug metabolism. d. raises the pH of the interstitial fluid to facilitate passage out of the cells.

ANS: D Sodium bicarbonate increases the pH of interstitial fluid and plasma, allowing the acidic aspirin ions to move outside the cells and thus removing them from the site where they have toxic effects. It does not have laxative effects and does not alter the rate of passage through the gut. It is not a CYP isoenzyme inducer and therefore has no effect on drug metabolism. It does not alter renal excretion of aspirin. page 56

7. A patient is brought to the emergency department by friends, who report finding the patient difficult to awaken. The friends report removing two fentanyl transdermal patches from the patient's arm. On admission to the emergency department, the patient has pinpoint pupils and a respiratory rate of 6 breaths per minute. A few minutes after administration of naloxone, the respiratory rate is 8 breaths per minute and the patient's pupils are dilated. The nurse recognizes these symptoms as signs of: a. a mild opioid overdose. b. decreased opioid drug levels. c. improved ventilation. d. worsening hypoxia.

ANS: D The classic triad of symptoms of opioid overdose are coma, respiratory depression, and pinpoint pupils. The pupils may dilate as hypoxia worsens, and this symptom, along with continued respiratory depression (fewer than 12 breaths per minute), indicates worsening hypoxia. Fentanyl is a strong opioid, so this is not likely to be a mild overdose, because the patient was wearing two patches. Fentanyl continues to be absorbed even after the patches are removed because of residual drug in the skin, so the drug levels are not likely to be decreasing. The patient does not have improved ventilation, because the respiratory rate is still fewer than 12 breaths per minute. REF: p. 267

A nurse is reviewing a medication administration record before administering medications. Which order will the nurse implement? a. Furosemide [Lasix] 20 mg QD PO b. Furosemide [Lasix] 20 mg qd PO c. Furosemide [Lasix] 20 mg daily d. Furosemide [Lasix] 20 mg PO daily

ANS: D The correct answer is a complete order; it contains the medication, dose, route, and time. "qd" is no longer an accepted abbreviation; it should be written out as "daily" or "every day." The order of "20 mg daily" does not specify the route to be used. page 70

The nurse is caring for a child who has ingested a toxic amount of aspirin. The provider orders an intravenous drug that will increase pH in the blood and urine. The nurse understands that this effect is necessary to: a. decrease the gastric absorption of aspirin. b. decrease the lipid solubility of aspirin. c. increase the serum protein binding of aspirin. d. increase the urinary excretion of aspirin.

ANS: D The phenomenon of pH-dependent ionization can be used to accelerate renal excretion of drugs. When children have been exposed to toxic amounts of aspirin, they can be treated, in part, by giving an agent that elevates urinary pH, leading to less passive reabsorption of the now ionized molecules of aspirin and, hence, more excretion. Elevating the pH of the blood and urine does not affect absorption in the stomach. Ionization of aspirin does not affect lipid solubility or protein binding. page 25-26

7. An older male patient with an increased risk of MI is taking furosemide [Lasix] and low-dose aspirin. The patient is admitted to the hospital, and the nurse notes an initial blood pressure of 140/80 mm Hg. The patient has had a 10-pound weight gain since a previous admission 3 months earlier. The patient has voided only a small amount of concentrated urine. The serum creatinine and blood urea nitrogen (BUN) levels are elevated. The nurse will contact the provider to discuss: a. adding an antihypertensive medication. b. obtaining serum electrolytes. c. ordering a potassium-sparing diuretic. d. withdrawing the aspirin.

ANS: D This patient shows signs of renal impairment, as evidenced by weight gain despite the use of diuretics, decreased urine output, hypertension, and elevated serum creatinine and BUN. ASA can cause acute, reversible renal impairment and should be withdrawn. Hypertensive medications do not treat the underlying cause. Serum electrolytes are not indicated. Addition of a potassium-sparing diuretic is not indicated. REF: p. 853

5. 5. A patient who takes aspirin for rheumatoid arthritis is admitted to the hospital complaining of headache and ringing in the ears. The plasma salicylate level is 300 mcg/mL, and the urine pH is 6.0. What will the nurse do? a. Increase the aspirin dose to treat the patient's headache. b. Notify the provider of possible renal toxicity. c. Prepare to provide respiratory support, because the patient shows signs of overdose. d. Withhold the aspirin until the patient's symptoms have subsided

ANS: D This patient shows signs of salicylism, which occurs when ASA levels climb just slightly above the therapeutic level. Salicylism is characterized by tinnitus, sweating, headache, and dizziness. Tinnitus is an indication that the maximum acceptable dose has been achieved. Toxicity occurs at a salicylate level of 400 mcg/mL or higher. ASA should be withheld until the symptoms subside and then should be resumed at a lower dose. Increasing the dose would only increase the risk of toxicity. Signs of renal impairment include oliguria and weight gain, which are not present in this patient. This patient has salicylism, not salicylate toxicity, so respiratory support measures are not indicated. REF: pp. 853-854

15. A patient develops CDAD. Which antibiotic is recommended for treating this infection? a. Chloramphenicol b. Clindamycin [Cleocin] c. Linezolid [Zyvox] d. Vancomycin

ANS: D Vancomycin and metronidazole are the drugs of choice for treating REF: p. 1043

8. A patient has a localized skin infection, which is most likely caused by a gram-positive cocci. Until the culture and sensitivity results are available, the nurse will expect the provider to order a ____-spectrum ____ agent. a. broad; systemic b. broad; topical c. narrow; systemic d. narrow; topical

ANS: D When infections are treated before the causative agent has been identified, and after cultures have been obtained, antibiotics may be used based on the knowledge of which microbes are most likely to cause infection at that particular site. Because this is a localized infection, a topical agent is recommended. Unless the infection is very serious, a narrow-spectrum antibiotic is best. REF: p. 1007

9. A parent asks a nurse if the provider will prescribe an antibiotic for a child who attends school with several children who have strep throat. The child is complaining of a sore throat and has a fever. What will be the nurse's response? a. "Because strep throat is likely, your child should be treated empirically." b. "With good hand washing, your child should not get strep throat." c. "Your child probably has strep throat, so your provider will order an antibiotic." d. "Your child should come to the clinic to have a throat culture done today."

ANS: D Whenever possible, the infecting organism should be identified before antibiotics are started, even if there is a strong suspicion that a particular organism is present. The nurse is correct in telling the parent to bring the child to the clinic for a throat culture. Fever and sore throat have other causes, so it is not correct to treat this child empirically, especially because these symptoms are not severe or life threatening. Although good hand washing is always indicated, this child already has symptoms and needs to be evaluated. Antibiotics should not be started until indicated by cultures. REF: p. 1009

A postoperative patient has orders for morphine sulfate 1 to 2 mg IV every 1 hour PRN for severe pain and acetaminophen-hydrocodone [Lortab] 7.5 mg PO every 4 to 6 hours PRN for moderate pain. The patient reports pain at a level of 8 on a scale of 1 to 10, with 10 being the worst pain. Which action by the nurse is appropriate? a . Administer acetaminophen-hydrocodone 7.5 mg PO every 4 hours. b . Administer acetaminophen-hydrocodone 7.5 mg PO every 6 hours and change to every 4 hours if not effective. c . Administer morphine sulfate 1 mg IV every 1 hour until pain subsides. d . Administer morphine sulfate 2 mg IV and evaluate the patient's pain in 15 to 30 minutes.

ANS: D With PRN medications, the schedule is not fixed and the administration of these medications depends on the patient's condition. It is the nurse's responsibility to assess the patient's condition and then give the appropriate PRN medication. In this case, the patient has severe pain and should receive MS IV. Either 1 mg or 2 mg may be given, but the nurse must evaluate the effectiveness of the pain medication within 15 to 30 minutes to help determine subsequent doses. Acetaminophen-hydrocodone is not appropriate because it is ordered for moderate pain and this patient reports severe pain. Giving MS IV every hour is not appropriate for a PRN medication unless the patient's condition warrants it. page 8

1. A nurse is preparing to administer medications to a four month old infant. Which of the following pharmacokinetic principles should the nurse consider when administering medication to the client? Select all that apply a. infants have more rapid gastric emptying time. b. Infants have immune liver function until 1 year of age c. infants blood-brain barrier is poorly developed d. infants have an increased ability to absorb topical medication e. infants have an increased number of protein binding sites

Answer: b, c, d ATI Chapter 6

4. A nurse is preparing to administer eye drops to a client. Which of the following actions should the nurse take? Select all that apply a. Have the client lie on one side b. Ask the client to look up at the ceiling c. Tell the client to blink when the drops enter the eye d. Stop the medication into the client's conjunctival sac e. Instruct the client to close the eyes gently after instillation

Answer: B,D,E ATI Chapter 1

2. a nurse in a provider's office is reviewing the medical record of a client who is pregnant and at the first prenatal visit. Which of the following immunizations can the nurse administer safely to this client? a. Varicella vaccine b. rubella vaccine c. inactivated influenza vaccine d. measles vaccine

Answer: C ATI Chapter 6

1. The nurse is preparing a client's medication. Which of the following actions should the nurse take in following legal practice guidelines? Select all that apply a. Teach the client about medication b. Determine the dosage c. Monitor for adverse effects d. Lock compartments for controlled substances e. Determine the client's insurance status

Answer: a,c,d ATI Chapter 2

1. A nurse in a clinic is caring for a group of clients. The nurse should contact the provider about potential contraindication to a medication for which of the following clients? Select all that apply a. A client at 8 weeks of gestation who asks for an influenza immunization b. A client who takes prednisone and has a possible fungal infection c. A client who has chronic liver disease and is taking hydrocodone / acetaminophen d. A client who has peptic ulcer disease, takes sucralfate, and has started taking OTC aluminum hydroxide e. A client who has a prosthetic heart valve, takes Warfarin, and reports of suspected pregnancy

Answer: b, c, e ATI Chapter 5

2. A nurse is preparing to initiate IV therapy for an older adult client. Which of the following actions should the nurse plan to take? a. Use a disposable razor to remove excess hair on the extremity b. Select the back of the clients hand to insert IV catheter c. Distended veins by using a blood pressure cuff d. Direct the client to raise their arm above their heart

Answer: c ATI Chapter 4

4. A nurse is reviewing a client's health record and notes that the client experienced permanent extrapyramidal effects caused by a previous medication. The nurse should recognize that the medication affected which of the following systems in the client? a. Cardiovascular b. Immune c. Central nervous d. Gastrointestinal

Answer: c ATI Chapter 5

5. A nurse is caring for a client who is taking oral oxycodone. The client is also taking ibuprofen in three recommended doses daily. The nurse should identify that an interaction between these two medications will cause which of the following findings? a. A decrease in blood levels of ibuprofen, possibly leading to a need for increased doses of this medication b. A decrease in blood levels of Oxycodone, possibly leading to a c. need for increased doses of this medication c. An increase in the expected therapeutic effect of both medications d. An increase in expected adverse effects for both medications

Answer: c ATI Chapter 5

2. A nurse is preparing to administer digoxin to a client who states, "I don't want to take that medication. I do not want one more pill." Which of the following responses should the nurse make? a. Your physician prescribed it for you, so you really should take it b. Well, let's just get it over quickly then c. Okay, I'll give you your other medications d. Tell me your concerns about taking this medicine

Answer: d ATI Chapter 2

3. A nurse is giving discharge instructions for a client who has a new prescription for an antihypertensive medication. Which of the following statements should the nurse give? a. Be sure to limit your potassium intake while taking the medication b. You should check your blood pressure every 8 hours while taking this medication c. Your medication dosage will be increased if you develop tachycardia d. Change positions slowly when you move from sitting to standing

Answer: d ATI Chapter 5

A health care professional is caring for a patient who is about to begin taken acyclovir (Zovirax) to treat a herpes simplex infection. The health care professional should monitor which of the following laboratory values for the patient a. prothrombin time b. Hct c. BUN d. aspartate aminotransferase

C. BUN Rational: Acyclovir, an antiviral drug, can cause renal toxicity because of drug accumulation in renal tubules. The health care professional should monitor the patient's urine output, BUN, and creatinine levels, and increase fluid intake to hydrate and flush the kidneys ATI Infection

4) Which laboratory test result suggests that a patient's excretion of a drug might be impaired? a) WBC: 13,000/mm3 b) eGFR: 30 mL.min c) INR: 4.2 d) AST: 72 IU

b) eGFR: 30 mL.min Quiz 1 Health Care Quality and Safety

5)A male patient is prescribed clopidogrel to prevent blood clots after experiencing a myocardial infarction (MI, AKA heart attack). He takes over the counter omeprazole for heartburn. Because omeprazole can inhibit the efficacy of clopidogrel, it is a priority for the nurse to teach the patient to report which effect? a) Shortness of breath or chest pain b) Joint pain c) Nausea and vomiting d) Dizziness

a) Shortness of breath or chest pain Quiz 1 Health Care Quality and Safety

9. A patient with chronic pain has a fentanyl patch applied to his right shoulder. The patient reports that his arm hurts and he requests a warm pack to apply to the area. Which statement by the nurse is correct? a. "Putting a warm pack on the area where the fentanyl patch is located could accelerate fentanyl release." b. "Your arm probably hurts because of the fentanyl patch." c. "I can remove your patch and reapply it after you are done with the warm pack." d. "The fentanyl patch is heat resistant, so I will get you a warm pack."

a. "Putting a warm pack on the area where the fentanyl patch is located could accelerate fentanyl release." Warn patients using fentanyl patches to avoid exposing the patch to direct heat (eg, heating pad, hot tub) because doing so can accelerate fentanyl release. Patches should not be removed and reapplied; use a new patch as ordered if one is removed. Patches should not cause pain, and they are not heat resistant. Chapter 28

6. When administering a central nervous system depressant, the nurse should closely observe for drug toxicity in which patient? a. A 3-week-old neonate b. A 12-year-old boy c. A 25-year-old woman d. A 15-month-old infant

a. A 3-week-old neonate The blood-brain barrier is not fully developed at birth. As a result, newborns are much more sensitive than older children or adults to medicines that act on the brain. Chapter 4

6. After administering acetylcysteine [Mucomyst] to a patient who overdosed on acetaminophen [Tylenol], a nurse should recognize which outcome as an indicator of the therapeutic effects of acetylcysteine? a. Absence of jaundice b. Clear breath sounds c. Increased bowel sounds d. Palpable pedal pulses

a. Absence of jaundice Acetylcysteine [Mucomyst] substitutes for depleted glutathione in the reaction that removes the toxic metabolite of acetaminophen [Tylenol] (which accumulates with acetaminophen poisoning) and thereby minimizes liver damage. Severe hepatic injury may occur with acetaminophen [Tylenol] poisoning, which is manifested by jaundiced sclera and skin. The assessment of bowel sounds, breath sounds, and pedal pulses is not used to determine the therapeutic effects of acetylcysteine [Mucomyst] for the treatment of acetaminophen overdose. Chapter 71

10. What does the nurse identify as a possible adverse effect of long-term glucocorticoid therapy? (Select all that apply.) a. Adrenal insufficiency b. Osteoporosis c. Hypoglycemia d. Hyperkalemia e. Cataracts

a. Adrenal insufficiency b. Osteoporosis e. Cataracts Adverse effects of long-term glucocorticoid therapy include adrenal insufficiency, osteoporosis, hyperglycemia, hypokalemia, and cataracts. Chapter 72

10. Characteristics unique to each patient can influence pharmacokinetic and pharmacodynamic processes. What characteristics may determine the patient's response to a drug? (Select all that apply.) a. Age b. Gender c. Weight d. Mood e. Genetics

a. Age b. Gender c. Weight e. Genetics Characteristics unique to each patient can influence pharmacokinetic and pharmacodynamic processes and, by doing so, can help determine the patient's response to a drug. Sources of individual variation include drug interactions; physiologic variables (eg, age, gender, weight); pathologic variables (especially diminished function of the kidneys and liver, the major organs of drug elimination); and genetic variables Chapter 1

6. The nurse understands drug response varies from one individual to another. What are examples of individual variations? (Select all that apply.) a. Age b. Genetic makeup c. Gender d. Diet e. Failure to take medication as prescribed All of the responses are examples of individual variations in drug responses.

a. Age b. Genetic makeup c. Gender d. Diet e. Failure to take medication as prescribed All of the responses are examples of individual variations in drug responses. Chapter 8

9. Which statements about medication administration would the nurse identify as true? (Select all that apply.) a. All drugs have the potential to produce undesired effects. b. Drug therapy often can be enhanced by nonpharmacologic measures. c. Patients taking two drugs are not likely to have a drug interaction. d. Nurses' knowledge of pharmacology is more important for standing orders than for PRN medications. e. Patient adherence is essential in achieving the therapeutic objective of medications. All drugs have the potential to produce undesired effects. Drug therapy often can be enhanced by nonpharmacologic measures, such as physical therapy or dietary changes. Patients taking two drugs are at risk for drug interactions. PRN medications require a high level of nursing discretion, judgment, and knowledge and are not less important than standing orders. Patient adherence is the extent to which a patient's behavior coincides with medical advice. This is especially important for patients taking medications at home.

a. All drugs have the potential to produce undesired effects. b. Drug therapy often can be enhanced by nonpharmacologic measures. e. Patient adherence is essential in achieving the therapeutic objective of medications. All drugs have the potential to produce undesired effects. Drug therapy often can be enhanced by nonpharmacologic measures, such as physical therapy or dietary changes. Patients taking two drugs are at risk for drug interactions. PRN medications require a high level of nursing discretion, judgment, and knowledge and are not less important than standing orders. Patient adherence is the extent to which a patient's behavior coincides with medical advice. This is especially important for patients taking medications at home. Chapter 2

5. The drug dobutamine acts as an agonist of norepinephrine (NE) receptors. Which effect is the nurse most likely to observe in a patient receiving this medication? a. Increased heart rate b. Atrioventricular heart block c. Sinus bradycardia d. Reduced cardiac output

a. Increased heart rate Dobutamine mimics the action of NE at receptors on the heart, thereby causing an increase in the heart's rate and force of contraction. Chapter 5

10. Which statements about the Family Smoking Prevention and Tobacco Control Act (2009) would the nurse identify as true? (Select all that apply.) a. All ingredients in tobacco products must be listed on the warning label. b. A gradual reduction of nicotine to nonaddictive levels is required. c. Marketing to youth is prohibited. d. Harmful additives are restricted. e. Purchasers of tobacco products must be age 21 years or older.

a. All ingredients in tobacco products must be listed on the warning label. b. A gradual reduction of nicotine to nonaddictive levels is required. c. Marketing to youth is prohibited. d. Harmful additives are restricted. All of the statements about the Family Smoking Prevention and Tobacco Control Act (2009) are true except that purchasers of tobacco products must be age 21 years or older. TECHNICALLY the book is old and ALL answers are correct, but for the intent of the book, e is wrong. Chapter 3

6. The nurse prepares to give a drug that will prevent receptor activation. Which term would describe this drug? a. Antagonist b. Selective c. Agonist d. Potent

a. Antagonist An antagonist is a drug that prevents receptor activation. An agonist is a molecule that activates receptors. A selective drug has only the desired response but may not activate receptors. A potent drug requires a lower dose to achieve its effect. Chapter 5

3. The nurse is reviewing the laboratory value for a patient prescribed atorvastatin [Lipitor]. Which laboratory value is most useful for monitoring this drug? a. Aspartate aminotransferase (AST) b. Blood urea nitrogen (BUN) c. International normalized ratio (INR) d. C-reactive protein (CRP)

a. Aspartate aminotransferase (AST) AST is a liver enzyme that is helpful for monitoring liver function (hepatotoxicity). Lipitor, a lipid-lowering drug, is a commonly prescribed example of a hepatotoxic drug. The BUN is a measure of kidney function. The INR is a comparative rating of prothrombin time ratios that is used to monitor patients taking the anticoagulant agent warfarin. The CRP is elevated in inflammatory and neoplastic disease, myocardial infarction, and the third trimester of pregnancy. It is used as a cardiac risk marker. Chapter 7

7. The nurse reviews all of the patient's prescriptions. Which prescribed medications may cause a detrimental potentiative drug interaction? a. Aspirin and warfarin b. Sulbactam and ampicillin c. Propranolol and albuterol d. Isoniazid and rifampin

a. Aspirin and warfarin Aspirin and warfarin both suppress clotting. As a result, if aspirin and warfarin are taken concurrently, the risk of bleeding is significantly increased. Chapter 6

1. The nurse is caring for a 12-year-old boy who weighs 72 pounds. The healthcare provider should make the most precise dosage adjustments for this patient's medications based on what? a. Body surface area b. Body mass index c. Body weight d. Body fat percentage

a. Body surface area Adjustments based on the body surface area account not only for the patient's weight but also for the patient's relative amount of body adiposity. Chapter 8

3. Why is it important for drugs to have ease of administration? a. Fewer administration errors b. Less risk of side effects c. Greater chemical stability d. Greater likelihood of reversibility

a. Fewer administration errors Ease of administration increases convenience and adherence and can reduce administration errors. Ease of administration is not related to side effects, chemical stability, or reversibility. Chapter 1

3. A nurse teaches a patient who takes daily low-dose aspirin for protection against myocardial infarction and stroke to avoid also taking which medication? a. Ibuprofen [Motrin] b. Zolpidem [Ambien] c. Loratadine [Claritin] d. Diphenhydramine [Benadryl]

a. Ibuprofen [Motrin] Ibuprofen [Motrin] can block the antiplatelet effects of aspirin; therefore, patients who take low-dose aspirin to protect against myocardial infarction and thrombosis should avoid taking ibuprofen [Motrin]. It is not necessary to avoid taking zolpidem [Ambien], loratadine [Claritin], or diphenhydramine [Benadryl] while taking aspirin. Chapter 71

9. What does the nurse identify as possible complications of glucocorticoid therapy? (Select all that apply.) a. Impaired carbohydrate and glucose tolerance b. Increased production of glucocorticoids from the adrenals c. Predisposition to gastrointestinal ulceration d. Decrease in serum sodium and glucose levels e. Increase in plaque development in coronary arteries

a. Impaired carbohydrate and glucose tolerance c. Predisposition to gastrointestinal ulceration Adverse effects of glucocorticoids occur in response to pharmacologic doses; they include suppression of adrenal glucocorticoids; glucose and carbohydrate intolerance, elevated blood glucose level; water and sodium retention; and increased gastric acid secretion. Plaque development in arteries is not affected. Chapter 72

1. Which response would the nurse anticipate when giving two drugs that have a potentiative effect, such as meperidine and Phenergan? a. Increased pain relief b. Increased nausea and vomiting c. Decreased itching d. Increased alertness

a. Increased pain relief A potentiative effect occurs when one drug intensifies the effects of another. An inhibitory effect would cause reduced therapeutic effects or reduced adverse effects. Potentiative effects are not unique responses. Meperidine is a morphine derivative for pain relief. Phenergan is an antiemetic that potentiates the effect of meperidine. The patient should experience decreased pain and also may be drowsy. Chapter 6

8. What is the ultimate concern for the nurse when administering a drug? a. Intensity of the response b. Dosage c. Route of administration d. Timing of administration

a. Intensity of the response The ultimate concern for the nurse when administering a drug is the intensity of the response, which is determined by the dosage size, route of administration, and timing of administration. Chapter 1

3. A nurse in the emergency department is performing an admission assessment on a patient admitted with hepatotoxicity related to long-term acetaminophen use. The nurse would expect to find: (Select All That Apply) a. Intestinal distress b. Jaundice skin c. Increased creatinine levels d. Increased glomerular filtration rate e. Icteric sclera

a. Intestinal distress b. Jaundice skin e. Icteric sclera Quiz 2 Pain and Thermoregulation

5. The nurse is preparing a staff education inservice about specific safety measures that reduce patient medication errors. Which measure improves safety for patients during care transition? a. Medication reconciliation b. MEDWATCH program c. Risk Evaluation and Mitigation Strategy d. Regional Medication Safety Program

a. Medication reconciliation Medication reconciliation is a widely used specific patient safety measure that reduces medication errors during transitions of care (eg, hospital admission and discharge and moving to a different level of care within a hospital). b. Medication reconciliation has resulted in decreasing medication errors by 70% and reducing adverse drug reaction by 15%. Chapter 7

6. A diabetic patient is taught the signs and symptoms of hypoglycemia with insulin use. The patient knows to eat fast-acting carbohydrates. This is an example of which goal? a. Minimizing adverse effects b. Promoting therapeutic effects c. Minimizing adverse interactions d. Managing toxicity

a. Minimizing adverse effects Hypoglycemia can be an adverse effect of insulin. Knowledge of adverse drug effects will enable the patient to avoid some adverse effects and minimize others through early detection. Promoting therapeutic effects, minimizing adverse interaction (between two medications or between medications and food), and managing toxicity (too much) Chapter 2

6. Which agent is most likely to cause serious respiratory depression as a potential adverse reaction? a. Morphine [Duramorph] b. Pentazocine [Talwin] c. Hydrocodone [Lortab] d. Nalmefene [Revex]

a. Morphine [Duramorph] Morphine is a strong opioid agonist and as such has the highest likelihood of causing respiratory depression. Pentazocine, a partial agonist, and hydrocodone, a moderate to strong agonist, may cause respiratory depression, but they do not do so as often or as seriously as morphine. Nalmefene, an opioid antagonist, would be used to reverse respiratory Chapter 28

4. The nurse is caring for a group of female patients receiving medication therapy. Which factor is of greatest concern with regard to drug therapy in these patients? a. Most drug research has been carried out exclusively in male subjects. b. Hormonal differences make managing drug therapy more difficult in most women. c. Overall, women tend to be less compliant with medication therapy. d. Women tend to be caregivers and may not take time to care for themselves.

a. Most drug research has been carried out exclusively in male subjects. For most drugs, not much is known about gender-related differences, because until recently all drug research was done in men. Chapter 8

9. Which statements about over-the-counter (OTC) drugs would the nurse identify as true? (Select all that apply.) a. Most illnesses initially are treated with an OTC agent. b. More prescription drugs than OTC agents are administered each year in the United States. c. The average home medicine cabinet contains eight OTC preparations. d. Forty percent of Americans take at least one OTC drug every 2 days. e. Some drugs that originally were sold by prescription are now OTC.

a. Most illnesses initially are treated with an OTC agent. d. Forty percent of Americans take at least one OTC drug every 2 days. e. Some drugs that originally were sold by prescription are now OTC. Most illnesses are initially treated with an OTC agent. OTC drugs are administered more than prescription drugs. OTC drugs account for 60% of all doses administered. The average home medicine cabinet contains 24 OTC preparations. Forty percent of Americans take at least one OTC drug every 2 days. Some drugs that were originally sold only by prescription are now sold over the counter. Chapter 3

1. A postoperative patient has an epidural infusion of morphine sulfate [Astramorph]. The patient's respiratory rate declines to 8 breaths per minute. Which medication would the nurse anticipate administering? a. Naloxone [Narcan] b. Acetylcysteine [Mucomyst] c. Methylprednisolone [Solu-Medrol] d. Protamine sulfate

a. Naloxone [Narcan] Naloxone is a narcotic antagonist that can reverse the effects, both adverse and therapeutic, of opioid narcotic analgesics. Chapter 28

7. The nurse is caring for a patient prescribed Isoniazid for the treatment of tuberculosis. The nurse should assess for which signs and symptoms of drug-induced liver toxicity? (Select all that apply.) a. Nausea b. Malaise c. Jaundice d. Vomiting e. Clear urine

a. Nausea b. Malaise c. Jaundice d. Vomiting Drug toxicity is an adverse drug reaction in which certain drugs are toxic to specific organs. Signs and symptoms of liver toxicity include jaundice, dark urine, light-colored stools, nausea, vomiting, malaise, abdominal discomfort, and loss of appetite. Chapter 7

8. The nurse is obtaining a drug history for a patient admitted to the unit. The nurse obtains information about past and present health histories, currently used prescription drugs, behavioral factors, and use of over-the-counter (OTC) drugs. What other information does the nurse need to obtain? (Select all that apply.) a. Use of recreational drugs and substances b. Usual sleep patterns and disturbances c. Highest level of education completed d. Use of herbal remedies e. Self-treatment with complementary and alternative drugs When answering a multiple-response question, consider each option independently. In this case, three answers are correct. The nurse must take a holistic approach when assessing the drug history. Recreational drugs and substances, herbal remedies, and self-treatment with complementary and alternative drugs are vital components of a patient's drug history. Sleep patterns and level of education are not considered vital information in the drug history.

a. Use of recreational drugs and substances d. Use of herbal remedies e. Self-treatment with complementary and alternative drugs When answering a multiple-response question, consider each option independently. In this case, three answers are correct. The nurse must take a holistic approach when assessing the drug history. Recreational drugs and substances, herbal remedies, and self-treatment with complementary and alternative drugs are vital components of a patient's drug history. Sleep patterns and level of education are not considered vital information in the drug history. Chapter 2

10. When educating patients about their medications, the nurse includes information about which topics? (Select all that apply.) a. What to do if a dose is missed b. The duration of treatment c. Prescription of drug coverage d. The method of drug storage e. Symptoms of adverse effects

a. What to do if a dose is missed b. The duration of treatment d. The method of drug storage e. Symptoms of adverse effects Prescription of drug coverage is not considered part of essential patient drug information. The remaining options are topics the nurse would include in patient education. Chapter 2

A health care professional is caring for a patient who is taking warfarin (Coumadin) and is about to begin taking trimethoprim/sulfamethoxazole (Bactrim) to treat a urinary tract infection. The health care professional should question the drug regimen because taking these two drugs concurrently can increase the patient's risk for which of the following? a. bleeding b. thrombosis c. ECG changes d. ototoxicity

a. bleeding Rational: Trimethoprim/sulfamethoxazole, a sulfonamide combination can increase the effects of warfarin and increase the patient's risk for bleeding. ATI Infection

A patient who is taking ciprofloxacin to treat a respiratory tract infection contacts the health care professional to report dyspepsia. The health care professional to report dyspepsia. The health care care professional should recommend which of the following instructions? a. take an antacid at least 2 hr after taking the drug b. take the drug with a cup of coffee c. take an ion supplement with the drug d. drink 8 oz of milk with the drug

a. take an antacid at least 2 hr after taking the drug ATI Infection

A health care professional is caring for a patient who is about to begin taking chloroquine to prevent malaria. When talking with the patient about taking the drug, the health care professional should include which of the following instructions? (Select all that apply.) a. wear sunglasses outdoors b. take the drug with an antacid c. avoid driving d. take the drug with lemonade e. take the drug with food

a. wear sunglasses outdoors c. avoid driving e. take the drug with food Rational: chloroquine, an antimalarial drug, can cause photophobia. Sunglasses can help reduce the risk of eye damage and irritation because of this effect. It can cause blurred vision, nausea and diarrhea. Taking the drug with food can help minimize these effects ATI Infection

3. A nurse is teaching a group of nursing students why glucocorticoids are preferred over nonsteroidal anti-inflammatory drugs in the treatment of inflammation. Which statement by a student indicates a need for further teaching? a. "Glucocorticoids reduce the immune component of inflammation." b. "Glucocorticoids have fewer side effects than nonsteroidal anti-inflammatory drugs." c. "Glucocorticoids help avert damage to tissues from lysosomal enzymes." d. "Glucocorticoids act by multiple mechanisms and have more anti-inflammatory effects than NSAIDs."

b. "Glucocorticoids have fewer side effects than nonsteroidal anti-inflammatory drugs." Quiz 3 Inflammation and Immunity

3. A patient admitted to the hospital with a diagnosis of pneumonia asks the nurse, "Why am I receiving codeine? I don't have any pain." The nurse's response is based on the knowledge that codeine also has which effect? a. Immunostimulant b. Antitussive c. Expectorant d. Decongestant

b. Antitussive Codeine provides analgesic and antitussive therapeutic effects. Chapter 28

4. N-acetyl-para-aminophenol is an example of which type of drug name? a. Proprietary b. Chemical c. Generic d. Trade

b. Chemical The chemical name is a description of a drug using the nomenclature of chemistry. The proprietary or trade name of the drug is the brand name under which the drug is marketed by one or more companies. The generic name is the name assigned by the U.S. Adopted Names Council and is the same regardless of who manufactures the drug. Chapter 3

6. When studying the effects of drugs in humans, the nurse is learning about what? a. Pharmacology b. Clinical pharmacology c. Therapeutics d. Effectiveness

b. Clinical pharmacology Clinical pharmacology is the study of the effects of drugs in humans. Pharmacology can be defined as the study of drugs and their interactions with living systems. Therapeutics, also known as pharmacotherapeutics, is the use of drugs to diagnose, prevent, or treat disease or to prevent pregnancy. The term effectiveness indicates that the drug elicits the intended response or responses. Chapter 1

4. The nurse teaches a patient not to consume alcohol with nitroglycerine, because the blood pressure often drops significantly when nitroglycerine is taken with alcohol. Which drug property does this illustrate? a. Chemical instability b. Drug interaction c. Reversible action d, Drug selectivity

b. Drug interaction When two or more drugs are taken together, they can interact, causing either increased or decreased drug responses. In this case, alcohol would increase the nitroglycerine response. Chemical instability, reversible action, and drug selectivity are not related to this situation. Chapter 1

8. Which is a true statement about new drug development? (Select all that apply.) a. Preclinical testing of drugs is always performed in healthy, nonpregnant adults. b. Drug trials require that those involved don't know which subjects are receiving the drug or control. c. During preclinical testing, drugs are evaluated for toxicities, kinetic properties, and useful effects. d. When a new drug is released, all adverse effects are known.

b. Drug trials require that those involved don't know which subjects are receiving the drug or control c. During preclinical testing, drugs are evaluated for toxicities, kinetic properties, and useful effects Preclinical drug testing is not performed in humans; it is done mostly in animals. Because drug testing occurs in a relatively small number of patients, those patients are carefully selected, the drug is taken for a relatively short time, and not all adverse effects are detected during the drug testing process. During the testing process, randomized controlled trials are used, in which the participants are unaware of which subjects are getting drug, placebo, or control. Kinetics, toxicities, and effectiveness are tested during the preclinical phase of drug testing. Chapter 3

9. The nurse administers 100 mg of drug X by mouth. After the drug moves through the hepatic system, very little active drug is left in the general circulation as a result of what? a. Therapeutic range b. First-pass effect c. Drug half-life d. Plasma protein binding

b. First-pass effect The term first-pass effect refers to the rapid hepatic inactivation of certain oral drugs. Drugs that undergo the first-pass effect often are administered parenterally. The therapeutic range is the range of drug level between the minimum effective concentration (MEC) and the toxic concentration. The drug half-life is the time required for the amount of drug in the body to decrease by 50%. Plasma protein binding is involved with the transport of drugs through the bloodstream. Chapter 4

5) A patient has been taking ibuprofen for pain and inflammation. The nurse knows that ibuprofen is a non-steroidal anti-inflammatory, cyclo-oxygenase inhibitor (COX1 and COX2) and therefore will warn the patient of which potential side effect? a. Acidosis b. Gastric ulceration c. Prolonged QT interval d. Hepatotoxicity

b. Gastric ulceration Quiz 2 Pain and Thermoregulation

1. The nurse assesses a patient who takes ibuprofen [Advil] on a regular basis. Which finding does the nurse know is an adverse effect of ibuprofen [Advil] therapy? a. Hives b. Hematemesis c. Dysmenorrhea d. Jaundice

b. Hematemesis Ibuprofen is a member of the non-aspirin first-generation nonsteroidal anti-inflammatory drugs (NSAIDs). Through inhibition of cyclooxygenase-1 (COX-1) and cyclooxygenase-2 (COX-2), ibuprofen poses a risk for gastric ulceration and bleeding, which may lead to hematemesis. Ibuprofen is used to reduce inflammation, fever, and pain and therefore is effective in reducing dysmenorrhea (painful menstrual cramping). It is not known to cause hives or jaundice, which are signs of impaired liver function. Chapter 71

2. The nurse is planning care for a patient receiving morphine sulfate [Duramorph] by means of a patient-controlled analgesia (PCA) pump. Which intervention may be required because of a potential adverse effect of this drug? a. Administering a cough suppressant b. Inserting a Foley catheter c. Administering an antidiarrheal d. Monitoring liver function tests

b. Inserting a Foley catheter Morphine can cause urinary hesitancy and urinary retention. If bladder distention or inability to void is noted, the prescriber should be notified. Urinary catheterization may be required. Morphine acts as a cough suppressant and an antidiarrheal, so neither of those types of drugs would be needed to counteract an adverse effect of morphine. Liver toxicity is not a common adverse effect of morphine. Chapter 28

7. Why does the nurse administer naloxone to a patient receiving morphine sulfate who has a respiratory rate of 8 breaths per minute? a. Naloxone causes hypersensitivity of the opioid receptors. b. Naloxone prevents the activation of opioid receptors. c. Naloxone is a partial agonist, requiring a lesser dose to achieve pain relief. d. Naloxone is an agonist, leading to desensitization of the opioid receptors.

b. Naloxone prevents the activation of opioid receptors. Naloxone is an antagonist, which prevents the activation of opioid receptors, reversing the respiratory depression effects of morphine. Continuous exposure of cells to antagonists can result in hypersensitivity. Continuous exposure of cells to agonists can lead to desensitization, refractoriness, or down-regulation. Chapter 5

4. The nurse is managing the care of a group of patients with cancer who will be receiving chemotherapy. The nurse defines goals, sets priorities, identifies interventions, and establishes criteria for evaluating success. Which phase of the nursing process does this represent? a. Assessment b. Planning c. Implementation d. Evaluation

b. Planning These activities are all carried out in the planning phase of the nursing process. Assessment is a time of data gathering, implementation begins with carrying out the interventions, and evaluation is performed to determine the degree to which treatment has succeeded. Chapter 2

7. A patient who has rheumatoid arthritis is scheduled to start taking celecoxib [Celebrex]. A nurse should recognize which factor from the patient's history as a contraindication to taking this medication? a. Hypothyroidism b. Recent heart bypass surgery c. Positive tuberculin skin test result d. Allergy to penicillin

b. Recent heart bypass surgery Celecoxib [Celebrex] should be avoided in patients who have undergone recent heart bypass surgery. Because it does not inhibit COX-1, platelet aggregation is not suppressed. It does inhibit COX-2 in blood vessels, which results in increased vasoconstriction. Unimpeded platelet aggregation and increased vasoconstriction pose a higher risk of thrombotic events in patients with certain cardiovascular risk factors. Hypothyroidism, a penicillin allergy, and a positive tuberculin skin test result are not contraindications to taking celecoxib [Celebrex]. Chapter 71

1. A new graduate nurse preparing to administer medications knows that which of the following is required for a drug to move through the body? a. Selectivity and effectiveness b. The ability to cross membranes c. Development of an electric charge d. A transporter protein

b. The ability to cross membranes To move throughout the body, drugs must cross membranes. They cross membranes to enter the bloodstream, to exit the bloodstream and reach the site of action, and to undergo metabolism and excretion. Selectivity and effectiveness are not related to drug movement. Development of an electric charge (ionization) reduces a drug's ability to be absorbed. Transporter proteins are not required for drugs to move through the body. Chapter 4

7. The nurse is working on a postoperative unit in which pain management is part of routine care. Which statement is the most helpful in guiding clinical practice in this setting? a. At least 30% of the U.S. population is prone to drug addiction and abuse. b. The development of opioid dependence is rare when opioids are used for acute pain. c. Morphine is a common drug of abuse in the general population. d. The use of PRN (as needed) dosing provides the most consistent pain relief without risk of addiction.

b. The development of opioid dependence is rare when opioids are used for acute pain. The development of dependence on or addiction to opioids as a result of clinical exposure is extremely rare. In fact, some estimate that only 25% of patients receive doses of opioids that are sufficient to relieve suffering. Only about 8% of the population is estimated to be prone to drug abuse. Morphine is a drug of abuse, but this fact is not helpful in guiding clinical practice. A patient-controlled analgesia (PCA) pump provides the most consistent pain relief, better than PRN and fixed-dosing schedules. Chapter 28

7. When questioning a patient about his or her allergic reaction to a drug, the nurse should ask for which additional information? a. The dose taken b. The type of reaction c. Who gave the drug d. What was eaten with the drug

b. The type of reaction When planning drug therapy, we must identify patients who are at high risk of reacting adversely. We must determine if the patient has had an allergic reaction to a drug in the past as well as details about the type of reaction. Patients may provide the other information, but it is not as important as identifying the type of reaction. Chapter 2

2. The nurse is caring for a patient who is experiencing a respiratory rate of 6 breaths per minute as a result of a large dose of pain medication. Which term most accurately describes this reaction? a. Side effect Incorrect b. Toxicity c. Allergic reaction d. Idiosyncratic effect

b. Toxicity Toxicity is the degree of detrimental physiologic effects caused by excessive drug dosing. A side effect is a nearly unavoidable secondary drug effect produced at a therapeutic dose. An allergic reaction is an immune response. An idiosyncratic effect is an uncommon drug response resulting from a genetic predisposition. Chapter 7

5. Major adverse effects of aminoglycoside antibiotics include damage to: (Select All That Apply) a. lungs b. cochlea c. stomach d. heart e. kidneys f. vestibular apparatus

b. cochlea e. kidneys f. vestibular apparatus Quiz 4 Infection

A health care professional is caring for a patient who is about to begin taking cephalexin (Keflex) to treat bacterial meningitis. The health care professional should explain to the patient the need to monitor which of the following laboratory tests? a. prothrombin time b. creatinine c. aspartate aminotransferase d. potassium

b. creatinine Rational: excreted by the kidneys. The health care professional should evaluate renal function prior to and during treatment with the drug. Patients who have renal insufficiency should receive a reduced dose of the drug or another drug that does not affect renal function. ATI Infection

A health care professional is caring for a patient who is about to begin receiving acyclovir (Zovirax) IV to treat a viral infection. The health care professional should recognize that cautions use of the drug is essential if the patient also has which of the following? a. heart failure b. dehydration c. asthma d. tinnitus

b. dehydration Rational: Acyclovir, can cause renal toxicity, especially in patients who are dehydrated. Hydration during and after IV infusion of the drug can help keep it from precipitating in the renal tubules. ATI Infection

Which of the following drugs should a provider prescribe for a patient who has streptococcal pharyngitis and is allergic to penicillin? a. nafcillin b. erythromycin c. cephalexin d. amoxicillin/clavulanic acid

b. erythromycin Rational: Erythromycin, a macrolide, is an acceptable alternative to penicillin for patients who have bacterial infections and are allergic to penicillin. The drug is effective against most gram-positive bacteria, including streptococci, and some gram-negative bacteria ATI Infection

A patient who is taking tetracycline orally to treat a chlamydia infection contacts the health care professional to report severe blood-tinged diarrhea. Recognizing the adverse effects of tetracycline, the health care professional should suspect which of the following? a. hemorrhoids b. pseudomembranous enterocolitis c. diverticular disease d. small bowel obstruction

b. pseudomembranous enterocolitis Rational: severe diarrhea, often containing mucus and blood, can indiciate pseudomembranous colitis. Treatment includes stopping drug therapy and replacing fluids and electrolytes. ATI Infection

A patient who is taking imipenem to treat a bacterial infection contacts the health care professional to report inability to eat because of mouth pain. Recognizing the adverse effects of imipenem, the health care professional should suspect which of the following? a. malabsorption b. suprainfection c. anorexia d. dental caries

b. suprainfection ATI Infection

11. The nurse will include which statements when teaching a patient about the use of acetaminophen [Tylenol]? (Select all that apply.) a. "Acetaminophen is a useful drug for the treatment of inflammation, such as a rheumatoid arthritis." b. "The most common side effect of treatment with the drug is kidney failure." c. "Notify your healthcare provider if you notice that your skin or eyes are turning yellow." d. "Do not routinely use acetaminophen to prevent vaccine-associated fever and pain." e. "Use of this drug can prevent heart attack and stroke."

c. "Notify your healthcare provider if you notice that your skin or eyes are turning yellow." d. "Do not routinely use acetaminophen to prevent vaccine-associated fever and pain." Acetaminophen [Tylenol] is used to treat fever and pain. It is not an anti-inflammatory drug. The most serious side effect of acetaminophen therapy is liver failure; therefore, the healthcare provider should be notified if indications of jaundice are seen, such as yellowing of the skin or sclera. Acetaminophen therapy has no antiplatelet activity; therefore, it is not used to prevent heart attack or stroke. Routine use of acetaminophen may blunt the immune response to vaccines; therefore, it should be avoided as routine treatment for vaccine-associated fever and pain. Chapter 71

2. The nurse is preparing a discharge teaching plan to a patient prescribed phenobarbital and oral contraceptives which are known to induce CYP isoenzymes. What patient teaching should the nurse include in the discharge plan? a. "Continue taking your medications as prescribed." b. "Condoms are not necessary while taking phenobarbital. It is not an antibiotic." c. "Plan to use another form of birth control while taking phenobarbital." d. "Your dose of birth control pills will be reduced while you are taking phenobarbital."

c. "Plan to use another form of birth control while taking phenobarbital." Phenobarbital induces CYP isoenzymes; therefore, it will increase the metabolism of other drugs. Because phenobarbital is an inducing agent, it will increase the metabolism of oral contraceptives. The nurse should anticipate that this will likely reduce the blood levels of birth control pills. The patient should use another form of birth control while taking phenobarbital. Chapter 6

5. The nurse is teaching a patient with cancer about a new prescription for a fentanyl [Sublimaze] patch, 25 mcg/hr, for chronic back pain. Which statement is the most appropriate to include in the teaching plan? a. "You will need to change this patch every day, regardless of your pain level." Incorrect b. "This type of pain medication is not as likely to cause breathing problems." c. "With the first patch, it will take about 24 hours before you feel the full effects." d. "Use your heating pad for the back pain. It will also improve the patch's effectiveness."

c. "With the first patch, it will take about 24 hours before you feel the full effects." Full analgesic effects can take up to 24 hours to develop with fentanyl patches. Most patches are changed every 72 hours. Fentanyl has the same adverse effects as other opioids, including respiratory depression. Patients should avoid exposing the patch to external heat sources, because this may increase the risk of toxicity. Chapter 28

8. The healthcare provider orders the following: Prednisone 5 mg PO q6h × 4 days 5 mg PO q12h × 3 days 5 mg PO qDay × 3 days Dispense 5-mg tablets After completing 6 days of treatment the patient calls and asks how many pills should be left in the bottle because some may have been lost. How many pills should be left? a. 10 b. 6 c. 5 d. 4

c. 5 After completing 6 days of treatment, the patient would have taken 20 out of 25 total tablets. There should be 5 tablets remaining; 2 tablets for the last day of taking them every 12 hours, and then 3 tablets left for the last 3 days of taking them every day. Chapter 72

8. The nurse is preparing to administer an intravenous (IV) medication. What is the minimum injection time to reduce the risk of harm to the patient? a. 10 seconds b. 30 seconds c. 60 seconds d. 30 minutes

c. 60 seconds Most IV drugs should be injected slowly over at least 1 minute or longer, because all the blood in the body is circulated about once every minute. This allows the drug to be diluted in the largest volume of blood possible. Chapter 4

4. The nurse is monitoring for adverse drug reactions (ADRs) of assigned patients. Which patient is most at risk for the development of drug toxicity? a. A 30-year-old man admitted for altered mental status b. A 55-year-old woman with abnormal arterial blood gas values c. A 70-year-old woman with an elevated creatinine level d. A laboring 25-year-old woman with a positive Homans' sign

c. A 70-year-old woman with an elevated creatinine level The liver, kidneys, and bone marrow are important sites of drug toxicity. Creatinine is a measure of kidney function and would be the most helpful for monitoring for ADRs. In addition, patients over age 65 years are at greater risk for ADRs. Mental status is a measure of the function of the central nervous system (CNS), which may be affected by drugs but is not one of the most important and common sites of drug toxicity. Arterial blood gas measurements reflect respiratory and acid-base function. Chapter 7

2. Cefazolin has been prescribed at discharge for a patient with pelvic inflammatory disease. Due to the possibility of a disulfiram-like reaction, the patient should avoid consuming what? a. Milk Products b. Aspirin c. Alcohol d. Tyramine containing foods

c. Alcohol Quiz 4 Infection

1. The nurse understands that the dose-response relationship is graded and should expect to observe which response? a. Once a drug is given, the response is predictably all-or-nothing. b. The response is maintained at a specific level when the therapeutic objective is achieved. c. As the dosage increases, the response becomes progressively greater. d. A graded response is based on relative potency and maximal efficacy.

c. As the dosage increases, the response becomes progressively greater. If drug responses were all-or-nothing instead of graded, drugs could produce only one intensity level of response. The response may be maintained at a specific level when the therapeutic objective is achieved, but that option does not pertain to a dose-response relationship that is graded. Chapter 5

2. The nurse demonstrates the concept of maximal efficacy by administering which drug for a headache that the patient describes as a "mild dullness" and as a 2 on a 1 to 10 scale? a. Meperidine [Demerol] b. Pentazocine [Talwin] c. Aspirin d. Morphine sulfate

c. Aspirin Maximal efficacy is the largest effect a drug can produce. Potency is the amount of drug that must be given to elicit an effect. Maximal efficacy illustrates the fact that all drugs have a maximal effect, and dosages beyond this do not increase the effect. The goal is to match the intensity of the response to the patient's needs; therefore, a drug with high maximal efficacy is not always most desirable. Demerol, Talwin, and morphine all have a higher maximal efficacy than aspirin; therefore, aspirin is the most desirable drug for a headache rated as "mild." Chapter 5

6. Why are trade names much easier to say and remember than generic names? a. The FDA assigns every drug's generic name. b. Trade names must be the same regardless of which drug company manufactures the drug. c. Companies have a marketing advantage when the trade name is easier to recognize. d. Trade names improve oral and written communication in the healthcare system.

c. Companies have a marketing advantage when the trade name is easier to recognize. The U.S. Adopted Names Council assigns a drug's generic name. Generic names, not trade names, are the same regardless of which company manufactures the drug. Trade names do not contribute to ease and clarity of communication in the healthcare system, because healthcare professionals do not always know what compound is described by the trade name. Chapter 3

1. The nurse is caring for a patient who has jaundice, dark urine, malaise, light-colored stools, nausea, and vomiting. What is this patient most likely experiencing? a. An idiosyncratic drug effect on the bone marrow b. Iatrogenic disease of the kidneys c. Drug toxicity of the liver d. An allergic reaction

c. Drug toxicity of the liver Drug toxicity is an adverse drug reaction in which certain drugs are toxic to specific organs. Signs and symptoms of liver toxicity include jaundice, dark urine, light-colored stools, nausea, vomiting, malaise, abdominal discomfort, and loss of appetite. Chapter 7

7. Which statement by a new nurse indicates that further study is indicated? a. Effectiveness is the most important property a drug can have. b. There is no such thing as a safe drug. c. Drugs are defined as illegal substances. d. There is no such thing as a selective drug; all medications cause side effects.

c. Drugs are defined as illegal substances. A drug is any chemical that can affect living processes. All the other statements are correct. Chapter 1

1. When administering prednisone to a patient, the nurse will do what? a. Administer the prednisone in the evening to coincide with the natural secretion pattern of the adrenal cortex. b. Instruct the patient to stop taking the prednisone immediately if diarrhea develops. c. Ensure that meals are at bedside so that administration with food reduces gastric irritation. d. Avoid intravenous delivery to prevent adverse effects.

c. Ensure that meals are at bedside so that administration with food reduces gastric irritation. Glucocorticoids given in larger pharmacologic doses for nonendocrine causes many adverse effects, including gastric irritation and ulcers. They should be given with food. Doses should be administered before 9:00 AM to maximize endocrine function. Prednisone should not be stopped abruptly; the patient should contact the healthcare provider before discontinuing the medication. The drug may be administered by many routes, including the parenteral route. Chapter 72

8. A nurse administers naloxone [Narcan] to a postoperative patient experiencing respiratory sedation. What undesirable effect would the nurse anticipate after giving this medication? a. Drowsiness b. Tics and tremors c. Increased pain d. Nausea and vomiting

c. Increased pain Naloxone reverses the effects of narcotics. Although the patient's respiratory status will improve after administration of naloxone, the pain will be more acute. Chapter 28

6. The nurse is screening a patient before administering a live virus vaccine. The patient is currently taking a glucocorticoid medication. Which action should the nurse take? a. Continue screening and give the vaccine if appropriate. b. Note the contraindication but give the vaccine anyway. c. Note the contraindication and clarify the order with the healthcare provider. d. Hold the vaccine and notify the department of health.

c. Note the contraindication and clarify the order with the healthcare provider. The nurse should note the contraindication and clarify the order with the provider. Because of their immunosuppressant actions, glucocorticoids can decrease antibody responses to vaccines. Accordingly, immunization should not be attempted while glucocorticoids are in use. Furthermore, if a live virus vaccine is employed, there is an increased risk of developing viral disease. There is no need to contact the health department. Chapter 72

2. A patient is scheduled to start taking aspirin for the treatment of rheumatoid arthritis. The nurse anticipates that which medication most likely will be prescribed? a. Docusate sodium [Colace] b. Ascorbic acid (vitamin C) c. Pantoprazole [Protonix] d. Furosemide [Lasix]

c. Pantoprazole [Protonix] Aspirin inhibits COX-2 and thus suppresses inflammation and reduces moderate pain and fever. It also suppresses COX-1, which increases the risk for gastric ulceration and bleeding. This risk can be reduced through administration of a proton pump inhibitor, such as pantoprazole [Protonix]. It is not necessary for a patient to take vitamin C or docusate sodium while taking aspirin. Although aspirin may cause sodium and water retention in patients who have pre-existing renal dysfunction, it is not necessary for all patients to take furosemide [Lasix] with aspirin. Chapter 71

3. A patient is being prescribed vancomycin orally for clostridium dificil (C.diff) that was unable to be treated with metronidazole. The patients lab results reveal a creatinine level of 2.2mg/dL. What is the best action by the nurse? a. Discontinue the medication b. Hold the medication and prepare the patient for dialysis c. Have the labs redrawn d. Administer the medication

d. Administer the medication Quiz 4 Infection

3. Which is a true statement about new drug development in the United States? a. Development and testing of new drugs take about 3 to 5 years. b. About 50% of drugs undergoing clinical trials gain approval. c. Randomized controlled trials are the best way to assess drug therapy. d. The cost of developing a new drug is usually around $1 million.

c. Randomized controlled trials are the best way to assess drug therapy. New drug development takes about 10 to 15 years, and costs can exceed $1.2 billion. Only about one in five drugs undergoing clinical trials gains approval. Randomized controlled trials are the most reliable way to objectively assess drug therapies, and are used to evaluate all new drugs. Chapter 3

3. The drug the nurse is about to give induces P-glycoprotein (PGP). Which outcome should the nurse expect when this drug is given with other drugs? a. Increased levels of other drugs b. Increased side effects of other drugs c. Reduced absorption of other drugs d. Reduced drug elimination

c. Reduced absorption of other drugs Drugs that induce PGP can cause reduced absorption of other drugs, which would reduce their levels. A PGP inducer would not increase the side effects of other drugs and could increase elimination of other drugs. Chapter 6

4. A nurse should recognize that a patient who takes an angiotensin-converting enzyme (ACE) inhibitor while also taking high-dose aspirin is at risk of developing what complication? a. Congestive heart failure b. Liver toxicity c. Renal failure d. Hemorrhage

c. Renal failure High-dose aspirin therapy should be avoided in patients taking ACE inhibitors. In susceptible patients, these medications can impair renal function when they are combined with aspirin. Liver toxicity, congestive heart failure, and hemorrhage are not effects of ACE inhibitor and aspirin interactions. Chapter 71

1. A nurse is preparing to give an oral dose of drug X to treat a patient's high blood pressure. After giving the drug, the nurse finds that it not only reduces the blood pressure without serious harmful effects, but it also causes the patient to have nausea and headache. Based on this information, which property of an ideal drug is this drug lacking? a. Effectiveness b. Safety c. Selectivity d. Ease of administration

c. Selectivity The drug is effective in lowering the blood pressure and safe in that it does not cause harmful effects. However, as do most drugs, it causes other effects besides the one response desired; therefore, it lacks selectivity. The oral form provides ease of administration. Chapter 1

4. A patient is about to receive prednisone for tendonitis. The nurse reviewing the chart would be concerned about which of the following in the patient's medical history? a. Asthma and allergic rhinitis b. Gouty Arthritis c. Systemic Fungal infection d. Seborrheic dermatitis

c. Systemic Fungal infection Quiz 3 Inflammation and Immunity

5. When studying the impact a drug has on the body, the nurse is reviewing what? a. The drug's pharmacokinetics b. The drug's selectivity c. The drug's pharmacodynamics d. The drug's predictability

c. The drug's pharmacodynamics Pharmacodynamics can be thought of as the impact of drugs on the body. Pharmacokinetics describes the movement of drugs through the body. Selectivity is the ability of a drug to elicit only the response for which it is given. Predictability is the degree of certainty about how a patient will respond to a certain drug. Chapter 1

8. Why does the nurse monitor the patient closely when administering a drug with a low therapeutic index? a. The average lethal dose of the drug is much higher than the therapeutic dose. b. The dose required to produce a therapeutic response in 50% of patients is low. c. The highest dose needed to produce a therapeutic effect is close to the lethal dose. d. There is a low variability of responses to this drug.

c. The highest dose needed to produce a therapeutic effect is close to the lethal dose. A low therapeutic index indicates that the high doses needed to produce therapeutic effects in some people may be large enough to cause death. A high therapeutic index is more desirable, because the average lethal dose is higher than the therapeutic dose. Low variability of responses to a drug is not the definition of a low therapeutic index. Chapter 5

2. The nurse is preparing to administer a medication with the following order: "Aldomet 250 mg daily." What should the nurse do? a. Administer the medication as it was given last time. b. Administer the medication by mouth. c. Verify the order with the prescriber. d. Ask the patient how this medication is usually given.

c. Verify the order with the prescriber. This order does not include a drug route. The nurse should clarify any questionable orders with the prescriber. The other responses are incorrect. Chapter 2

A health care professional is preparing to administer amphotericin B IV to a patient who has a systemic fungal infection. Which of the following drugs should the health care professional administer prior to the infusion to prevent or minimize adverse reactions during amphotericin B administration? (Select all that apply.) a. aspirin b. hydrocortisone c. acetaminophen d. diphenhydramine e. ibuprofen

c. acetaminophen d. diphenhydramine Rational: Infusion reactions to IV amphotericin B start 1 to 2 hr after the infusion begins and subside within 4 hr. The health care professional can help prevent these effects by administering acetaminophen prior to the infusion. The health care professional can help prevent adverse reactions by administering diphenhydramine prior to the infusion. ATI Infection

A patient who is taking amoxicillin to treat a respiratory infection contacts the health care professional to report a rash and wheezing. Which of the following instructions should the health care professional provide? a. wait 1 hr and contact the provider if there is no improvement b. skip today's dose of amoxicillin and resume taking the drug tomorrow c. call emergency services immediately d. take an NSAID to reduce the skin and airway inflammation

c. call emergency services immediately ATI Infection

A health care professional is caring for a patient who is about to begin taking nitrofurantoin (Macrodantin) to treat a urinary tract infection. The health care professional should tell the patient to report which of the following adverse effects of the drugs? a. constipation b. dark brown urine c. cough d. tremors

c. cough Rational: Nitrofurantoin, a urinary tract antiseptic, can cause respiratory symptoms, such as cough, SOB, chest pain, and fever. ATI Infection

A health care professional is caring for a patient who is about to begin taking ketoconazole to treat a fungal infection. The health care professional should tell the patient to report which of the following adverse effects of the drug? a. tingling in the hands and feet b. joint pain c. gynecomastia d. excessive sweating

c. gynecomastia Rational: Ketoconazole, an azole antifungal drug can cause a temporary decrease in sexual hormone synthesis, resulting in gynecomastia, reduced libido, low sperm counts, and menstrual irregularities ATI Infection

A health care professional is caring for a patient who is about to begin gentamicin therapy to treat an infection. The health care professional should monitor the patient for which of the following? a. bowel function b. peripheral pulses c. urine output d. level of consciousness

c. urine output Rational: Gentamicin, can cause nephrotoxicity. The health care professional should monitor BUN and creatinine and for an increased output of diluted urine. It is also essential to monitor serum gentamicin levels and maintain a therapeutic range. ATI Infection

3. A nursing student is caring for a patient who has been taking morphine sulfate for pain for 2 weeks. The nursing student shows an understanding of pharmacodynamic tolerance by describing it to the instructor in what way? a. "It is a form of tolerance that is a reduction in drug responsiveness brought on by repeated dosing over a short period." b. "It affects the minimum effective concentration." c. "It is a drug response caused by psychologic factors, not by biochemical or physiological properties." d. "It is a condition in which the patient requires increased doses of morphine sulfate to achieve pain relief."

d. "It is a condition in which the patient requires increased doses of morphine sulfate to achieve pain relief." Pharmacodynamic tolerance is the phenomenon of decreased responsiveness to a drug as a result of repeated drug administration. Tachyphylaxis is a reduction in drug responsiveness as a result of repeated dosing over a short time. Metabolic tolerance results from accelerated drug metabolism and not repeated dosages; therefore, the minimum effective concentration is not affected. The placebo effect is a drug response caused by psychologic factors and not by the drug's biochemical or physiologic properties. Chapter 8

9. A nurse instructs the parent of a child with influenza that which medication or medications may be used safely to reduce fever? a. Ibuprofen [Advil] b. Naproxen [Aleve] c. Aspirin [Bayer] d. Acetaminophen [Tylenol]

d. Acetaminophen [Tylenol] The use of NSAIDs, which include ibuprofen [Advil], naproxen [Aleve], indomethacin [Indocin], and especially aspirin [Bayer], by children with influenza or chickenpox may precipitate Reye's syndrome. This is a potentially fatal multisystem organ disease. Acetaminophen [Tylenol] may be used safely to reduce fever in children with influenza. Indomethacin [Indocin] Chapter 71

6. Which nursing action results in the most common cause of fatal medication errors? a. Miscalculation of dosage b. Miscommunication of drug orders c. Misreading of the healthcare provider's handwriting d. Administering a drug intravenously (IV) instead of intramuscularly (IM)

d. Administering a drug intravenously (IV) instead of intramuscularly (IM) According to the Institute of Medicine (IOM), among fatal medication errors, the most common cause is human factors. Of the human factors, the main causes of errors are performance deficits (29.8%), such as administering a drug IV instead of IM, followed by knowledge deficits (14.2%) and miscalculation of dosage (13%). Chapter 7

5. A patient taking a glucocorticoid for arthritis reports feeling bloated. The nurse notes edema of the patient's hands and feet. Which action by the nurse is correct? a. Instruct the patient to stop taking the drug immediately b. Obtain a blood glucose level c. Tell the patient to limit potassium intake d. Ask the patient about sodium intake

d. Ask the patient about sodium intake Quiz 3 Inflammation and Immunity

10. A nurse instructs a patient to discontinue the scheduled use of high-dose aspirin before undergoing which procedures? (Select all that apply.) a. Routine dental cleaning b. Removal of a skin mole c. Cataract surgery d. Cholecystectomy Correct e. Hysterectomy Correct

d. Cholecystectomy e. Hysterectomy Aspirin promotes bleeding by causing irreversible suppression of platelet aggregation. High-dose aspirin should be discontinued 1 week before elective surgery (cholecystectomy, hysterectomy). There is no need to stop aspirin before procedures with a low risk of bleeding, such as dental cleaning or dermatologic or cataract surgery. Chapter 71

5. The nurse is caring for a patient prescribed abacavir [Ziagen] to treat human immunodeficiency virus (HIV) infection. To reduce the potential fatal hypersensitivity reaction, which recommendation is suggested prior to initiating abacavir drug therapy? a. Administer a test dose b. Obtain liver function studies c. Drug skin testing d. Genetic screening

d. Genetic screening Gene variants that affect the immune system can increase the risk of severe hypersensitivity reaction to the drug abacavir [Ziagen]. The FDA recommends screening for the variant gene code HLA-B5701 before a patient uses this drug. If the test is positive, abacavir should be avoided. Chapter 8

1. The nurse is preparing to administer a dose of penicillin. Before administering the medication, the nurse assesses the patient's allergy history. Which aspect of drug therapy does this represent? a. Making PRN (as needed) decisions b. Evaluating therapeutic effects c. Ensuring proper dosage d. Identifying high-risk patients

d. Identifying high-risk patients Patients receiving penicillin are at high risk for dangerous allergic reactions. This intervention represents the nurse's role in identifying situations with high risk. This situation does not represent the remaining responses. Chapter 2

2. The nurse is developing a plan to minimize the risk of adrenal insufficiency for a patient who is receiving long-term glucocorticoid therapy. Which outcome should be included? a. Increases daily intake of sodium for vascular expansion b. Tapers blood pressure medications to avoid hypotension c. Obtains periodic ultrasound scans of adrenal glands d. Increases or supplements dosage of glucocorticoid at times of stress

d. Increases or supplements dosage of glucocorticoid at times of stress Exogenous steroids inhibit the synthesis and release of endogenous steroids by the adrenals, and recovery is variable, taking from days to a year. Failure to increase or supplement doses at times of stress may be life threatening. Increasing sodium intake and tapering blood pressure medications could cause harm. Adrenal gland ultrasound scans are not a valid way to minimize adrenal insufficiency. Chapter 72

10. The nurse is preparing to administer a drug with a high therapeutic index. What does this mean about the drug? a. It is very potent. b. It has a low efficacy. c. It is probably lethal. d. It is relatively safe.

d. It is relatively safe. Therapeutic index is a measure of a drug's safety. It is the ratio between the therapeutic and lethal dose of the drug. A large or high therapeutic index indicates that the drug is relatively safe. Chapter 5

4. A patient takes glucocorticoids and digoxin [Lanoxin]. It is most important for the nurse to monitor which electrolyte? a. Calcium b. Magnesium c. Sodium d. Potassium

d. Potassium Because of the mineralocorticoid activity of sodium and water retention and potassium loss, glucocorticoids can increase the risk of toxicity from digoxin. They also can exacerbate hypokalemia caused by thiazide and loop diuretics. Calcium, magnesium, and sodium do not require more frequent monitoring as a result of digoxin and glucocorticoid use. Chapter 72

5. The nurse identifies which as the goal of drug therapy in the treatment of patients? a. Cure of the disease b. Follow-up with the prescriber c. Correct administration technique d. Production of maximum benefit with minimum harm

d. Production of maximum benefit with minimum harm The goal of drug therapy in the treatment of patients is the production of maximum benefit with minimum harm. The other options are not the main goal of drug therapy. Chapter 2

5. The nurse should provide which teaching point when administering an enteric-coated oral tablet to a patient? a. "Chew the tablet before swallowing." b. "Break the tablet in half before swallowing." c. "Allow the tablet to be absorbed under the tongue." d. Swallow the tablet whole after double-checking the dose."

d. Swallow the tablet whole after double-checking the dose." Enteric-coated tablets are covered with a material designed to dissolve in the intestine instead of the stomach. They should not be chewed or broken before administration. Sublingual tablets are placed under the tongue for absorption and are not enteric coated. Chapter 4

5. A new medication becomes available for treatment of a debilitating neurologic disease. What would the nurse expect about the safety of the drug? a. The drug is safe for administration to children. b. The drug has been tested in pregnant women. c. All possible adverse effects have been identified by animal testing. d. The drug has passed the FDA approval process.

d. The drug has passed the FDA approval process. A new medication must meet the FDA's stringent requirements before it comes to market. Most drugs have not been tested in women (including pregnant women) or children. Animal testing cannot identify all possible adverse effects. Chapter 3

2. The nurse is preparing to give a drug with certain properties. Which property of the drug is the most compelling indication that it should not be given? a. The drug produces an unwanted side effect. b. The drug is difficult to administer. c. The drug's effects are reversible. d. The drug is not effective for its intended purpose.

d. The drug is not effective for its intended purpose. If a drug is not effective, there is no justification for giving it. Some drugs may be given even though they produce unwanted side effects or are difficult to administer. Reversible action is a desired property for most drugs. Chapter 1

4. A patient takes oxycodone [OxyContin] 40 mg PO twice daily for the management of chronic pain. Which intervention should be added to the plan of care to minimize the gastrointestinal adverse effects? a. The patient should take an antacid with each dose. b. The patient should eat foods high in lactobacilli. c. The patient should take the medication on an empty stomach. d. The patient should increase fluid and fiber in the diet.

d. The patient should increase fluid and fiber in the diet. Narcotic analgesics reduce intestinal motility, leading to constipation. Increasing fluid and fiber in the diet can help manage this adverse effect. If increased fluid and fiber is not sufficiently effective, use of a laxative may be considered. Chapter 28

7. A patient with a chronic illness is on daily glucocorticoids. He is brought to the hospital following a severe motor vehicle accident. He is wearing a medical alert bracelet stating his home medications. Noting that the patient takes glucocorticoids daily, the nurse knows which of the following to be true? a. The patient will need lower doses of glucocorticoids following the trauma. b. The patient will have to stop all glucocorticoids immediately. c. The patient will need the same dose of glucocorticoids he used at home. d. The patient will need higher doses of glucocorticoids following a trauma.

d. The patient will need higher doses of glucocorticoids following a trauma. During times of stress, the adrenal glands normally secrete large amounts of glucocorticoids. If the stress is sufficiently severe (eg, trauma, surgery), these glucocorticoids are essential for supporting life. Accordingly, because of adrenal suppression, it is imperative that patients receiving long-term glucocorticoid therapy be given increased doses at times of stress. Chapter 72

When administering erythromycin to a patient who has pneumococcal pneumonia, the health care professional should monitor for which of the following adverse effects of the drug? a. hypothermia b. blurred vision c. constipation d. cardiac dysrhythmias

d. cardiac dysrhythmias ATI Infection

A primary care provider is considering the various pharmacologic options for a patient who has a gynecologic infection and a history of alcohol use disorder. Which of the following medications can cause a reaction similar to disulfiram (Antabuse) if the patient drinks alcohol while taking it? (Select all that apply.) a. nitrofurantoin b. amoxicillin c. aztreonam d. cefotetan e. metronidazole

d. cefotetan e. metronidazole Rational: Cefotetan, can cause a reaction similar to what disulfiram causes when patients take it and also drink alcohol. This reaction manifests as nausea, severe vomiting, headache, weakness, and hypotension. Metronidazole, an antiparasitic drug, can cause a reaction similar to what disulfiram causes when patients take it and also drink alcohol. ATI Infection

1. When performing a shift assessment, the nurse notes a maculopapular rash over the trunk of a patient who has been taking ceftriaxone for 4 days. The best initial action by the nurse is: a. hold the ceftriaxone b. call the prescriber immediately c. administer epinephrine stat d. complete the assessment

d. complete the assessment Quiz 4 Infection

A primary care provider is prescribing drug therapy for a patient whose sputum culture results indicate methicillin-resistant Staphylococcus aureus (MRSA). Which of the following drugs should be administered? a. Trimethoprim/sulfamethoxazole (Bactrim) b. Tetracycline c. cephalexin d. vancomycin

d. vancomycin Rationale: Vancomycin, a potentially toxic antibiotic, is used primarily for serious infections either in patients who are allergic to penicillin or in those whose infecting bacteria are resistant to penicillin, such as MRSA. ATI Infection


Conjuntos de estudio relacionados

Introduction to Clinical Rad Therapy & Operations Chapter 7

View Set

Absolute advantage and comparitive advantage

View Set

The "Basic Nine" Functions Algebra II Trigonometry

View Set

PL3232 Ch8.1 Rhythms of Waking and Sleeping

View Set

PSY 452 practice questions for final

View Set

Intro to Human communication FINAL

View Set